Med-Surg Exam 3 - Lewis + ATI Chapter 31-37

¡Supera tus tareas y exámenes ahora con Quizwiz!

) are a first-line therapy for this type of angina. Lipid-lowering drugs help reduce atherosclerosis (i.e., plaque formation), and β-adrenergic blockers decrease sympathetic stimulation of the heart (i.e., palpitations). Medications or activities that increase myocardial contractility will increase the incidence of angina by increasing oxygen demand.

...

You are caring for a group of PTs and should recognize which PT is at risk for a dysrhythmia? Select All 1. PT with metabolic acidosis 2. PT with K level of 4.3 3. PT whose O2 sat is 96% 4. COPD PT 5. PT who underwent a stent replacement

1. PT with metabolic acidosis-acid base imbalance = @ risk 4. COPD PT- at risk 5. PT who underwent a stent replacement- at risk

You are to give 0.9NS IV at 50ml/hr to a PT who is receiving hemodynamic monitoring and has an indwelling IV catheter in the left hand. Which sites can it be given in? Select all 1. Peripheral saline lock 2. Port on the arterial line 3. Port on the proximal CVP lumen of the pulmonary artery catheter 4. Port on distal lumen of PA catheter 5. Ballon Inflammation port.

1. Peripheral saline lock 3. Port on the proximal CVP lumen of the pulmonary artery catheter- can be used for fluids also

You are caring for PT with heart failure and Pt asks how should he limit his fluid to 2000ml/day. What is the best response? 1. Pour the amount of fluid you drink into an empty 2 liter bottle to keep track of how much you drink 2. Each glass contains 8 oz there are 30ml per oz so you can have a total of 8 glasses a day 3. the is the same as 2 quarts or about the same as two pots of coffee 4. Take sips of water or ice chips so you will not take too much fluid

1. Pour the amount of fluid you drink into an empty 2 liter bottle to keep track of how much you drink- provides a visual

The RN is discussing the use of cardiopulmonary bypass during surgery for coronary artery bypass graft CABG. Which should be included when discussing? Select all 1. the demand for O2 is lowered 2. Motion of the heart ceases 3. Rewarming the client takes place 4. The PT metabolic rate is increased 5. Blood flow to the heart is stopped.

1. The demand for O2 is lowered-reduces risk of inadequate oxygenation vital organs 2. Motion of the heart ceases- to allow for placement of graft 3. Rewarming the client takes place- its lowered for procedure so this will take place

Which of the following clients has the greatest risk of acquiring rheumatic endocarditis? 1. An older adult who has chronic obstructive pulmonary disease. 2. A child who has an upper respiratory streptococcal infection. 3. A middle aged adult who has lupus erythematous. 4. A young adult who is at 24 weeks of gestation.

2. A child who has an upper respiratory streptococcal infection.- strep places him at highest risk 50% will develop rheumatic fever

You are caring for PT with heart failure and reports increased SOB you increase O2 per protocol. What action do you do next? 1. Obtain weight 2.Assist client to high fowlers 3. Auscultate lung sounds 4. Check O2 sat with pulse ox.

2. Assist client to high fowlers- ABCs will decrease venous return to the heart(preload) and help relieve lung congestion

You are caring for a client in the first 8 hrs following coronary artery bypass graft (CABG) surgery what findings should you report? 1. Mediastinal drainage 100mL/hr 2. BP 160/80 3.Temp-37.1 4. K 3.8

2. BP 160/80-increased vascular pressure can cause bleeding at incision site

PT is prescribed an ace inhibitor. the RN knows the Pt understands the adverse effects when he reports the following? 1. tendon pain 2. Persistent cough 3. Frequent urination 4. constipation

2. Persistent cough- adverse effect of this drug

You are providing discharge teaching to a PT who has a prescription for the transdermal nitroglycerin patch. Which instruction would you include? 1. Apply new patch to the same site as the previous patch. 2. Place the new patch on an area skin away from skin folds and joints 3. Keep the patch on 24 hr per day 4. Replace the patch at the onset of angina

2. Place the new patch on an area skin away from skin folds and joints- should apply to skin that is prone not to move or wrinkle

You are assessing a PT with Left sided heart failure. What manifestation should you expect to find? 1. Increased abdominal girth 2. Weak peripheral pulses 3. Jugular venous neck distention 4. Dependent edema

2. Weak peripheral pulses- r/t decreased CO from LSHF

You are caring for a patient who is scheduled for a CABG in 2hr. Which PT statement needs further clarification? 1. My arthritis is really bothering me b/c i haven't taken my aspirin in a week 2. My BP shouldn't be high b/c i took my BPmeds this morning 3. I took my warfarin last night according to my schedule. 4. I will check my blood sugar b/c i took a reduced dose of insulin this morning

3. I took my warfarin last night according to my schedule.- should have stopped 5-7 days prior for bleeding

A nurse on a cardiac unit is reviewing the laboratory findings of a client who has a diagnosis of MI and reports that his dyspnea began 2 weeks ago. Which of the following cardiac enzymes would conform the infraction occurred 14 days ago? 1. CK-MB 2. Troponin I 3. Troponin T 4. Myoglobin

3. Troponin T- level will still be evident 14-21 days after MI **1. CK-MB- no longer evident after 3 days 2. Troponin I- no longer evident after 7 days 4. Myoglobin- no longer evident after 24hr

You are caring for a PT in the 1st hr following an aortic aneurysm repair. Which of the following findings indicate shock and should be reported. 1. Serosanguineous drainage on dressing 2. Severe pain with coughing 3. Urinary output of 20ml/hr 4. Increase temp from 36.8 to 37.5

3. UOP of 20ml/hr- should be 30ml/hr can indicate shock r/t hemorrhage

A nurse is caring for a client who has hemophilia. The nurse should anticipate a prescription from the provider for which of the following tests? 1. RBC 2. TIBC 3. aPIT 4. MCH

3. aPIT- checks clotting factors **1. RBC-identfies the presence of anemia not clotting 2. TIBC-identfies iron deficiency anemia not clotting 4. MCH- indicates the presence of anemia

The RN is teaching a student on the care of a PT who is receiving hemodynamic monitoring. Which statement indicates teaching was effective? 1. Air should be instilled into the monitoring system 2. The client should be in the prone position 3. The transducer should be level with the 2nd intercostal space 4. A chest X-ray is needed to verify placement.

4. A chest X-ray is needed to verify placement.

A nurse is caring for a client who has pericarditis. Which of the following expected findings should the nurse anticipate? 1. Petechiae 2. Murmur 3. Rash 4. Friction rub

4. Friction rub- can be heard during auscultation of a client who has pericarditis

You are preparing PT for coronary angiography. You should report what finding to the provider prior to procedure? 1. Hgb 14.4 2. HX: PAD peripheral arterial disease 3. UOP of 200ml in 4 hr 4. Previous allergic reaction to shell fish

4. Previous allergic reaction to shell fish- shellfish reaction will often react to iodine

When the patient is being examined for venous thromboembolism (VTE) in the calf, what diagnostic test should the nurse expect to teach the patient about first? A) Duplex ultrasound B) Contrast venography C) Magnetic resonance venography D) Computed tomography venography

A) Duplex ultrasound

The nurse obtains a blood pressure of 176/82 mm Hg for a patient. What is the patient's mean arterial pressure (MAP)?

ANS: 113 mm Hg MAP = (SBP + 2 DBP)/3

The nurse obtains a blood pressure of 176/82 mm Hg for a patient. What is the patient's mean arterial pressure (MAP)?

ANS: 113 mm Hg MAP = (SBP + 2 DBP)/3 DIF: Cognitive Level: Apply (application)

The nurse obtains a blood pressure of 176/83 mm Hg for a patient. What is the patient's mean arterial pressure (MAP)?

ANS: 114 mm Hg MAP = (SBP + 2 DBP)/3

When evaluating the discharge teaching for a patient with chronic peripheral artery disease (PAD), the nurse determines a need for further instruction when the patient says, "I will a. use a heating pad on my feet at night to increase the circulation." b. buy some loose clothes that do not bind across my legs or waist." c. walk to the point of pain, rest, and walk again for at least 30 minutes 3 times a week." d. change my position every hour and avoid long periods of sitting with my legs crossed."

ANS: A Because the patient has impaired circulation and sensation to the feet, the use of a heating pad could lead to burns. The other patient statements are correct and indicate that teaching has been successful.

10. When auscultating over the patients abdominal aorta, the nurse hears a humming sound. The nurse documents this finding as a a. thrill. b. bruit. c. murmur. d. normal finding.

b. bruit.

5. To determine the effects of therapy for a patient who is being treated for heart failure, which laboratory result will the nurse plan to review? a. Troponin b. Homocysteine (Hcy) c. Low-density lipoprotein (LDL) d. B-type natriuretic peptide (BNP)

d. B-type natriuretic peptide (BNP)

When providing dietary instruction to a patient with hypertension, the nurse would advise the patient to restrict intake of which meat? A Broiled fish B Roasted duck C Roasted turkey D Baked chicken breast

B Roasted duck Roasted duck is high in fat, which should be avoided by the patient with hypertension. Weight loss may slow the progress of atherosclerosis and overall CVD risk. The other meats are lower in fat and are therefore acceptable in the diet.

A 39-yr-old woman with a history of smoking and oral contraceptive use is admitted with a venous thromboembolism (VTE) and prescribed unfractionated heparin. What laboratory test should the nurse review to evaluate the expected effect of the heparin? A) Platelet count B) Activated clotting time (ACT) C) International normalized ratio (INR) D) Activated partial thromboplastin time (APTT)

D) Activated partial thromboplastin time (APTT)

A patient was admitted for possible ruptured aortic aneurysm. No back pain was reported. Ten minutes later, the nurse notes sinus tachycardia 138 beats/min, blood pressure is palpable at 65 mm Hg, increasing waist circumference, and no urine output. How should the nurse interpret the findings? A) Tamponade will soon occur. B) The renal arteries are involved. C) Perfusion to the legs is impaired. D) Bleeding into the abdomen is likely.

D) Bleeding into the abdomen is likely.

The patient reports tenderness when she touches her leg over a vein. The nurse assesses warmth and a palpable cord in the area. The nurse knows the patient needs treatment to prevent which sequela? A) Pulmonary embolism B) Pulmonary hypertension C) Post-thrombotic syndrome D) Venous thromboembolism

D) Venous thromboembolism

The nurse obtains the following information from a patient newly diagnosed with prehypertension. Which finding is most important to address with the patient? a. Low dietary fiber intake b. No regular physical exercise c. Drinks a beer with dinner every night d. Weight is 5 pounds above ideal weight

ANS: B The recommendations for preventing hypertension include exercising aerobically for 30 minutes most days of the week. A weight that is 5 pounds over the ideal body weight is not a risk factor for hypertension. The Dietary Approaches to Stop Hypertension (DASH) diet is high in fiber, but increasing fiber alone will not prevent hypertension from developing. The patient's alcohol intake is within guidelines and will not increase the hypertension risk.

The nurse obtains the following information from a patient newly diagnosed with prehypertension. Which finding is most important to address with the patient? a. Low dietary fiber intake b. No regular physical exercise c. Drinks a beer with dinner every night d. Weight is 5 pounds above ideal weight

ANS: B The recommendations for preventing hypertension include exercising aerobically for 30 minutes most days of the week. A weight that is 5 pounds over the ideal body weight is not a risk factor for hypertension. The Dietary Approaches to Stop Hypertension (DASH) diet is high in fiber, but increasing fiber alone will not prevent hypertension from developing. The patient's alcohol intake is within guidelines and will not increase the hypertension risk. DIF: Cognitive Level: Analyze (analysis)

Which actions could the nurse delegate to unlicensed assistive personnel (UAP) who are providing care for a patient who is at risk for venous thromboembolism? a. Monitor for any bleeding after anticoagulation therapy is started. b. Apply sequential compression device whenever the patient is in bed. c. Ask the patient about use of herbal medicines or dietary supplements. d. Instruct the patient to call immediately if any shortness of breath occurs.

ANS: B UAP training includes the use of equipment that requires minimal nursing judgment, such as sequential compression devices. Patient assessment and teaching require more education and critical thinking and should be done by the registered nurse (RN).

Which diagnostic test will be most useful to the nurse in determining a patient admitted with acute shortness of breath has heart failure? a. Serum troponin b. Arterial blood gases c. B-type natriuretic peptide d. 12-lead electrocardiogram

ANS: C BNP is secreted when ventricular pressures increase, as they do with heart failure. Elevated BNP indicates a probable or very probable diagnosis of heart failure. A 12-lead ECG, ABG, and troponin may also be used to determine the causes of effects of heart failure but are not as clearly diagnostic of heart failure as BNP.

17. A patient is recovering from a myocardial infarction (MI) and develops chest pain on day 3 that increases when taking a deep breath and is relieved by leaning forward. Which action should the nurse take next? a. Assess the feet for pedal edema. b. Palpate the radial pulses bilaterally. c. Auscultate for a pericardial friction rub. d. Check the heart monitor for dysrhythmias.

ANS: C The patient's symptoms are consistent with the development of pericarditis, a possible complication of MI. The other assessments listed are not consistent with the description of the patient's symptoms.

While doing the hospital admission assessment for a thin older adult, the nurse observes pulsation of the abdominal aorta in the epigastric area. Which action should the nurse take next? a. Teach the patient about aneurysms. b. Notify the hospital rapid response team. c. Instruct the patient to remain on bed rest. d. Document the finding in the patient chart.

ANS: D Visible pulsation of the abdominal aorta is commonly observed in the epigastric area for thin individuals. The nurse should simply document the finding in the admission assessment. Unless there are other abnormal findings (such as a bruit, pain, or hyper/hypotension) associated with the pulsation, the other actions are not necessary.

4. Which information given by a patient admitted with chronic stable angina will help the nurse confirm this diagnosis? a. The patient states that the pain "wakes me up at night." b. The patient rates the pain at a level 3 to 5 (0 to 10 scale). c. The patient states that the pain has increased in frequency over the last week. d. The patient states that the pain "goes away" with one sublingual nitroglycerin tablet.

ANS: D Chronic stable angina is typically relieved by rest or nitroglycerin administration. The level of pain is not a consistent indicator of the type of angina. Pain occurring at rest or with increased frequency is typical of unstable angina.

The nurse obtains a health history from a 65-year-old patient with a prosthetic mitral valve who has symptoms of infective endocarditis (IE). Which question by the nurse is most appropriate? a. "Do you have a history of a heart attack?" b. "Is there a family history of endocarditis?" c. "Have you had any recent immunizations?" d. "Have you had dental work done recently?"

ANS: D Dental procedures place the patient with a prosthetic mitral valve at risk for infective endocarditis (IE). Myocardial infarction (MI), immunizations, and a family history of endocarditis are not risk factors for IE.

What is Pulsus Paradoxus?

When the systolic BP is 10 points higher on expiration than on inspiration **This is indicative of cardiac tamponade

20. A transesophageal echocardiogram (TEE) is ordered for a patient with possible endocarditis. Which action included in the standard TEE orders will the nurse need to accomplish first? a. Start an IV line. b. Place the patient on NPO status. c. Administer O2 per nasal cannula. d. Give lorazepam (Ativan) 1 mg IV.

b. Place the patient on NPO status.

When the nurse is screening patients for possible peripheral arterial disease, indicate where the posterior tibial artery will be palpated. (imagine picture) a. 1 pelvic b. 2. knee c. 3 ankle d. 4 pedal

c. 3 ankle The posterior tibial site is located behind the medial malleolus of the tibia.

22. The nurse is reviewing the laboratory results for newly admitted patients on the cardiovascular unit. Which patient laboratory result is most important to communicate as soon as possible to the health care provider? a. Patient whose triglyceride level is high b. Patient who has very low homocysteine level c. Patient with increase in troponin T and troponin I level d. Patient with elevated high-sensitivity C-reactive protein level

c. Patient with increase in troponin T and troponin I level

11. The nurse has received the laboratory results for a patient who developed chest pain 4 hours ago and may be having a myocardial infarction. The most important laboratory result to review will be a. myoglobin b. low-density lipoprotein (LDL) cholesterol. c. troponins T and I. d. creatine kinase-MB (CK-MB).

c. troponins T and I.

2. When reviewing the 12-lead electrocardiograph (ECG) for a healthy 79-year-old patient who is having an annual physical examination, what will be of most concern to the nurse? a. The PR interval is 0.21 seconds. b. The QRS duration is 0.13 seconds. c. There is a right bundle-branch block. d. The heart rate (HR) is 42 beats/minute.

d. The heart rate (HR) is 42 beats/minute.

3. During a physical examination of a 74-year-old patient, the nurse palpates the point of maximal impulse (PMI) in the sixth intercostal space lateral to the left midclavicular line. The most appropriate action for the nurse to take next will be to a. ask the patient about risk factors for atherosclerosis. b. document that the PMI is in the normal anatomic location. c. auscultate both the carotid arteries for the presence of a bruit. d. assess the patient for symptoms of left ventricular hypertrophy.

d. assess the patient for symptoms of left ventricular hypertrophy.

1. After noting a pulse deficit when assessing a 74-year-old patient who has just arrived in the emergency department, the nurse will anticipate that the patient may require a. emergent cardioversion. b. a cardiac catheterization. c. hourly blood pressure (BP) checks. d. electrocardiographic (ECG) monitoring.

d. electrocardiographic (ECG) monitoring.

9. The nurse teaches the patient being evaluated for rhythm disturbances with a Holter monitor to a. connect the recorder to a computer once daily. b. exercise more than usual while the monitor is in place. c. remove the electrodes when taking a shower or tub bath. d. keep a diary of daily activities while the monitor is worn.

d. keep a diary of daily activities while the monitor is worn.

In teaching a patient with hypertension about controlling the condition, the nurse recognizes that: a. all patients with elevated BP require medication b. obese persons must achieve a normal weight to lower BP c. It is not necessary to limit salt in the diet if taking a diuretic d. lifestyle modifications are indicated for all persons with elevated BP

d. lifestyle modifications are indicated for all persons with elevated BP

for the patient and in counseling other family members. The patient should be counseled against the use of stimulant drugs, but the limited past history indicates that the patient is not at current risk for cocaine use. Viral infections and CAD are risk factors for dilated cardiomyopathy, but not for HC.

...

PT has complete Heart block HR 34, BP 83/48, he is lethargic, and unable to complete sentences. Which action should you do first? 1. Clean skin with soap and water 2. Prepare PT for insertion of permanent pacemaker 3. Obtain signed informed consent form for a pacemaker 4. Apply transcutaneous pacemaker pads.

1. Clean skin with soap and water- need to do this before applying transcutaneous pads

You are caring for a PT receiving hemodynamic monitoring and has the following readings: PAS 34, PAD 21, PAWP 16, CVP 12. which is the PT at risk for? Select All 1. Heart failure 2. Cor pulmonale 3. Hypovolemic shock 4. Pulmonary hypertension 5. Peripheral edema

1. Heart failure-r/t left ventricular failure=high hemodynamic readings 2. Cor pulmonale r/t right side of heart/pulmonary problems= high readings 4. Pulmonary hypertension- would present with high readings 5. Peripheral edema - would indicate left ventricular failure r/t high readings

You are caring for a PT following insertion of a permanent pacemaker. Which PT statement indicates a potential complication of the insertion procedure? 1. I can't get rid of these hiccups 2. I feel dizzy when i stand 3. My incision stings 4. i have a headache

1. I can't get rid of these hiccups-can indicate the pacemaker is stimulating the chest wall or diaphragm indicating wire lead perforation

You are caring for a PT with dilated cardiomyopathy. PT reports increasing difficulty completing her daily 1 mile walks. You recognize that this is a finding of? 1. left ventricular failure 2.Peripheral vasodilation 3.Pericardial effusion 4. decreased vascular volume

1. Left ventricular failure- activity intolerance is a finding of left ventricular failure r/t dilated cardiomyopthy

You are planning a presentation about HTN which lifestyle modifications would you include? Select All 1. Limit alcohol intake 2. Regular exercise program 3. Decreased mag intake 4. Reduced K intake 5. Smoking cessation

1. Limit alcohol intake- Should limit if HTN 2. Regular exercise program- will reduce HTN 5. Smoking cessation- exacerbates HTN

You are caring for a PT who is experiencing Afib. You should report what finding to the provider? 1. Slurred speech 2. Irregular pulse 3. Dependent edema 4. Persistant fatigue

1. Slurred speech- greatest risk is an embolus and this can indicate embolus

Pt is being evaluated for possible valvular heart disease. What are risk factors for this condition? Select all 1. Surgical repair of an arterial septal defect at age 2 2. Measles infection in childhood 3. HTN for 5 yrs 4. Weight gain of 10lb in past year 5. Diastolic murmur present

1. Surgical repair of an arterial septal defect at age 2-congential malformations= risk 3. HTN for 5 yrs- HTN=risk 5. Diastolic murmur present- indicates turbulent blood flow which is often caused by valvular disease

Rn admitted a PT following placement of a temporary pacemaker. Which of the following actions should the nurse use to promote safety? Select all 1. Wear gloves when handling pacemaker leads 2. Verify the use of three pronged grounding plugs 3. Minimize PT shoulder movements 4. Keep the lead wires taut when turning the client 5. additional batteries should be kept at the RN's station

1. Wear gloves when handling pacemaker leads- gloves should be worn 2. Verify the use of three pronged grounding plugs-reduce the risk of accidental electrical charge 3. Minimize PT shoulder movements- minimze or wear a sling to promote secure anchoring of the lead wires

You are providing teaching to a PT with HF you would instruct him to report what finding? 1. Weight gain of 2lbs in 24hr 2. Increase in 10mmhg in Systolic BP 3. Dyspnea with exertion 4. Dizziness when rising quickly

1. Weight gain of 2lbs in 24hr- fluid retention r/t worsen HF

A nurse is reviewing clinical manifestations of a theoretic aortic aneurysm with a newly hired nurse. Which of the following should the nurse include in the discussion? (select all that apply) 1. cough 2. shortness of breath 3. upper chest pain 4. diaphoresis 5. altered swallowing

1. cough-manifestation 2. shortness of breath-manifestion 5. altered swallowing - manifestation of thoracic aortic aneurysm

A nurse in a clinic is caring for a client who has been on long-term NSAID therapy to treat myocarditis. Which of the laboratory findings should be reported to the provider? 1. platelets 100,000 2. serum glucose 110 3. serum creatinine 0.7 4. amino alanine transferase (ALT) 30

1. platelets 100,000-long term NSAIDs can lower Plts

You are screening for HTN which action would increase the risk for HTN? Select all 1. drinkning 8oz of nonfat milk daily 2. Eating popcorn at movie theatre 3. Walking 1 mile daily at 12min/mile 4. consuming 36oz of beer daily 5. getting a massage once a week

2. Eating popcorn at movie theatre contains high Na and Fat- high risk 4. consuming 36oz of beer daily- more than 24oz/day can lead to weight gain-high risk

You are completing discharge teaching to a PT with a mechanical heart valve what demonstrates understanding? 1.I will be glad to get back to my exercise routine right away. 2.I will have my prothrombin time checked on a regular basis 3. I will talk to my dentist about no longer needing antibiotics before dental exams 4. I will continue to limit my intake of foods containing K

2. I will have my prothrombin time checked on a regular basis

You are caring for a PT after CABG hemodynamic monitoring has been initiated what action would facilitate correct monitoring readings? Select all 1. Place the PT in high Fowlers 2. Level transducer to phlebostatic axis 3. Zero Transducer to room air 4. Observe trends in readings 5. compare readings to physical assessment.

2. Level transducer to phlebostatic axis 3. Zero Transducer to room air 4. Observe trends in readings 5. compare readings to physical assessment.

You are caring for a PT who is being treated for HF and has prescriptions for digoxin and furosemide. You plan to monitor for which of the following as an adverse effect of these meds? 1. SOB 2. Lightheadedness 3. Dry cough 4. Metallic taste

2. Lightheadedness- can cause a sudden drop in BP= lightheadedness

You are providing teaching to a group of PTs which of the Pts is at risk for developing PAD? 1. PT with hypothroidism 2. PT with Diabetes Mellitus 3. PT whose daily diet consists of 25% fat 4. Pt who consumes 2 bottles of beer a day

2. PT with Diabetes Mellitus-at risk for microvascular and progressive peripheral arterial disease

A nurse is planning care for a client who has septic shock. Which of the following is the priority action for the nurse to take? 1. maintaining adequate fluid volume with IV infusions 2. administering antibiotic therapy 3. monitoring hemodynamic status 4. administering vasopressor medication

2. administering antibiotic therapy-eliminating endotoxins/bacteria will reduce vasodilation

A nurse in a cardiac unit is assisting with the admission of a client who is to undergo hemodynamic monitoring. which of the following actions should the nurse anticipate preforming? 1. Administer large volumes of IV fluids 2. assist with insertion of pulmonary artery catheter 3. obtain doppler pulses of the extremities 4. gather supplies for insertion of a peripheral IV catheter

2. assist with insertion of pulmonary artery catheter- used for hemodynamic monitoring **1. Administer large volumes of IV fluids- potency is maintained with slow cont. fluids. 3. obtain doppler pulses of the extremities-ECG is done prior 4. gather supplies for insertion of a peripheral IV catheter- would need an arterial line for ABGs and blood samples for hemodynamic monitoring

A nurse in the emergency department is completing an assessment of a client who is in shock. which of the following findings should the nurse expect? (select all that apply) 1. heart rate 60/min 2. seizure activity 3. respiratory rate 42/min 4. increase urine output 5. weak, thready pulse

2. seizure activity- may be present 3. respiratory rate 42/min-Exepected 5. weak, thready pulse- Expected **1. heart rate 60/min- would be tachycardic 4. increase urine output- would be decreased

You are monitoring a PT following coronary bypass graft surgery (CABG). Which finding indicates cardiac tamponade? 1. Sternal instability 2. Increased WBC count 3. BP of 140/82 on inspiration and 154/90 on expiration 4. Sinus rhythm with occasional premature atrial contractions and HR 88

3. BP of 140/82 on inspiration and 154/90 on expiration= pulses paradoxus

You are doing discharge teaching with PT who has a permanent pacemaker. Which statement indicates a need for further teaching? 1. I will notify the airport screeners about my pacemaker 2. I will call my doctor about hiccups 3. I will have to disconnect my garage door opener 4. I will take my HR every morning

3. I will have to disconnect my garage door opener-household appliances do not affect it

You are caring for PT with BP of 254/139, you recognize the PT is in a hypertensive crisis. Which action should you take first? 1. Obtain blood samples for testing 2. Tell the PT to report vision changes 3. Place HOB at 45 degrees 4. Initiate IV access

3. Place HOB at 45 degrees-ABC's this will promote respiratory status and promote venous return reducing workload on heart **Initiate IV access- not first action

A nurse is discussing a new diagnosis of an aneurysm with a client. The client asks the nurse to explain what causes an aneurysm to rupture. Which of the following is an appropriate response by the nurse? 1. "The wall of and artery becomes thin and flexible." 2. "It is due to turbulence in blood flow in the artery." 3. "It is due to abdominal enlargement." 4. "It is due to hypertension."

4. "It is due to hypertension."- increases pressure within walls = rupture **1. "The wall of and artery becomes thin and flexible."- ruptures b/c thickening and lack of elasticity 2. "It is due to turbulence in blood flow in the artery."- indicates the presence of an aneurysm not a rupture 3. "It is due to abdominal enlargement."-not the cause but may occur

A nurse is instructing a client who has angina about a new prescription for metoprolol tartrate. which of the following statements by the client indicates understanding of the teaching? 1. "I should place the tablet under my tongue." 2. "I should have my clotting time checked weekly." 3. "I will report any ringing in my ears." 4. "I will call my doctor if my pulse is less than 60."

4. I will call my doctor if my pulse is less than 60.

PT is on warfarin r/t to a DVT which finding provides evidence of effectiveness? 1. Hgb 14 2.Minimal bruising 3. Reduced circumference of affected extremity 4. INR 2.5

4. INR 2.5-is within desired therapeutic range this is best evidence

You are caring for a PT following an abdominal aortic aneurysm resection. Which of the following is priority assessment? 1. Neck vein distention 2. Bowel sounds 3. Peripheral edema 4. Urinary output

4. UOP- greatest risk is graft occlusion or rupture which will reflect blood flow in the kidneys

You are caring for a PT who has endocarditis. Which finding should you recognize as a complication? 1. Ventricular depolarization 2. Guillain Barre Syndrome 3. Myelodysplastic syndrome 4. Valvular disease

4. Valvular disease- damage will occur as a result of inflammation or infection of the endocardium

You are teaching a PT the importance of remaining still following angiography. Which statement is the best to say? 1. Moving in bed raises your BP 2. Too much activity increases your risk for infection. 3. moving in bed increases your risk of a complication due to anesthesia 4. too much activity places you at risk for bleeding

4. too much activity places you at risk for bleeding

Despite a high dosage, a male patient who is taking nifedipine (Procardia XL) for antihypertensive therapy continues to have blood pressures over 140/90 mmHg. What should the nurse do next? A Assess his adherence to therapy. B Ask him to make an exercise plan. C Instruct him to use the DASH diet. D Request a prescription for a thiazide diuretic.

A Assess his adherence to therapy. A long-acting calcium-channel blocker such as nifedipine causes vascular smooth muscle relaxation resulting in decreased SVR and arterial BP and related side effects. The patient data the nurse has about this patient is very limited, so the nurse needs to assess his adherence to therapy.

The nurse teaches a patient with hypertension that uncontrolled hypertension may damage organs in the body primarily by which mechanism? A Hypertension promotes atherosclerosis and damage to the walls of the arteries. Hypertension causes direct pressure on organs, Bresulting in necrosis and replacement of cells with scar tissue. C Hypertension causes thickening of the capillary membranes, leading to hypoxia of organ systems. D Hypertension increases blood viscosity, which contributes to intravascular coagulation and tissue necrosis distal to occlusions.

A Hypertension promotes atherosclerosis and damage to the walls of the arteries. Hypertension is a major risk factor for the development of atherosclerosis by mechanisms not yet fully known. However, once atherosclerosis develops, it damages the walls of arteries and reduces circulation to target organs and tissues.

The nurse is caring for a patient with hypertension who is scheduled to receive a dose of esmolol (Brevibloc). The nurse should withhold the dose and consult the prescribing physician for which vital sign taken just before administration? A Pulse 48 B Respirations 24 C Blood pressure 118/74 D Oxygen saturation 93%

A Pulse 48 Because esmolol is a β1-adrenergic blocking agent, it can cause hypotension and bradycardia as adverse effects. The nurse should withhold the dose and consult with the health care provider for parameters regarding pulse rate limits.

Which person should the nurse identify as having the highest risk for abdominal aortic aneurysm? A) A 70-yr-old man with high cholesterol and hypertension B) A 40-yr-old woman with obesity and metabolic syndrome C) A 60-yr-old man with renal insufficiency who is physically inactive D) A 65-yr-old woman with hyperhomocysteinemia and substance abuse

A) A 70-yr-old man with high cholesterol and hypertension

The patient has chronic venous insufficiency and a venous ulcer. The unlicensed assistive personnel (UAP) decides to apply compression stockings because that is what patients 'always' have ordered. Which assessment finding would indicate the application of compression stockings could harm the patient? A) Rest pain B) High blood pressure C) Elevated blood sugar D) Dry, itchy, flaky skin

A) Rest pain

The nurse is admitting a 68-yr-old preoperative patient with a suspected abdominal aortic aneurysm (AAA). The medication history reveals that the patient has been taking warfarin (Coumadin) on a daily basis. Based on this history and the patient's admission diagnosis, the nurse should prepare to administer which medication? A) Vitamin K B) Cobalamin C) Heparin sodium D) Protamine sulfate

A) Vitamin K

The patient had myocarditis and is now experiencing fatigue, weakness, palpitations, and dyspnea at rest. The nurse assesses pulmonary crackles, edema, and weak peripheral pulses. Sinoatrial tachycardia is evident on the cardiac monitor. The Doppler echocardiography shows dilated cardiomyopathy. What collaborative and nursing care of this patient should be done to improve cardiac output and the quality of life? (Select all that apply.) A. Decrease preload and afterload. B. Relieve left ventricular outflow obstruction. C. Control heart failure by enhancing myocardial contractility. D. Improve diastolic filling and the underlying disease process. E. Improve ventricular filling by reducing ventricular contractility

A. Decrease preload and afterload. C. Control heart failure by enhancing myocardial contractility. The patient is experiencing dilated cardiomyopathy. To improve cardiac output and quality of life, drug, nutrition, and cardiac rehabilitation will be focused on controlling heart failure by decreasing preload and afterload and improving cardiac output, which will improve the quality of life. Relief of left ventricular outflow obstruction and improving ventricular filling by reducing ventricular contractility is done for hypertrophic cardiomyopathy. There are no specific treatments for restrictive cardiomyopathy, but interventions are aimed at improving diastolic filling and the underlying disease process

While doing an admission assessment, the nurse notes clubbing of the patient's fingers. Based on this finding, the nurse will question the patient about which disease process? A. Endocarditis B. Acute kidney injury C. Myocardial infarction D. Chronic thrombophlebitis

A. Endocarditis Clubbing of the fingers is a loss of the normal angle between the base of the nail and the skin. This finding can be found in endocarditis, congenital defects, and/or prolonged oxygen deficiency. Clinical manifestations of acute kidney injury, myocardial infarction, and chronic thrombophlebitis will not include clubbing of the fingers.

When caring for a patient with infective endocarditis, the nurse will assess the patient for which vascular manifestations (select all that apply)? A. Osler's nodes B. Janeway's lesions C. Splinter hemorrhages D. Subcutaneous nodules E. Erythema marginatum lesions

A. Osler's nodes B. Janeway's lesions C. Splinter hemorrhages Osler's nodes, Janeway's lesions, and splinter hemorrhages are all vascular manifestations of infective endocarditis. Subcutaneous nodules and erythema marginatum lesions occur with rheumatic fever.

The nurse is teaching a community group about preventing rheumatic fever. What information should the nurse include? A. Prompt recognition and treatment of streptococcal pharyngitis B. Completion of 4 to 6 days of antibiotic therapy for infective endocarditis of respiratory infections in children born with heart defects C. Avoidance of respiratory infections in children who have rheumatoid arthritis D. Requesting antibiotics before dental surgery for individuals with rheumatoid arthritis

A. Prompt recognition and treatment of streptococcal pharyngitis The nurse should emphasize the need for prompt and adequate treatment of streptococcal pharyngitis infection, which can lead to the complication of rheumatic fever.

While admitting a patient with pericarditis, the nurse will assess for what manifestations of this disorder? A. Pulsus paradoxus B. Prolonged PR intervals C. Widened pulse pressure D. Clubbing of the fingers

A. Pulsus paradoxus Pericarditis can lead to cardiac tamponade, an emergency situation. Pulsus paradoxus greater than 10 mm Hg is a sign of cardiac tamponade that should be assessed at least every 4 hours in a patient with pericarditis. Prolonged PR intervals occur with first-degree AV block. Widened pulse pressure occurs with valvular heart disease. Clubbing of fingers may occur in subacute forms of infective endocarditis and valvular heart disease.

What is some expected assessment findings of cariogenic shock? Select All 1. tachycardia 2. hypotension 3.crackles 4. angina 5. cool and clammy skin

ALL OF THEM **Cardiogenic shock- complication of pump failure that occurs commonly after MI and injury greater than 40% to the left ventricle. - treat with vasopressors and interpose (digoxin) to increase CO to maintain organ perfusion

When analyzing an electrocardiographic (ECG) rhythm strip of a patient with a regular heart rhythm, the nurse counts 30 small blocks from one R wave to the next. The nurse calculates the patient's heart rate as ____.

ANS: 50 There are 1500 small blocks in a minute, and the nurse will divide 1500 by 30. DIF: Cognitive Level: Apply (application)

When preparing to defibrillate a patient, in which order will the nurse perform the following steps? (Put a comma and a space between each answer choice [A, B, C, D, E].) a. Turn the defibrillator on. b. Deliver the electrical charge. c. Select the appropriate energy level. d. Place the hands-free, multifunction defibrillator pads on the patient's chest. e. Check the location of other staff and call out "all clear."

ANS: A, C, D, E, B This order will result in rapid defibrillation without endangering hospital staff. DIF: Cognitive Level: Analyze (analysis)

When assessing a patient with possible peripheral artery disease (PAD), the nurse obtains a brachial blood pressure (BP) of 147/82 mm Hg and an ankle pressure of 112/74 mm Hg. The nurse calculates the patient's ankle-brachial index (ABI) as ________ (round up to the nearest hundredth).

ANS: 0.76 The ABI is calculated by dividing the ankle systolic BP by the brachial systolic BP.

Following an acute myocardial infarction, a previously healthy 67-year-old develops clinical manifestations of heart failure. The nurse anticipates discharge teaching will include information about a. Angiotensin-converting enzyme (ACE) inhibitors. b. Digitalis preparations. c. B-adrenergic agonists. d. Calcium channel blockers.

ANS: A ACE inhibitor therapy is currently recommended to prevent the development of heart failure in patients who have had a myocardial infarction and as a first-line therapy for patients with chronic heart failure. Digoxin therapy for heart failure is no longer considered a first-line measure, and digoxin is added to the treatment protocol when therapy with other medications such as ACE inhibitors, diuretics, & b-adrenergic blockers is insufficient. Calcium channel blockers are not generally used in the treatment of heart failure. The b-adrenergic blockers are not used as initial therapy for new onset heart failure.

Which action should the nurse include in the plan of care when caring for a patient admitted with ADHF who is receiving nesiritide? a. Monitor BP frequently b. Encourage patient to ambulate around the room c. Titrate nesiritide slowly before stopping d. Teach the patient home use of the drug

ANS: A Nesiritide is a potent arterial and venous dilator, and the major adverse effect is hypotension. Because the patient is likely to have orthostatic hypotension, the patient should not be encouraged to ambulate. Nesiritide does not require titration and is used for ADHF but not in a home setting.

The nurse hears a murmur between the S1 and S2 heart sounds at the patient's left fifth intercostal space and midclavicular line. How will the nurse record this information? a. Systolic murmur heard at mitral area b. Systolic murmur heard at Erb's point c. Diastolic murmur heard at aortic area d. Diastolic murmur heard at the point of maximal impulse

ANS: A The S1 signifies the onset of ventricular systole. S2 signifies the onset of diastole. A murmur occurring between these two sounds is a systolic murmur. The mitral area is the intersection of the left fifth intercostal space and the midclavicular line. The other responses describe murmurs heard at different landmarks on the chest and/or during the diastolic phase of the cardiac cycle.

The nurse is reviewing the laboratory results for newly admitted patients on the cardiovascular unit. Which laboratory result is most important to communicate as soon as possible to the health care provider? a. High troponin I level b. Increased triglyceride level c. Very low homocysteine level d. Elevated high-sensitivity C-reactive protein level

ANS: A The elevation in troponin I indicates that the patient has had an acute myocardial infarction. Further assessment and interventions are indicated. The other laboratory results are indicative of increased risk for coronary artery disease but are not associated with acute cardiac problems that need immediate intervention.

The standard policy on the cardiac unit states, "Notify the health care provider for mean arterial pressure (MAP) less than 70 mm Hg." The nurse will need to call the health care provider about the a. postoperative patient with a BP of 116/42 mm Hg. b. newly admitted patient with a BP of 150/87 mm Hg. c. patient with left ventricular failure who has a BP of 110/70 mm Hg. d. patient with a myocardial infarction who has a BP of 140/86 mm Hg.

ANS: A The mean arterial pressure (MAP) is calculated using the formula MAP = (systolic BP + 2 diastolic BP)/3. The MAP for the postoperative patient in answer 3 is 67. The MAP in the other three patients is higher than 70 mm Hg.

Aftering receiving change-of-shift report on a heart failure unit, which patient should the nurse assess first? a. A patient who is cool and clammy, with new-onset confusion and restlessness b. A patient who has crackles bilaterally in the lung bases and is receiving oxygen c. A patient who had dizziness after receiving the first dose of captopril d. A patient who is receiving IV nesiritide and has a blood pressure of 100/62

ANS: A The patient who has wet-cold clinical manifestations of heart failure is perfusing inadequately and needs rapid assessment and changes in management. The other patients also should be assessed as quickly as possible but do not have indications of severe decreases in tissue perfusion.

The nurse is developing a discharge teaching plan for a patient diagnosed with thromboangiitis obliterans (Buerger's disease). Which expected outcome has the highest priority for this patient? a. Cessation of all tobacco use b. Control of serum lipid levels c. Maintenance of appropriate weight d. Demonstration of meticulous foot care

ANS: A Absolute cessation of nicotine use is needed to reduce the risk for amputation in patients with Buerger's disease. Other therapies have limited success in treatment of this disease.

29. When caring for a patient who has just arrived on the medical-surgical unit after having cardiac catheterization, which nursing intervention should the nurse delegate to a licensed practical/vocational nurse (LPN/LVN)? a. Give the scheduled aspirin and lipid-lowering medication. b. Perform the initial assessment of the catheter insertion site. c. Teach the patient about the usual postprocedure plan of care. d. Titrate the heparin infusion according to the agency protocol.

ANS: A Administration of oral medications is within the scope of practice for LPNs/LVNs. The initial assessment of the patient, patient teaching, and titration of IV anticoagulant medications should be done by the registered nurse (RN).

The nurse has just finished teaching a hypertensive patient about the newly prescribed ramipril (Altace). Which patient statement indicates that more teaching is needed? a. "A little swelling around my lips and face is okay." b. "The medication may not work as well if I take any aspirin." c. "The doctor may order a blood potassium level occasionally." d. "I will call the doctor if I notice that I have a frequent cough."

ANS: A Angioedema occurring with angiotensin-converting enzyme (ACE) inhibitor therapy is an indication that the ACE inhibitor should be discontinued. The patient should be taught that if any swelling of the face or oral mucosa occurs, the health care provider should be immediately notified because this could be life threatening. The other patient statements indicate that the patient has an accurate understanding of ACE inhibitor therapy

A patient with dilated cardiomyopathy has new onset atrial fibrillation that has been unresponsive to drug therapy for several days. Teaching for this patient would include information about a. anticoagulant therapy. c. emergency cardioversion. b. permanent pacemakers. d. IV adenosine (Adenocard).

ANS: A Atrial fibrillation therapy that has persisted for more than 48 hours requires anticoagulant treatment for 3 weeks before attempting cardioversion. This is done to prevent embolization of clots from the atria. Cardioversion may be done after several weeks of anticoagulation therapy. Adenosine is not used to treat atrial fibrillation. Pacemakers are routinely used for patients with bradydysrhythmias. Information does not indicate that the patient has a slow heart rate. DIF: Cognitive Level: Apply (application)

A patient with dilated cardiomyopathy has new-onset atrial fibrillation that has been unresponsive to drug therapy for several days. Teaching for this patient would include information about a. anticoagulant therapy. c. emergency cardioversion. b. permanent pacemakers. d. IV adenosine (Adenocard).

ANS: A Atrial fibrillation therapy that has persisted for more than 48 hours requires anticoagulant treatment for 3 weeks before attempting cardioversion. This is done to prevent embolization of clots from the atria. Cardioversion may be done after several weeks of anticoagulation therapy. Adenosine is not used to treat atrial fibrillation. Pacemakers are routinely used for patients with bradydysrhythmias. Information does not indicate that the patient has a slow heart rate. DIF: Cognitive Level: Apply (application)

After the nurse gives IV atropine to a patient with symptomatic type 1, second-degree atrioventricular (AV) block, which finding indicates that the drug has been effective? a. Increase in the patient's heart rate b. Increase in strength of peripheral pulses c. Decrease in premature atrial contractions d. Decrease in premature ventricular contractions

ANS: A Atropine will increase the heart rate and conduction through the AV node. Because the drug increases electrical conduction, not cardiac contractility, the quality of the peripheral pulses is not used to evaluate the drug effectiveness. The patient does not have premature atrial or ventricular contractions. DIF: Cognitive Level: Apply (application)

Which patient statement to the nurse is most consistent with the diagnosis of venous insufficiency? a. "I can't get my shoes on at the end of the day." b. "I can't ever seem to get my feet warm enough." c. "I have burning leg pains after I walk two blocks." d. "I wake up during the night because my legs hurt."

ANS: A Because the edema associated with venous insufficiency increases when the patient has been standing, shoes will feel tighter at the end of the day. The other patient statements are characteristic of peripheral artery disease.

The nurse is caring for a patient with critical limb ischemia who has just arrived on the nursing unit after having percutaneous transluminal balloon angioplasty. Which action should the nurse perform first? a. Obtain vital signs. c. Assess pedal pulses. b. Teach wound care. d. Check the wound site.

ANS: A Bleeding is a possible complication after catheterization of the femoral artery, so the nurse's first action should be to assess for changes in vital signs that might indicate hemorrhage. The other actions are also appropriate but can be done after determining that bleeding is not occurring.

The nurse is caring for a 78-year-old patient with aortic stenosis. Which assessment data obtained by the nurse would be most important to report to the health care provider? a. The patient complains of chest pressure when ambulating. b. A loud systolic murmur is heard along the right sternal border. c. A thrill is palpated at the second intercostal space, right sternal border. d. The point of maximum impulse (PMI) is at the left midclavicular line.

ANS: A Chest pressure (or pain) occurring with aortic stenosis is caused by cardiac ischemia, and reporting this information would be a priority. A systolic murmur and thrill are expected in a patient with aortic stenosis. A PMI at the left midclavicular line is normal.

34. Which information about a patient who has been receiving thrombolytic therapy for an acute myocardial infarction (AMI) is most important for the nurse to communicate to the health care provider? a. No change in the patient's chest pain b. An increase in troponin levels from baseline c. A large bruise at the patient's IV insertion site d. A decrease in ST-segment elevation on the electrocardiogram

ANS: A Continued chest pain suggests that the thrombolytic therapy is not effective and that other interventions such as percutaneous coronary intervention (PCI) may be needed. Bruising is a possible side effect of thrombolytic therapy, but it is not an indication that therapy should be discontinued. The decrease of the ST-segment elevation indicates that thrombolysis is occurring and perfusion is returning to the injured myocardium. An increase in troponin levels is expected with reperfusion and is related to the washout of cardiac markers into the circulation as the blocked vessel is opened.

The nurse is caring for a 64-year-old patient admitted with mitral valve regurgitation. Which information obtained by the nurse when assessing the patient should be communicated to the health care provider immediately? a. The patient has bilateral crackles. b. The patient has bilateral, 4+ peripheral edema. c. The patient has a loud systolic murmur across the precordium. d. The patient has a palpable thrill felt over the left anterior chest.

ANS: A Crackles that are audible throughout the lungs indicate that the patient is experiencing severe left ventricular failure with pulmonary congestion and needs immediate interventions such as diuretics. A systolic murmur and palpable thrill would be expected in a patient with mitral regurgitation. Although 4+ peripheral edema indicates a need for a change in therapy, it does not need to be addressed urgently.

25. The nurse is caring for a patient who was admitted to the coronary care unit following an acute myocardial infarction (AMI) and percutaneous coronary intervention the previous day. Teaching for this patient would include a. when cardiac rehabilitation will begin. b. the typical emotional responses to AMI. c. information regarding discharge medications. d. the pathophysiology of coronary artery disease.

ANS: A Early after an AMI, the patient will want to know when resumption of usual activities can be expected. At this time, the patient's anxiety level or denial will interfere with good understanding of complex information such as the pathophysiology of coronary artery disease (CAD). Teaching about discharge medications should be done closer to discharge. The nurse should support the patient by decreasing anxiety rather than discussing the typical emotional responses to myocardial infarction (MI).

A patient is admitted to the hospital with possible acute pericarditis. The nurse should plan to teach the patient about the purpose of a. echocardiography. b. daily blood cultures. c. cardiac catheterization. d. 24-hour Holter monitor.

ANS: A Echocardiograms are useful in detecting the presence of the pericardial effusions associated with pericarditis. Blood cultures are not indicated unless the patient has evidence of sepsis. Cardiac catheterization and 24-hour Holter monitor is not a diagnostic procedure for pericarditis.

A 20-yr-old patient has a mandatory electrocardiogram (ECG) before participating on a college soccer team and is found to have sinus bradycardia, rate 52. Blood pressure (BP) is 114/54 mm Hg, and the student denies any health problems. What action by the nurse is most appropriate? a. Allow the student to participate on the soccer team. b. Refer the student to a cardiologist for further testing. c. Tell the student to stop playing immediately if any dyspnea occurs. d. Obtain more detailed information about the student's family health history.

ANS: A In an aerobically trained individual, sinus bradycardia is normal. The student's normal BP and negative health history indicate that there is no need for a cardiology referral or for more detailed information about the family's health history. Dyspnea during an aerobic activity such as soccer is normal. DIF: Cognitive Level: Apply (application)

26. A patient who has recently started taking pravastatin (Pravachol) and niacin (Nicobid) reports the following symptoms to the nurse. Which is most important to communicate to the health care provider? a. Generalized muscle aches and pains b. Dizziness when changing positions quickly c. Nausea when taking the drugs before eating d. Flushing and pruritus after taking the medications

ANS: A Muscle aches and pains may indicate myopathy and rhabdomyolysis, which have caused acute kidney injury and death in some patients who have taken the statin medications. These symptoms indicate that the pravastatin may need to be discontinued. The other symptoms are common side effects when taking niacin, and although the nurse should follow-up with the health care provider, they do not indicate that a change in medication is needed.

Propranolol (Inderal) is prescribed for a patient diagnosed with hypertension. The nurse should consult with the health care provider before giving this medication when the patient reveals a history of a. asthma. b. daily alcohol use. c. peptic ulcer disease. d. myocardial infarction (MI).

ANS: A Nonselective b-blockers block b1- and b2-adrenergic receptors and can cause bronchospasm, especially in patients with a history of asthma. b-Blockers will have no effect on the patient's peptic ulcer disease or alcohol use. b-Blocker therapy is recommended after MI

After teaching a patient with newly diagnosed Raynaud's phenomenon about how to manage the condition, which action by the patient best demonstrates that the teaching has been effective? a. The patient exercises indoors during the winter months. b. The patient immerses hands in hot water when they turn pale. c. The patient takes pseudoephedrine (Sudafed) for cold symptoms. d. The patient avoids taking nonsteroidal antiinflammatory drugs (NSAIDs).

ANS: A Patients should avoid temperature extremes by exercising indoors when it is cold. To avoid burn injuries, the patient should use warm rather than hot water to warm the hands. Pseudoephedrine is a vasoconstrictor and should be avoided. There is no reason to avoid taking NSAIDs with Raynaud's phenomenon

The nurse suspects cardiac tamponade in a patient who has acute pericarditis. To assess for the presence of pulsus paradoxus, the nurse should a. note when Korotkoff sounds are auscultated during both inspiration and expiration. b. subtract the diastolic blood pressure (DBP) from the systolic blood pressure (SBP). c. check the electrocardiogram (ECG) for variations in rate during the respiratory cycle. d. listen for a pericardial friction rub that persists when the patient is instructed to stop breathing.

ANS: A Pulsus paradoxus exists when there is a gap of greater than 10 mm Hg between when Korotkoff sounds can be heard during only expiration and when they can be heard throughout the respiratory cycle. The other methods described would not be useful in determining the presence of pulsus paradoxus.

A patient in the outpatient clinic has a new diagnosis of peripheral artery disease (PAD). Which group of drugs will the nurse plan to include when teaching about PAD management? a. Statins b. Antibiotics c. Thrombolytics d. Anticoagulants

ANS: A Research indicates that statin use by patients with PAD improves multiple outcomes. There is no research that supports the use of the other drug categories in PAD.

While caring for a patient with aortic stenosis, the nurse identifies a nursing diagnosis of acute pain related to decreased coronary blood flow. A priority nursing intervention for this patient would be to a. promote rest to decrease myocardial oxygen demand. b. teach the patient about the need for anticoagulant therapy. c. teach the patient to use sublingual nitroglycerin for chest pain. d. raise the head of the bed 60 degrees to decrease venous return.

ANS: A Rest is recommended to balance myocardial oxygen supply and demand and to decrease chest pain. The patient with aortic stenosis requires higher preload to maintain cardiac output, so nitroglycerin and measures to decrease venous return are contraindicated. Anticoagulation is not recommended unless the patient has atrial fibrillation.

The charge nurse observes a new registered nurse (RN) doing discharge teaching for a patient with hypertension who has a new prescription for enalapril (Vasotec). The charge nurse will need to intervene if the new RN tells the patient to a. increase the dietary intake of high-potassium foods. b. make an appointment with the dietitian for teaching. c. check the blood pressure (BP) with a home BP monitor at least once a day. d. move slowly when moving from lying to sitting to standing.

ANS: A The ACE inhibitors cause retention of potassium by the kidney, so hyperkalemia is a possible adverse effect. The other teaching by the new RN is appropriate for a patient with newly diagnosed hypertension who has just started therapy with enalapril

The charge nurse observes a new registered nurse (RN) doing discharge teaching for a patient with hypertension who has a new prescription for enalapril (Vasotec). The charge nurse will need to intervene if the new RN tells the patient to a. increase the dietary intake of high-potassium foods. b. make an appointment with the dietitian for teaching. c. check the blood pressure (BP) at home at least once a day. d. move slowly when moving from lying to sitting to standing.

ANS: A The ACE inhibitors cause retention of potassium by the kidney, so hyperkalemia is a possible adverse effect. The other teaching by the new RN is appropriate for a patient with newly diagnosed hypertension who has just started therapy with enalapril

The charge nurse observes a new registered nurse (RN) doing discharge teaching for a patient with hypertension who has a new prescription for enalapril (Vasotec). The charge nurse will need to intervene if the new RN tells the patient to a. increase the dietary intake of high-potassium foods. b. make an appointment with the dietitian for teaching. c. check the blood pressure (BP) at home at least once a day. d. move slowly when moving from lying to sitting to standing.

ANS: A The ACE inhibitors cause retention of potassium by the kidney, so hyperkalemia is a possible adverse effect. The other teaching by the new RN is appropriate for a patient with newly diagnosed hypertension who has just started therapy with enalapril. DIF: Cognitive Level: Apply (application)

The nurse is reviewing the laboratory test results for a patient who has recently been diagnosed with hypertension. Which result is most important to communicate to the health care provider? a. Serum creatinine of 2.8 mg/dL b. Serum potassium of 4.5 mEq/L c. Serum hemoglobin of 14.7 g/dL d. Blood glucose level of 96 mg/dL

ANS: A The elevated creatinine indicates renal damage caused by the hypertension. The other laboratory results are normal

The nurse is reviewing the laboratory test results for a patient who has recently been diagnosed with hypertension. Which result is most important to communicate to the health care provider? a. Serum creatinine of 2.8 mg/dL c. Serum hemoglobin of 14.7 g/dL b. Serum potassium of 4.5 mEq/L d. Blood glucose level of 96 mg/dL

ANS: A The elevated serum creatinine indicates renal damage caused by the hypertension. The other laboratory results are normal.

The nurse is reviewing the laboratory test results for a patient who has recently been diagnosed with hypertension. Which result is most important to communicate to the health care provider? a. Serum creatinine of 2.8 mg/dL b. Serum potassium of 4.5 mEq/L c. Serum hemoglobin of 14.7 g/dL d. Blood glucose level of 96 mg/dL

ANS: A The elevated serum creatinine indicates renal damage caused by the hypertension. The other laboratory results are normal. DIF: Cognitive Level: Analyze (analysis)

Which nursing action should the nurse take first in order to assist a patient with newly diagnosed stage 1 hypertension in making needed dietary changes? a. Collect a detailed diet history. b. Provide a list of low-sodium foods. c. Help the patient make an appointment with a dietitian. d. Teach the patient about foods that are high in potassium.

ANS: A The initial nursing action should be assessment of the patient's baseline dietary intake through a thorough diet history. The other actions may be appropriate, but assessment of the patient's baseline should occur first

Which nursing action should the nurse take first to assist a patient with newly diagnosed stage 1 hypertension in making needed dietary changes? a. Collect a detailed diet history. b. Provide a list of low-sodium foods. c. Help the patient make an appointment with a dietitian. d. Teach the patient about foods that are high in potassium.

ANS: A The initial nursing action should be assessment of the patient's baseline dietary intake through a thorough diet history. The other actions may be appropriate, but assessment of the patient's baseline should occur first.

Which nursing action should the nurse take first to assist a patient with newly diagnosed stage 1 hypertension in making needed dietary changes? a. Collect a detailed diet history. b. Provide a list of low-sodium foods. c. Help the patient make an appointment with a dietitian. d. Teach the patient about foods that are high in potassium.

ANS: A The initial nursing action should be assessment of the patient's baseline dietary intake through a thorough diet history. The other actions may be appropriate, but assessment of the patient's baseline should occur first. DIF: Cognitive Level: Analyze (analysis)

27. A patient who is being admitted to the emergency department with intermittent chest pain gives the following list of medications to the nurse. Which medication has the most immediate implications for the patient's care? a. Sildenafil (Viagra) b. Furosemide (Lasix) c. Captopril (Capoten) d. Warfarin (Coumadin)

ANS: A The nurse will need to avoid giving nitrates to the patient because nitrate administration is contraindicated in patients who are using sildenafil because of the risk of severe hypotension caused by vasodilation. The other home medications also should be documented and reported to the health care provider but do not have as immediate an impact on decisions about the patient's treatment.

19. Three days after experiencing a myocardial infarction (MI), a patient who is scheduled for discharge asks for assistance with hygiene activities, saying, "I am too nervous to take care of myself." Based on this information, which nursing diagnosis is appropriate? a. Ineffective coping related to anxiety b. Activity intolerance related to weakness c. Denial related to lack of acceptance of the MI d. Disturbed personal identity related to understanding of illness

ANS: A The patient data indicate that ineffective coping after the MI caused by anxiety about the impact of the MI is a concern. The other nursing diagnoses may be appropriate for some patients after an MI, but the data for this patient do not support denial, activity intolerance, or disturbed personal identity.

A patient who is 2 days post femoral popliteal bypass graft to the right leg is being cared for on the vascular unit. Which action by a licensed practical/vocational nurse (LPN/LVN) caring for the patient requires the registered nurse (RN) to intervene? a. The LPN/LVN has the patient to sit in a chair for 2 hours. b. The LPN/LVN gives the prescribed aspirin after breakfast. c. The LPN/LVN assists the patient to walk 40 feet in the hallway. d. The LPN/LVN places the patient in Fowler's position for meals.

ANS: A The patient should avoid sitting for long periods because of the increased stress on the suture line caused by leg edema and because of the risk for venous thromboembolism (VTE). The other actions by the LPN/LVN are appropriate.

The nurse on the intermediate care unit received change-of-shift report on four patients with hypertension. Which patient should the nurse assess first? a. 48-yr-old with a blood pressure of 160/92 mm Hg who reports chest pain b. 52-yr-old with a blood pressure of 198/90 mm Hg who has intermittent claudication c. 50-yr-old with a blood pressure of 190/104 mm Hg who has a creatinine of 1.7 mg/dL d. 43-yr-old with a blood pressure of 172/98 mm Hg whose urine shows microalbuminuria

ANS: A The patient with chest pain may be experiencing acute myocardial infarction and rapid assessment and intervention are needed. The symptoms of the other patients also show target organ damage but are not indicative of acute processes.

The nurse on the intermediate care unit received change-of-shift report on four patients with hypertension. Which patient should the nurse assess first? a. 48-yr-old with a blood pressure of 160/92 mm Hg who reports chest pain b. 52-yr-old with a blood pressure of 198/90 mm Hg who has intermittent claudication c. 50-yr-old with a blood pressure of 190/104 mm Hg who has a creatinine of 1.7 mg/dL d. 43-yr-old with a blood pressure of 172/98 mm Hg whose urine shows microalbuminuria

ANS: A The patient with chest pain may be experiencing acute myocardial infarction and rapid assessment and intervention are needed. The symptoms of the other patients also show target organ damage but are not indicative of acute processes. DIF: Cognitive Level: Analyze (analysis)

The nurse on the intermediate care unit received change-of-shift report on four patients with hypertension. Which patient should the nurse assess first? a. 43-year-old with a (blood pressure (BP) of 160/92 who is complaining of chest pain b. 52-year-old with a BP of 212/90 who has intermittent claudication c. 50-year-old with a BP of 190/104 who has a creatinine of 1.7 mg/dL d. 48-year-old with a BP of 172/98 whose urine shows microalbuminuria

ANS: A The patient with chest pain may be experiencing acute myocardial infarction, and rapid assessment and intervention are needed. The symptoms of the other patients also show target organ damage but are not indicative of acute processes

Two days after an acute myocardial infarction (MI), a patient complains of stabbing chest pain that increases with a deep breath. Which action will the nurse take first? a. Auscultate the heart sounds. b. Check the patient's temperature. c. Notify the patient's health care provider. d. Give the PRN acetaminophen (Tylenol).

ANS: A The patient's clinical manifestations and history are consistent with pericarditis, and the first action by the nurse should be to listen for a pericardial friction rub. Checking the temperature and notifying the health care provider are also appropriate actions but would not be done before listening for a rub. It is not stated for what symptom (e.g., headache) or finding (e.g., increased temperature) the PRN acetaminophen (Tylenol) is ordered.

21. A few days after experiencing a myocardial infarction (MI) and successful percutaneous coronary intervention, the patient states, "I just had a little chest pain. As soon as I get out of here, I'm going for my vacation as planned." Which reply would be most appropriate for the nurse to make? a. "What do you think caused your chest pain?" b. "Where are you planning to go for your vacation?" c. "Sometimes plans need to change after a heart attack." d. "Recovery from a heart attack takes at least a few weeks."

ANS: A When the patient is experiencing denial, the nurse should assist the patient in testing reality until the patient has progressed beyond this step of the emotional adjustment to MI. Asking the patient about vacation plans reinforces the patient's plan, which is not appropriate in the immediate post-MI period. Reminding the patient in denial about the MI is likely to make the patient angry and lead to distrust of the nursing staff.

A patient in the intensive care unit with ADHF complains of severe dyspnea and is anxious, tachypneic, and tachycardic. All of the following medications have been ordered for the patient. The nurses priority action will be to a. Give IV morphine sulfate 4 mg b. Give IV diazepam 2.5 mg c. Increase nitroglycerin infusion by 5 mcg/min d. Increase dopamine infusion by 2 mcg/kg/min

ANS: A Morphine improves alveolar gas exchange, improves cardiac output by reducing ventricular preload and afterload, decreases anxiety, and assists in reducing the subjective feeling of dyspnea. Diazepam may decrease patient anxiety, but it will not improve the cardiac output or gas exchange. Increasing the dopamine may improve cardiac output, but it will also increase the HR and myocardial oxygen consumption. Nitroglycerin will improve cardiac output and may appropriate for this patient, but will not directly reduce anxiety and will not act as quickly as morphine to decrease dyspnea.

During a visit to a 78-year-old with chronic heart failure, the home care nurse finds that the patient has ankle edema, a 2-kg weight gain over the past 2 days, and complains of "feeling too tired to get out of bed." Based on these data, the best nursing diagnosis for the patient is a. Activity intolerance related to fatigue. b. Disturbed body image related to weight gain. c. Impaired skin integrity related to ankle edema. d. Impaired gas exchange related to dyspnea on exertion.

ANS: A The patient's statement supports the diagnosis of activity intolerance. There are no data to support the other diagnoses, although the nurse will need to assess for other patient problems.

When auscultating over the patient's abdominal aorta, the nurse hears a loud humming sound. The nurse documents this finding as a a. thrill. c. murmur. b. bruit. d. normal finding.

ANS: B A bruit is the sound created by turbulent blood flow in an artery. Thrills are palpable vibrations felt when there is turbulent blood flow through the heart or in a blood vessel. A murmur is the sound caused by turbulent blood flow through the heart. Auscultating a bruit in an artery is not normal and indicates pathology

The nurse has received the laboratory results for a patient who developed chest pain 4 hours ago and may be having a myocardial infarction. The laboratory test result most helpful in indicating myocardial damage will be a. myoglobin. c. homocysteine (Hcy) b. troponins T and I. d. creatine kinase-MB (CK-MB).

ANS: B Cardiac troponins start to elevate 4 to 6 hours after myocardial injury and are highly specific to myocardium. They are the preferred diagnostic marker for myocardial infarction. Myoglobin rises in response to myocardial injury within 30 to 60 minutes. It is rapidly cleared from the body, thus limiting its use in the diagnosis of myocardial infarction. Low-density lipoprotein cholesterol is useful in assessing cardiovascular risk but is not helpful in determining whether a patient is having an acute myocardial infarction. Creatine kinase (CK-MB) is specific to myocardial injury and infarction and increases 4 to 6 hours after the infarction occurs. It is often trended with troponin levels. Homocysteine (Hcy) is an amino acid that is produced during protein catabolism. Elevated Hcy levels can be either hereditary or acquired from dietary deficiencies of vitamin B6 , cobalamin (vitamin B12), or folate. Elevated levels of Hcy have been linked to a higher risk of CVD, peripheral vascular disease, and stroke.

To auscultate for S3 or S4 gallops in the mitral area, the nurse listens with the a. diaphragm of the stethoscope with the patient lying flat. b. bell of the stethoscope with the patient in the left lateral position. c. diaphragm of the stethoscope with the patient in a supine position. d. bell of the stethoscope with the patient sitting and leaning forward.

ANS: B Gallop rhythms generate low-pitched sounds and are most easily heard with the bell of the stethoscope. Sounds associated with the mitral valve are accentuated by turning the patient to the left side, which brings the heart closer to the chest wall. The diaphragm of the stethoscope is best to use for the higher pitched sounds such as S1 and S2.

When assessing a newly admitted patient, the nurse notes a murmur along the left sternal border. To acquire more information about the murmur, which action will the nurse take? a. Palpate the peripheral pulses. b. Determine the timing of the sound. c. Find the point of maximal impulse. d. Compare apical and radial pulse rates.

ANS: B Murmurs are caused by turbulent blood flow, such as occurs when blood flows through a damaged valve. Relevant information includes the position in which the murmur is heard best (e.g., sitting and leaning forward), the timing of the murmur in relation to the cardiac cycle (e.g., systole, diastole), and where on the thorax the murmur is heard best. The other information is important in the cardiac assessment but will not provide information that is relevant to the murmur.

A registered nurse (RN) is observing a student nurse who is doing a physical assessment on a patient. The RN will need to intervene immediately if the student nurse a. presses on the skin over the tibia for 10 seconds to check for edema. b. palpates both carotid arteries simultaneously to compare pulse quality. c. documents a murmur heard along the right sternal border as a pulmonic murmur. d. places the patient in the left lateral position to check for the point of maximal impulse

ANS: B The carotid pulses should never be palpated at the same time to avoid vagal stimulation, dysrhythmias, and decreased cerebral blood flow. The other assessment techniques also need to be corrected. However, they are not dangerous to the patient.

When admitting a patient for a cardiac catheterization and coronary angiogram, which information about the patient is most important for the nurse to communicate to the health care provider? a. The patient's pedal pulses are +1. b. The patient is allergic to shellfish. c. The patient had a heart attack 1 year ago. d. The patient has not eaten anything today.

ANS: B The contrast dye used for the procedure is iodine based, so patients who have shellfish allergies will require treatment with medications such as corticosteroids and antihistamines before the angiogram. The other information is also communicated to the health care provider but will not require a change in the usual precardiac catheterization orders or medications.

The nurse and unlicensed assistive personnel (UAP) on the telemetry unit are caring for four patients. Which nursing action can be delegated to the UAP? a. Teaching a patient about exercise electrocardiography b. Attaching ECG monitoring electrodes after a patient bathes c. Checking the catheter insertion site for a patient who is recovering from a coronary angiogram d. Monitoring a patient who has just returned to the unit after a transesophageal echocardiogram

ANS: B UAP can be educated in standardized lead placement for ECG monitoring. Assessment of patients who have had procedures where airway maintenance (transesophageal echocardiography) or bleeding (coronary angiogram) is a concern must be done by the registered nurse (RN). Patient teaching requires RN level education and scope of practice.

Which action will the nurse include in the plan of care for a patient who was admitted with syncopal episodes of unknown origin? a. Explain the association between dysrhythmias and syncope. b. Instruct the patient to call for assistance before getting out of bed. c. Teach the patient about the need to avoid caffeine and other stimulants. d. Tell the patient about the benefits of implantable cardioverter-defibrillators.

ANS: B A patient with fainting episodes is at risk for falls. The nurse will plan to minimize the risk by having assistance whenever the patient is up. The other actions may be needed if dysrhythmias are found to be the cause of the patient's syncope but are not appropriate for syncope of unknown origin. DIF: Cognitive Level: Apply (application)

Which action by a new registered nurse (RN) who is orienting to the telemetry unit indicates a good understanding of the treatment of heart dysrhythmias? a. Prepares defibrillator settings at 360 joules for a patient whose monitor shows asystole. b. Injects IV adenosine (Adenocard) over 2 seconds to a patient with supraventricular tachycardia c. Turns the synchronizer switch to the "on" position before defibrillating a patient with ventricular fibrillation d. Gives the prescribed dose of diltiazem (Cardizem) to a patient with new-onset type II second degree AV block

ANS: B Adenosine must be given over 1 to 2 seconds to be effective. The other actions indicate a need for more teaching about treatment of heart dysrhythmias. The RN should hold the diltiazem until discussing it with the health care provider. The treatment for asystole is immediate CPR. The synchronizer switch should be "off" when defibrillating. DIF: Cognitive Level: Analyze (analysis)

The nurse has just finished teaching a hypertensive patient about the newly prescribed drug, ramipril (Altace). Which patient statement indicates that more teaching is needed? a. "The medication may not work well if I take aspirin." b. "I can expect some swelling around my lips and face." c. "The doctor may order a blood potassium level occasionally." d. "I will call the doctor if I notice that I have a frequent cough."

ANS: B Angioedema occurring with angiotensin-converting enzyme (ACE) inhibitor therapy is an indication that the ACE inhibitor should be discontinued. The patient should be taught that if any swelling of the face or oral mucosa occurs, the health care provider should be immediately notified because this could be life threatening. The other patient statements indicate that the patient has an accurate understanding of ACE inhibitor therapy

The nurse has just finished teaching a hypertensive patient about the newly prescribed drug, ramipril (Altace). Which patient statement indicates that more teaching is needed? a. "The medication may not work well if I take aspirin." b. "I can expect some swelling around my lips and face." c. "The doctor may order a blood potassium level occasionally." d. "I will call the doctor if I notice that I have a frequent cough."

ANS: B Angioedema occurring with angiotensin-converting enzyme (ACE) inhibitor therapy is an indication that the ACE inhibitor should be discontinued. The patient should be taught that if any swelling of the face or oral mucosa occurs, the health care provider should be immediately notified because this could be life threatening. The other patient statements indicate that the patient has an accurate understanding of ACE inhibitor therapy. DIF: Cognitive Level: Apply (application)

33. When admitting a patient with a non-ST-segment-elevation myocardial infarction (NSTEMI) to the intensive care unit, which action should the nurse perform first? a. Obtain the blood pressure. b. Attach the cardiac monitor. c. Assess the peripheral pulses. d. Auscultate the breath sounds.

ANS: B Because dysrhythmias are the most common complication of myocardial infarction (MI), the first action should be to place the patient on a cardiac monitor. The other actions also are important and should be accomplished as quickly as possible.

A patient with hypertension who has just started taking atenolol (Tenormin) returns to the health clinic after 2 weeks for a follow-up visit. The blood pressure (BP) is unchanged from the previous visit. Which action should the nurse take first? a. Tell the patient why a change in drug dosage is needed. b. Ask the patient if the medication is being taken as prescribed. c. Inform the patient that multiple drugs are often needed to treat hypertension. d. Question the patient regarding any lifestyle changes made to help control BP.

ANS: B Because nonadherence with antihypertensive therapy is common, the nurse's initial action should be to determine whether the patient is taking the atenolol as prescribed. The other actions also may be implemented, but these would be done after assessing patient adherence with the prescribed therapy.

A patient with hypertension who has just started taking atenolol (Tenormin) returns to the health clinic after 2 weeks for a follow-up visit. The blood pressure (BP) is unchanged from the previous visit. Which action should the nurse take first? a. Inform the patient about the reasons for a possible change in drug dosage. b. Question the patient about whether the medication is actually being taken. c. Inform the patient that multiple drugs are often needed to treat hypertension. d. Question the patient regarding any lifestyle changes made to help control BP.

ANS: B Because noncompliance with antihypertensive therapy is common, the nurse's initial action should be to determine whether the patient is taking the atenolol as prescribed. The other actions also may be implemented, but these would be done after assessing patient compliance with the prescribed therapy

39. To improve the physical activity level for a mildly obese 71-year-old patient, which action should the nurse plan to take? a. Stress that weight loss is a major benefit of increased exercise. b. Determine what kind of physical activities the patient usually enjoys. c. Tell the patient that older adults should exercise for no more than 20 minutes at a time. d. Teach the patient to include a short warm-up period at the beginning of physical activity.

ANS: B Because patients are more likely to continue physical activities that they already enjoy, the nurse will plan to ask the patient about preferred activities. The goal for older adults is 30 minutes of moderate activity on most days. Older adults should plan for a longer warm-up period. Benefits of exercises, such as improved activity tolerance, should be emphasized rather than aiming for significant weight loss in older mildly obese adults.

The nurse is admitting a patient newly diagnosed with peripheral artery disease. Which admission order should the nurse question? a. Cilostazol drug therapy b. Omeprazole drug therapy c. Use of treadmill for exercise d. Exercise to the point of discomfort

ANS: B Because the antiplatelet effect of clopidogrel is reduced when it is used with omeprazole, the nurse should clarify this order with the health care provider. The other interventions are appropriate for a patient with peripheral artery disease.

3. Which assessment data collected by the nurse who is admitting a patient with chest pain suggest that the pain is caused by an acute myocardial infarction (AMI)? a. The pain increases with deep breathing. b. The pain has lasted longer than 30 minutes. c. The pain is relieved after the patient takes nitroglycerin. d. The pain is reproducible when the patient raises the arms.

ANS: B Chest pain that lasts for 20 minutes or more is characteristic of AMI. Changes in pain that occur with raising the arms or with deep breathing are more typical of musculoskeletal pain or pericarditis. Stable angina is usually relieved when the patient takes nitroglycerin.

While working in the outpatient clinic, the nurse notes that a patient has a history of intermittent claudication. Which statement by the patient would support this information? a. "When I stand too long, my feet start to swell." b. "My legs cramp when I walk more than a block." c. "I get short of breath when I climb a lot of stairs." d. "My fingers hurt when I go outside in cold weather."

ANS: B Cramping that is precipitated by a consistent level of exercise is descriptive of intermittent claudication. Finger pain associated with cold weather is typical of Raynaud's phenomenon. Shortness of breath that occurs with exercise is not typical of intermittent claudication, which is reproducible. Swelling associated with prolonged standing is typical of venous disease.

The nurse identifies the nursing diagnosis of decreased cardiac output related to valvular insufficiency for the patient with infective endocarditis (IE) based on which assessment finding(s)? a. Fever, chills, and diaphoresis b. Urine output less than 30 mL/hr c. Petechiae on the inside of the mouth and conjunctiva d. Increase in heart rate of 15 beats/minute with walking

ANS: B Decreased renal perfusion caused by inadequate cardiac output will lead to decreased urine output. Petechiae, fever, chills, and diaphoresis are symptoms of IE, but are not caused by decreased cardiac output. An increase in pulse rate of 15 beats/minute is normal with exercise.

20. When caring for a patient who is recovering from a sudden cardiac death (SCD) event and has no evidence of an acute myocardial infarction (AMI), the nurse will anticipate teaching the patient that a. sudden cardiac death events rarely reoccur. b. additional diagnostic testing will be required. c. long-term anticoagulation therapy will be needed. d. limited physical activity after discharge will be needed to prevent future events.

ANS: B Diagnostic testing (e.g., stress test, Holter monitor, electrophysiologic studies, cardiac catheterization) is used to determine the possible cause of the SCD and treatment options. SCD is likely to recur. Anticoagulation therapy will not have any effect on the incidence of SCD, and SCD can occur even when the patient is resting.

A patient has a sinus rhythm and a heart rate of 72 beats/min. The nurse determines that the PR interval is 0.24 seconds. The most appropriate intervention by the nurse would be to a. notify the a provider immediately. b. document the finding and monitor the patient. c. give atropine per agency dysrhythmia protocol. d. prepare the patient for temporary pacemaker insertion.

ANS: B First-degree atrioventricular block is asymptomatic and requires ongoing monitoring because it may progress to more serious forms of heart block. The rate is normal, so there is no indication that atropine is needed. Immediate notification of the health care provider about an asymptomatic rhythm is not necessary. DIF: Cognitive Level: Apply (application)

A patient has a sinus rhythm and a heart rate of 72 beats/min. The nurse determines that the PR interval is 0.24 seconds. The most appropriate intervention by the nurse would be to a. notify the health care provider immediately. b. document the finding and monitor the patient. c. give atropine per agency dysrhythmia protocol. d. prepare the patient for temporary pacemaker insertion.

ANS: B First-degree atrioventricular block is asymptomatic and requires ongoing monitoring because it may progress to more serious forms of heart block. The rate is normal, so there is no indication that atropine is needed. Immediate notification of the health care provider about an asymptomatic rhythm is not necessary. DIF: Cognitive Level: Apply (application)

Which assessment finding for a patient who is receiving IV furosemide (Lasix) to treat stage 2 hypertension is most important to report to the health care provider? a. Blood glucose level of 175 mg/dL b. Blood potassium level of 3.0 mEq/L c. Most recent blood pressure (BP) reading of 168/94 mm Hg d. Orthostatic systolic BP decrease of 12 mm Hg

ANS: B Hypokalemia is a frequent adverse effect of the loop diuretics and can cause life-threatening dysrhythmias. The health care provider should be notified of the potassium level immediately and administration of potassium supplements initiated. The elevated blood glucose and BP also indicate a need for collaborative interventions but will not require action as urgently as the hypokalemia. An orthostatic drop of 12 mm Hg is common and will require intervention only if the patient is symptomatic

Which assessment finding for a patient who is receiving IV furosemide (Lasix) to treat stage 2 hypertension is most important to report to the health care provider? a. Blood glucose level of 175 mg/dL b. Serum potassium level of 3.0 mEq/L c. Orthostatic systolic BP decrease of 12 mm Hg d. Most recent blood pressure (BP) reading of 168/94 mm Hg

ANS: B Hypokalemia is a frequent adverse effect of the loop diuretics and can cause life-threatening dysrhythmias. The health care provider should be notified of the potassium level immediately and administration of potassium supplements initiated. The elevated blood glucose and BP also indicate a need for collaborative interventions but will not require action as urgently as the hypokalemia. An orthostatic drop of 12 mm Hg will require intervention only if the patient is symptomatic.

Which assessment finding for a patient who is receiving IV furosemide (Lasix) to treat stage 2 hypertension is most important to report to the health care provider? a. Blood glucose level of 175 mg/dL b. Serum potassium level of 3.0 mEq/L c. Orthostatic systolic BP decrease of 12 mm Hg d. Most recent blood pressure (BP) reading of 168/94 mm Hg

ANS: B Hypokalemia is a frequent adverse effect of the loop diuretics and can cause life-threatening dysrhythmias. The health care provider should be notified of the potassium level immediately and administration of potassium supplements initiated. The elevated blood glucose and BP also indicate a need for collaborative interventions but will not require action as urgently as the hypokalemia. An orthostatic drop of 12 mm Hg will require intervention only if the patient is symptomatic. DIF: Cognitive Level: Analyze (analysis)

After receiving report on the following patients, which patient should the nurse assess first? a. Patient with rheumatic fever who has sharp chest pain with a deep breath b. Patient with acute aortic regurgitation whose blood pressure is 86/54 mm Hg c. Patient with infective endocarditis who has a murmur and splinter hemorrhages d. Patient with dilated cardiomyopathy who has bilateral crackles at the lung bases

ANS: B Hypotension in patients with acute aortic regurgitation may indicate cardiogenic shock. The nurse should immediately assess this patient for other findings such as dyspnea or chest pain. The findings in the other patients are typical of their diagnoses and do not indicate a need for urgent assessment and intervention.

An older patient has been diagnosed with possible white coat hypertension. Which planned action by the nurse best addresses the suspected cause of the hypertension? a. Instruct the patient about the need to decrease stress levels. b. Teach the patient how to self-monitor and record BPs at home. c. Schedule the patient for regular blood pressure (BP) checks in the clinic. d. Inform the patient and caregiver that major dietary changes will be needed.

ANS: B In the phenomenon of "white coat" hypertension, patients have elevated BP readings in a clinical setting and normal readings when BP is measured elsewhere. Having the patient self-monitor BPs at home will provide a reliable indication about whether the patient has hypertension. Regular BP checks in the clinic are likely to be high in a patient with white coat hypertension. There is no evidence that this patient has elevated stress levels or a poor diet, and those factors do not cause white coat hypertension.

An older patient has been diagnosed with possible white coat hypertension. Which planned action by the nurse best addresses the suspected cause of the hypertension? a. Instruct the patient about the need to decrease stress levels. b. Teach the patient how to self-monitor and record BPs at home. c. Schedule the patient for regular blood pressure (BP) checks in the clinic. d. Inform the patient and caregiver that major dietary changes will be needed.

ANS: B In the phenomenon of "white coat" hypertension, patients have elevated BP readings in a clinical setting and normal readings when BP is measured elsewhere. Having the patient self-monitor BPs at home will provide a reliable indication about whether the patient has hypertension. Regular BP checks in the clinic are likely to be high in a patient with white coat hypertension. There is no evidence that this patient has elevated stress levels or a poor diet, and those factors do not cause white coat hypertension. DIF: Cognitive Level: Apply (application)

The health care provider prescribes an infusion of heparin and daily partial thromboplastin time (PTT) testing for a patient with venous thromboembolism (VTE). The nurse will plan to a. decrease the infusion when the PTT value is 65 seconds. b. avoid giving IM medications to prevent localized bleeding. c. have vitamin K available in case reversal of the heparin is needed. d. monitor posterior tibial and dorsalis pedis pulses with the Doppler

ANS: B Intramuscular injections are avoided in patients receiving anticoagulation to prevent hematoma formation and bleeding from the site. A PTT of 65 seconds is within the therapeutic range. Vitamin K is used to reverse warfarin. Pulse quality is not affected by VTE.

The nurse who works in the vascular clinic has several patients with venous insufficiency scheduled today. Which patient should the nurse assign to an experienced licensed practical/vocational nurse (LPN/LVN)? a. Patient who has been complaining of increased edema and skin changes in the legs b. Patient who needs wound care for a chronic venous stasis ulcer on the right lower leg c. Patient who has a history of venous thromboembolism and is complaining of dyspnea d. Patient who needs teaching about elastic compression stockings for venous insufficiency

ANS: B LPN education and scope of practice includes wound care. The other patients, which require more complex assessments or education, should be managed by the RN

A patient has ST segment changes that suggest an acute inferior wall myocardial infarction. Which lead would be best for monitoring the patient? a. I c. V2 b. II d. V6

ANS: B Leads II, III, and AVF reflect the inferior area of the heart and the ST segment changes. Lead II will best capture any electrocardiographic changes that indicate further damage to the myocardium. The other leads do not reflect the inferior part of the myocardial wall and will not provide data about further ischemic changes in that area. DIF: Cognitive Level: Analyze (analysis)

A patient has ST-segment changes that suggest an acute inferior wall myocardial infarction. Which lead would be best for monitoring the patient? a. I c. V2 b. II d. V6

ANS: B Leads II, III, and AVF reflect the inferior area of the heart and the ST segment changes. Lead II will best capture any electrocardiographic changes that indicate further damage to the myocardium. The other leads do not reflect the inferior part of the myocardial wall and will not provide data about further ischemic changes in that area. DIF: Cognitive Level: Analyze (analysis)

A patient with a venous thromboembolism (VTE) is started on enoxaparin (Lovenox) and warfarin (Coumadin). The patient asks the nurse why two medications are necessary. Which response by the nurse is most accurate? a. "Taking two blood thinners greatly reduces the risk for another clot to form." b. "Enoxaparin will work right away, but warfarin takes several days to begin preventing clots." c. "Enoxaparin will start to dissolve the clot, and warfarin will prevent any more clots from forming." d. "Because of the risk for a blood clot in the lungs, it is important for you to take more than one blood thinner."

ANS: B Low molecular weight heparin (LMWH) is used because of the immediate effect on coagulation and discontinued once the international normalized ratio (INR) value indicates that the warfarin has reached a therapeutic level. LMWH has no thrombolytic properties. The use of two anticoagulants is not related to the risk for pulmonary embolism, and two are not necessary to reduce the risk for another VTE. Anticoagulants do not thin the blood.

The nurse is assessing a patient with myocarditis before administering the scheduled dose of digoxin (Lanoxin). Which finding is most important for the nurse to communicate to the health care provider? a. Leukocytosis b. Irregular pulse c. Generalized myalgia d. Complaint of fatigue

ANS: B Myocarditis predisposes the heart to digoxin-associated dysrhythmias and toxicity. The other findings are common symptoms of myocarditis and there is no urgent need to report these.

During the assessment of a 25-year-old patient with infective endocarditis (IE), the nurse would expect to find a. substernal chest pressure. b. a new regurgitant murmur. c. a pruritic rash on the chest. d. involuntary muscle movement.

ANS: B New regurgitant murmurs occur in IE because vegetations on the valves prevent valve closure. Substernal chest discomfort, rashes, and involuntary muscle movement are clinical manifestations of other cardiac disorders such as angina and rheumatic fever.

The nurse has started discharge teaching for a patient who is to continue warfarin (Coumadin) after hospitalization for venous thromboembolism (VTE). The nurse determines that additional teaching is needed when the patient says which of the following? a. "I should get a Medic Alert device stating that I take warfarin." b. "I should reduce the amount of green, leafy vegetables that I eat." c. "I will need routine blood tests to monitor the effects of the warfarin." d. "I will check with my health care provider before I begin any new drugs."

ANS: B Patients taking warfarin are taught to follow a consistent diet with regard to foods that are high in vitamin K, such as green, leafy vegetables. The other patient statements are accurate.

10. The nurse will suspect that the patient with stable angina is experiencing a side effect of the prescribed metoprolol (Lopressor) if the a. patient is restless and agitated. b. blood pressure is 90/54 mm Hg. c. patient complains about feeling anxious. d. cardiac monitor shows a heart rate of 61 beats/minute.

ANS: B Patients taking β-adrenergic blockers should be monitored for hypotension and bradycardia. Because this class of medication inhibits the sympathetic nervous system, restlessness, agitation, hypertension, and anxiety will not be side effects.

7. After the nurse teaches the patient about the use of carvedilol (Coreg) in preventing anginal episodes, which statement by a patient indicates that the teaching has been effective? a. "Carvedilol will help my heart muscle work harder." b. "It is important not to suddenly stop taking the carvedilol." c. "I can expect to feel short of breath when taking carvedilol." d. "Carvedilol will increase the blood flow to my heart muscle."

ANS: B Patients who have been taking β-adrenergic blockers can develop intense and frequent angina if the medication is suddenly discontinued. Carvedilol (Coreg) decreases myocardial contractility. Shortness of breath that occurs when taking β-adrenergic blockers for angina may be due to bronchospasm and should be reported to the health care provider. Carvedilol works by decreasing myocardial oxygen demand, not by increasing blood flow to the coronary arteries.

To assess the patient with pericarditis for evidence of a pericardial friction rub, the nurse should a. listen for a rumbling, low-pitched, systolic murmur over the left anterior chest. b. auscultate by placing the diaphragm of the stethoscope on the lower left sternal border. c. ask the patient to cough during auscultation to distinguish the sound from a pleural friction rub. d. feel the precordial area with the palm of the hand to detect vibrations with cardiac contraction.

ANS: B Pericardial friction rubs are heard best with the diaphragm at the lower left sternal border. The nurse should ask the patient to hold his or her breath during auscultation to distinguish the sounds from a pleural friction rub. Friction rubs are not typically low pitched or rumbling and are not confined to systole. Rubs are not assessed by palpation.

9. Diltiazem (Cardizem) is ordered for a patient with newly diagnosed Prinzmetal's (variant) angina. When teaching the patient, the nurse will include the information that diltiazem will a. reduce heart palpitations. b. decrease spasm of the coronary arteries. c. increase the force of the heart contractions. d. help prevent plaque from forming in the coronary arteries.

ANS: B Prinzmetal's angina is caused by coronary artery spasm. Calcium channel blockers (e.g., diltiazem, amlodipine [Norvasc

The nurse is admitting a patient with possible rheumatic fever. Which question on the admission health history will be most pertinent to ask? a. "Do you use any illegal IV drugs?" b. "Have you had a recent sore throat?" c. "Have you injured your chest in the last few weeks?" d. "Do you have a family history of congenital heart disease?"

ANS: B Rheumatic fever occurs as a result of an abnormal immune response to a streptococcal infection. Although illicit IV drug use should be discussed with the patient before discharge, it is not a risk factor for rheumatic fever, and would not be as pertinent when admitting the patient. Family history is not a risk factor for rheumatic fever. Chest injury would cause musculoskeletal chest pain rather than rheumatic fever.

23. A patient who is recovering from an acute myocardial infarction (AMI) asks the nurse about when sexual intercourse can be resumed. Which response by the nurse is best? a. "Most patients are able to enjoy intercourse without any complications." b. "Sexual activity uses about as much energy as climbing two flights of stairs." c. "The doctor will provide sexual guidelines when your heart is strong enough." d. "Holding and cuddling are good ways to maintain intimacy after a heart attack."

ANS: B Sexual activity places about as much physical stress on the cardiovascular system as most moderate-energy activities such as climbing two flights of stairs. The other responses do not directly address the patient's question or may not be accurate for this patient.

Which instructions should the nurse include in a teaching plan for an older patient newly diagnosed with peripheral artery disease (PAD)? a. "Exercise only if you do not experience any pain." b. "It is very important that you stop smoking cigarettes." c. "Try to keep your legs elevated whenever you are sitting." d. "Put elastic compression stockings on early in the morning."

ANS: B Smoking cessation is essential for slowing the progression of PAD to critical limb ischemia and reducing the risk of myocardial infarction and death. Circulation to the legs will decrease if the legs are elevated. Patients with PAD are taught to exercise to the point of feeling pain, rest, and then resume walking. Support hose are not used for patients with PAD.

Which assessment finding in a patient who is admitted with infective endocarditis (IE) is most important to communicate to the health care provider? a. Generalized muscle aching b. Sudden onset right flank pain c. Janeway's lesions on the palms d. Temperature 100.7° F (38.1° C)

ANS: B Sudden onset of flank pain indicates possible embolization to the kidney and may require diagnostic testing such as a renal arteriogram and interventions to improve renal perfusion. The other findings are typically found in IE, but do not require any new interventions.

A patient with a history of hypertension treated with a diuretic and an angiotensin-converting enzyme (ACE) inhibitor arrives in the emergency department complaining of a severe headache and nausea and has a blood pressure (BP) of 238/118 mm Hg. Which question should the nurse ask first? a. "Did you take any acetaminophen (Tylenol) today?" b. "Have you been consistently taking your medications?" c. "Have there been any recent stressful events in your life?" d. "Have you recently taken any antihistamine medications?"

ANS: B Sudden withdrawal of antihypertensive medications can cause rebound hypertension and hypertensive crisis. Although many over-the-counter medications can cause hypertension, antihistamines and acetaminophen do not increase BP. Stressful events will increase BP but not usually to the level seen in this patient

A patient with a history of hypertension treated with a diuretic and an angiotensin-converting enzyme (ACE) inhibitor arrives in the emergency department complaining of a severe headache and nausea and has a blood pressure (BP) of 238/118 mm Hg. Which question should the nurse ask to follow up on these findings? a. "Have you recently taken any antihistamines?" b. "Have you consistently taken your medications?" c. "Did you take any acetaminophen (Tylenol) today?" d. "Have there been recent stressful events in your life?"

ANS: B Sudden withdrawal of antihypertensive medications can cause rebound hypertension and hypertensive crisis. Although many over-the-counter medications can cause hypertension, antihistamines and acetaminophen do not increase BP. Stressful events will increase BP but not usually to the level seen in this patient.

A patient with a history of hypertension treated with a diuretic and an angiotensin-converting enzyme (ACE) inhibitor arrives in the emergency department complaining of a severe headache and nausea and has a blood pressure (BP) of 238/118 mm Hg. Which question should the nurse ask to follow up on these findings? a. "Have you recently taken any antihistamines?" b. "Have you consistently taken your medications?" c. "Did you take any acetaminophen (Tylenol) today?" d. "Have there been recent stressful events in your life?"

ANS: B Sudden withdrawal of antihypertensive medications can cause rebound hypertension and hypertensive crisis. Although many over-the-counter medications can cause hypertension, antihistamines and acetaminophen do not increase BP. Stressful events will increase BP but not usually to the level seen in this patient. DIF: Cognitive Level: Apply (application)

Which blood pressure (BP) finding by the nurse indicates that no changes in therapy are needed for a 48-yr-old patient with newly diagnosed hypertension? a. 98/56 mm Hg c. 128/92 mm Hg b. 128/76 mm Hg d. 142/78 mm Hg

ANS: B The 8th Joint National Committee's recommended goal for antihypertensive therapy for a 30- to 59-yr-old patient with hypertension is a BP below 140/90 mm Hg. The BP of 98/56 mm Hg may indicate overtreatment of the hypertension and an increased risk for adverse drug effects. The other two blood pressures indicate a need for modifications in the patient's treatment.

Which blood pressure (BP) finding by the nurse indicates that no changes in therapy are needed for a 48-yr-old patient with newly diagnosed hypertension? a. 98/56 mm Hg b. 128/76 mm Hg c. 128/92 mm Hg d. 142/78 mm Hg

ANS: B The 8th Joint National Committee's recommended goal for antihypertensive therapy for a 30- to 59-yr-old patient with hypertension is a BP below 140/90 mm Hg. The BP of 98/56 mm Hg may indicate overtreatment of the hypertension and an increased risk for adverse drug effects. The other two blood pressures indicate a need for modifications in the patient's treatment. DIF: Cognitive Level: Apply (application)

28. Which assessment finding by the nurse caring for a patient who has had coronary artery bypass grafting using a right radial artery graft is most important to communicate to the health care provider? a. Complaints of incisional chest pain b. Pallor and weakness of the right hand c. Fine crackles heard at both lung bases d. Redness on both sides of the sternal incision

ANS: B The changes in the right hand indicate compromised blood flow, which requires immediate evaluation and actions such as prescribed calcium channel blockers or surgery. The other changes are expected and/or require nursing interventions.

Which blood pressure (BP) finding by the nurse indicates that no changes in therapy are needed for a patient with stage 1 hypertension who has a history of diabetes mellitus? a. 102/60 mm Hg b. 128/76 mm Hg c. 139/90 mm Hg d. 136/82 mm Hg

ANS: B The goal for antihypertensive therapy for a patient with hypertension and diabetes mellitus is a BP <130/80 mm Hg. The BP of 102/60 may indicate overtreatment of the hypertension and an increased risk for adverse drug effects. The other two blood pressures indicate a need for modifications in the patient's treatment

13. When titrating IV nitroglycerin (Tridil) for a patient with a myocardial infarction (MI), which action will the nurse take to evaluate the effectiveness of the medication? a. Monitor heart rate. b. Ask about chest pain. c. Check blood pressure. d. Observe for dysrhythmias.

ANS: B The goal of IV nitroglycerin administration in MI is relief of chest pain by improving the balance between myocardial oxygen supply and demand. The nurse also will monitor heart rate and blood pressure (BP) and observe for dysrhythmias, but these parameters will not indicate whether the medication is effective.

40. Which patient at the cardiovascular clinic requires the most immediate action by the nurse? a. Patient with type 2 diabetes whose current blood glucose level is 145 mg/dL b. Patient with stable angina whose chest pain has recently increased in frequency c. Patient with familial hypercholesterolemia and a total cholesterol of 465 mg/dL d. Patient with chronic hypertension whose blood pressure today is 172/98 mm Hg

ANS: B The history of more frequent chest pain suggests that the patient may have unstable angina, which is part of the acute coronary syndrome spectrum. This will require rapid implementation of actions such as cardiac catheterization and possible percutaneous coronary intervention. The data about the other patients suggest that their conditions are stable.

When developing a community health program to decrease the incidence of rheumatic fever, which action would be most important for the community health nurse to include? a. Vaccinate high-risk groups in the community with streptococcal vaccine. b. Teach community members to seek treatment for streptococcal pharyngitis. c. Teach about the importance of monitoring temperature when sore throats occur. d. Teach about prophylactic antibiotics to those with a family history of rheumatic fever.

ANS: B The incidence of rheumatic fever is decreased by treatment of streptococcal infections with antibiotics. Family history is not a risk factor for rheumatic fever. There is no immunization that is effective in decreasing the incidence of rheumatic fever. Teaching about monitoring temperature will not decrease the incidence of rheumatic fever.

Which action will the nurse in the hypertension clinic take in order to obtain an accurate baseline blood pressure (BP) for a new patient? a. Deflate the BP cuff at a rate of 5 to 10 mm Hg per second. b. Have the patient sit in a chair with the feet flat on the floor. c. Assist the patient to the supine position for BP measurements. d. Obtain two BP readings in the dominant arm and average the results.

ANS: B The patient should be seated with the feet flat on the floor. The BP is obtained in both arms, and the results of the two arms are not averaged. The patient does not need to be in the supine position. The cuff should be deflated at 2 to 3 mm Hg per second

Which action will the nurse in the hypertension clinic take to obtain an accurate baseline blood pressure (BP) for a new patient? a. Deflate the BP cuff at a rate of 5 to 10 mm Hg per second. b. Have the patient sit in a chair with the feet flat on the floor. c. Assist the patient to the supine position for BP measurements. d. Obtain two BP readings in the dominant arm and average the results.

ANS: B The patient should be seated with the feet flat on the floor. The BP is obtained in both arms, and the results of the two arms are not averaged. The patient does not need to be in the supine position. The cuff should be deflated at 2 to 3 mm Hg per second.

Which action will the nurse in the hypertension clinic take to obtain an accurate baseline blood pressure (BP) for a new patient? a. Deflate the BP cuff at a rate of 5 to 10 mm Hg per second. b. Have the patient sit in a chair with the feet flat on the floor. c. Assist the patient to the supine position for BP measurements. d. Obtain two BP readings in the dominant arm and average the results.

ANS: B The patient should be seated with the feet flat on the floor. The BP is obtained in both arms, and the results of the two arms are not averaged. The patient does not need to be in the supine position. The cuff should be deflated at 2 to 3 mm Hg per second. DIF: Cognitive Level: Understand (comprehension)

After receiving the following information about four patients during change-of-shift report, which patient should the nurse assess first? a. Patient with acute pericarditis who has a pericardial friction rub b. Patient who has just returned to the unit after balloon valvuloplasty c. Patient who has hypertrophic cardiomyopathy and a heart rate of 116 d. Patient with a mitral valve replacement who has an anticoagulant scheduled

ANS: B The patient who has just arrived after balloon valvuloplasty will need assessment for complications such as bleeding and hypotension. The information about the other patients is consistent with their diagnoses and does not indicate any complications or need for urgent assessment or intervention.

A patient whose heart monitor shows sinus tachycardia, rate 132, is apneic, and has no palpable pulses. What action should the nurse take next? a. Perform synchronized cardioversion. b. Start cardiopulmonary resuscitation (CPR). c. Give atropine per agency dysrhythmia protocol. d. Provide supplemental O2 via non-rebreather mask.

ANS: B The patient's clinical manifestations indicate pulseless electrical activity, and the nurse should immediately start CPR. The other actions would not be of benefit to this patient. DIF: Cognitive Level: Apply (application)

The nurse is assessing a patient who has been admitted to the intensive care unit (ICU) with a hypertensive emergency. Which finding is most important to report to the health care provider? a. Urine output over 8 hours is 250 mL less than the fluid intake. b. The patient cannot move the left arm and leg when asked to do so. c. Tremors are noted in the fingers when the patient extends the arms. d. The patient complains of a headache with pain at level 8/10 (0 to 10 scale).

ANS: B The patient's inability to move the left arm and leg indicates that a hemorrhagic stroke may be occurring and will require immediate action to prevent further neurologic damage. The other clinical manifestations are also likely caused by the hypertension and will require rapid nursing actions, but they do not require action as urgently as the neurologic changes

The nurse is assessing a patient who has been admitted to the intensive care unit (ICU) with a hypertensive emergency. Which finding is most important to report to the health care provider? a. Urine output over 8 hours is 250 mL less than the fluid intake. b. The patient cannot move the left arm and leg when asked to do so. c. Tremors are noted in the fingers when the patient extends the arms. d. The patient complains of a headache with pain at level 7 of 10 (0 to 10 scale).

ANS: B The patient's inability to move the left arm and leg indicates that a stroke may be occurring and will require immediate action to prevent further neurologic damage. The other clinical manifestations are also likely caused by the hypertension and will require rapid nursing actions, but they do not require action as urgently as the neurologic changes.

The nurse is assessing a patient who has been admitted to the intensive care unit (ICU) with a hypertensive emergency. Which finding is most important to report to the health care provider? a. Urine output over 8 hours is 250 mL less than the fluid intake. b. The patient cannot move the left arm and leg when asked to do so. c. Tremors are noted in the fingers when the patient extends the arms. d. The patient complains of a headache with pain at level 7 of 10 (0 to 10 scale).

ANS: B The patient's inability to move the left arm and leg indicates that a stroke may be occurring and will require immediate action to prevent further neurologic damage. The other clinical manifestations are also likely caused by the hypertension and will require rapid nursing actions, but they do not require action as urgently as the neurologic changes. DIF: Cognitive Level: Analyze (analysis)

A patient with rheumatic fever has subcutaneous nodules, erythema marginatum, and polyarthritis. Based on these findings, which nursing diagnosis would be most appropriate? a. Pain related to permanent joint fixation b. Activity intolerance related to arthralgia c. Risk for infection related to open skin lesions d. Risk for impaired skin integrity related to pruritus

ANS: B The patient's joint pain will lead to difficulty with activity. The skin lesions seen in rheumatic fever are not open or pruritic. Although acute joint pain will be a problem for this patient, joint inflammation is a temporary clinical manifestation of rheumatic fever and is not associated with permanent joint changes.

After receiving change of shift report, which patient admitted to the emergency department should the nurse assess first? a. A 67-yr-old patient who has a gangrenous left foot ulcer with a weak pedal pulse b. A 50-yr-old patient who is complaining of sudden sharp and severe upper back pain c. A 39-yr-old patient who has right calf tenderness, redness, and swelling after a plane ride d. A 58-yr-old patient who is taking anticoagulants for atrial fibrillation and has black stools

ANS: B The patient's presentation of sudden sharp and severe upper back pain is consistent with dissecting thoracic aneurysm, which will require the most rapid intervention. The other patients also require rapid intervention but not before the patient with severe pain.

A patient's heart monitor shows a pattern of undulations of varying contours and amplitude with no measurable ECG pattern. The patient is unconscious, apneic, and pulseless. Which action should the nurse take first? a. Give epinephrine (Adrenalin) IV. b. Perform immediate defibrillation. c. Prepare for endotracheal intubation. d. Ventilate with a bag-valve-mask device.

ANS: B The patient's rhythm and assessment indicate ventricular fibrillation and cardiac arrest; the initial action should be to defibrillate. If a defibrillator is not immediately available or is unsuccessful in converting the patient to a better rhythm, begin chest compressions. The other actions may also be appropriate but not first. DIF: Cognitive Level: Analyze (analysis)

The nurse obtains the following information from a patient newly diagnosed with prehypertension. Which finding is most important to address with the patient? a. Low dietary fiber intake b. No regular aerobic exercise c. Weight 5 pounds above ideal weight d. Drinks a beer with dinner on most nights

ANS: B The recommendations for preventing hypertension include exercising aerobically for 30 minutes most days of the week. A weight that is 5 pounds over the ideal body weight is not a risk factor for hypertension. The Dietary Approaches to Stop Hypertension (DASH) diet is high in fiber, but increasing fiber alone will not prevent hypertension from developing. The patient's alcohol intake is within guidelines and will not increase the hypertension risk

When planning care for a patient hospitalized with a streptococcal infective endocarditis (IE), which intervention is a priority for the nurse to include? a. Monitor labs for streptococcal antibodies. b. Arrange for placement of a long-term IV catheter. c. Teach the importance of completing all oral antibiotics. d. Encourage the patient to begin regular aerobic exercise.

ANS: B Treatment for IE involves 4 to 6 weeks of IV antibiotic therapy in order to eradicate the bacteria, which will require a long-term IV catheter such as a peripherally inserted central catheter (PICC) line. Rest periods and limiting physical activity to a moderate level are recommended during the treatment for IE. Oral antibiotics are not effective in eradicating the infective bacteria that cause IE. Blood cultures, rather than antibody levels, are used to monitor the effectiveness of antibiotic therapy.

Which admission order written by the health care provider for a patient admitted with infective endocarditis (IE) and a fever would be a priority for the nurse to implement? a. Administer ceftriaxone (Rocephin) 1 g IV. b. Order blood cultures drawn from two sites. c. Give acetaminophen (Tylenol) PRN for fever. d. Arrange for a transesophageal echocardiogram.

ANS: B Treatment of the IE with antibiotics should be started as quickly as possible, but it is essential to obtain blood cultures before initiating antibiotic therapy to obtain accurate sensitivity results. The echocardiogram and acetaminophen administration also should be implemented rapidly, but the blood cultures (and then administration of the antibiotic) have the highest priority.

Which nursing action can the registered nurse (RN) delegate to experienced unlicensed assistive personnel (UAP) working as telemetry technicians on the cardiac care unit? a. Decide whether a patient's heart rate of 116 requires urgent treatment. b. Observe heart rhythms for multiple patients who have telemetry monitoring. c. Monitor a patient's level of consciousness during synchronized cardioversion. d. Select the best lead for monitoring a patient admitted with acute coronary syndrome.

ANS: B UAP serving as telemetry technicians can monitor heart rhythms for individuals or groups of patients. Nursing actions such as assessment and choice of the most appropriate lead based on ST segment elevation location require RN-level education and scope of practice. DIF: Cognitive Level: Apply (application)

The nurse notes that a patient's heart monitor shows that every other beat is earlier than expected, has no visible P wave, and has a QRS complex that is wide and bizarre in shape. How will the nurse document the rhythm? a. Ventricular couplets b. Ventricular bigeminy c. Ventricular R-on-T phenomenon d. Multifocal premature ventricular contractions

ANS: B Ventricular bigeminy describes a rhythm in which every other QRS complex is wide and bizarre looking. Pairs of wide QRS complexes are described as ventricular couplets. There is no indication that the premature ventricular contractions are multifocal or that the R-on-T phenomenon is occurring. DIF: Cognitive Level: Apply (application)

Which action should the nurse perform when preparing a patient with supraventricular tachycardia for cardioversion who is alert and has a blood pressure of 110/66 mm Hg? a. Turn the synchronizer switch to the "off" position. b. Give a sedative before cardioversion is implemented. c. Set the defibrillator/cardioverter energy to 360 joules. d. Provide assisted ventilations with a bag-valve-mask device.

ANS: B When a patient has a nonemergency cardioversion, sedation is used just before the procedure. The synchronizer switch is turned "on" for cardioversion. The initial level of joules for cardioversion is low (e.g., 50). Assisted ventilations are not indicated for this patient. DIF: Cognitive Level: Apply (application)

22. When evaluating the effectiveness of preoperative teaching with a patient scheduled for coronary artery bypass graft (CABG) surgery using the internal mammary artery, the nurse determines that additional teaching is needed when the patient says which of the following? a. "They will circulate my blood with a machine during the surgery." b. "I will have small incisions in my leg where they will remove the vein." c. "They will use an artery near my heart to go around the area that is blocked." d. "I will need to take an aspirin every day after the surgery to keep the graft open."

ANS: B When the internal mammary artery is used there is no need to have a saphenous vein removed from the leg. The other statements by the patient are accurate and indicate that the teaching has been effective.

A 53-year-old patient with Stage D heart failure and type 2 diabetes asks the nurse whether heart transplant is a possible therapy. Which response by the nurse is most appropriate? a. Because you have diabetes, you would not be a candidate for a heart transplant b. The choice of a patient for a heart transplant depends on many different factors c. Your heart failure has not reached the stage in which heart transplants are needed d. People who have heart transplants are at risk for multiple complications after surgery.

ANS: B Indications for a heart transplant include end-stage heart failure (Stage D), but other factors such as coping skills, family support, and patient motivation to follow the rigorous posttransplant regimen are also considered. Diabetic patients who have well-controlled blood glucose levels may be candidates for heart transplant. Although heart transplants can be associated with many complications, this response does not address the patient's question.

A patient is scheduled for a cardiac catheterization with coronary angiography. Before the test, the nurse informs the patient that a. it will be important not to move at all during the procedure. b. monitored anesthesia care will be provided during the procedure. c. a flushed feeling may be noticed when the contrast dye is injected. d. arterial pressure monitoring will be required for 24 hours after the test.

ANS: C A sensation of warmth or flushing is common when the contrast material is injected, which can be anxiety producing unless it has been discussed with the patient. The patient may receive a sedative drug before the procedure, but monitored anesthesia care is not used. Arterial pressure monitoring is not routinely used after the procedure to monitor blood pressure. The patient is not immobile during cardiac catheterization and may be asked to cough or take deep breaths.

When the nurse is monitoring a patient who is undergoing exercise (stress) testing on a treadmill, which assessment finding requires the most rapid action by the nurse? a. Patient complaint of feeling tired b. Sinus tachycardia at a rate of 110 beats/min c. Inversion of T waves on the electrocardiogram d. Blood pressure (BP) increase from 134/68 to 150/80 mm Hg

ANS: C ECG changes associated with coronary ischemia (such as T-wave inversions and ST segment depression) indicate that the myocardium is not getting adequate O2 delivery and that the exercise test should be terminated immediately. Increases in BP and heart rate are normal responses to aerobic exercise. Feeling tired is also normal as the intensity of exercise increases during the stress testing.

Which information obtained by the nurse who is admitting the patient for magnetic resonance imaging (MRI) will be important to report to the health care provider before the MRI? a. The patient has an allergy to shellfish. b. The patient has a history of atherosclerosis. c. The patient has a permanent cardiac pacemaker. d. The patient took the prescribed heart medications today.

ANS: C MRI is contraindicated for patients with implanted metallic devices such as pacemakers. The other information does not affect whether or not the patient can have an MRI.

A patient who has chronic heart failure tells the nurse, I was fine when I went to bed, but I woke up in the middle of the night feeling like I was suffocating! The nurse will document this assessment finding as a. Orthopnea b. Pulsus alternans c. Paroxysmal nocturnal dyspnea d. Acute bilateral pleural effusion

ANS: C Paroxysmal nocturnal dyspnea is caused by the reabsorption of fluid from dependent body ares when the patient is sleeping and is characterized by waking up suddenly with the feeling of suffocation. Pulsus alternans is the alternation of strong and weak peripheral pulses during palpation. Orthopnea indicates that the patient is unable to lie flat because of dyspnea. Pleural effusions develop over a longer time period.

IV sodium nitroprusside is ordered for a patient with acute pulmonary edema. During the first hours of administration, the nurse will need to titrate the nitroprusside rate if the patient develops a. Ventricular ectopy b. A dry, hacking cough c. A systolic BP < 90 mmHg d. A heart rate < 50 beats/minute

ANS: C Sodium nitroprusside is a potent vasodilator, and the major adverse effect is severe hypotension. Coughing and bradycardia are not adverse effects of this medication. Nitroprusside does not cause increased ventricular ectopy.

A patient with chronic heart failure who is taking a diuretic and an ACE inhibitor and who is on a low-sodium diet tells the home health nurse about a 5-pound weight gain in the last 3 days. The nurses priority action will be to a. Have the patient recall the dietary intake for the last 3 days b. Ask the patient abou the use of prescribed medications c. Assess the patient for clinical manifestations of acute heart failure d. Teach the patient about the importance of restricting dietary sodium

ANS: C The 5-pound weight gain over 3 days indicates that the patients chronic heart failure may be worsening. It is important that the patient be assessed immediately for other clinical manifestations of decompensation, such as lung crackles. A dietary recall to detect hidden sodium in the diet, reinforcement of sodium restrictions, and assessment of medication compliance may be appropriate interventions but are not the first nursing actions indicated.

During a physical examination of an older patient, the nurse palpates the point of maximal impulse (PMI) in the sixth intercostal space lateral to the left midclavicular line. The best follow-up action for the nurse to take will be to a. ask about risk factors for atherosclerosis. b. determine family history of heart disease. c. assess for symptoms of left ventricular hypertrophy. d. auscultate carotid arteries for the presence of a bruit.

ANS: C The PMI should be felt at the intersection of the fifth intercostal space and left midclavicular line. A PMI located outside these landmarks indicates possible cardiac enlargement, such as with left ventricular hypertrophy (LVH). The other assessments are part of a general cardiac assessment but do not represent follow-up for LVH. Cardiac enlargement is not necessarily associated with atherosclerosis or carotid artery disease. DIF: Cognitive Level: Analyze (analysis)

While assessing a 68-year-old with ascites, the nurse also notes JVD with the head of the patient's bed elevated 45 degrees. The nurse knows this finding indicates a. decreased fluid volume b. jugular vein atherosclerosis c. increased right atrial pressure d. incompetent jugular vein pressure

ANS: C The jugular veins empty into the superior vena cava and then into the right atrium, so JVD with the patient sitting at a 45-degree angle reflects increased right atrial pressure. JVD is an indicator of excessive fluid volume (increased preload), not decreased fluid volume. JVD is not caused by incompetent jugular vein valves or atherosclerosis.

15. Which action will the nurse implement for a patient who arrives for a calcium- scoring CT scan? a. Insert an IV catheter. b. Administer oral sedative medications. c. Teach the patient about the procedure. d. Confirm that the patient has been fasting.

ANS: C The nurse will need to teach the patient that the procedure is rapid and involves little risk. None of the other actions are necessary.

A transesophageal echocardiogram (TEE) is ordered for a patient with possible endocarditis. Which action included in the standard TEE orders will the nurse need to accomplish first? a. Start an IV line. c. Place the patient on NPO status. b. Start O2 per nasal cannula. d. Give lorazepam (Ativan) 1 mg IV.

ANS: C The patient will need to be NPO for 6 hours preceding the TEE, so the nurse should place the patient on NPO status as soon as the order is received. The other actions also will need to be accomplished but not until just before or during the procedure.

A patient with a history of chronic heart failure is admitted to the ED with severe dyspnea and a dry, hacking cough. Which action should the nurse do first? a. Auscultate the abdomen b. Check capillary refill c. Auscultate the breath sounds d. Assess level of orientation

ANS: C This patient's severe dyspnea and cough indicate that acute decompensated heart failure is occurring. ADHF usually manifests as pulmonary edema, which should be detected and treated immediately to prevent ongoing volume status and also should accomplished rapidly, but detection (and treatment) of pulmonary complications is the priority.

15. Following an acute myocardial infarction (AMI), a patient ambulates in the hospital hallway. When the nurse is evaluating the patient's response to the activity, which assessment data would indicate that the exercise level should be decreased? a. Blood pressure (BP) changes from 118/60 to 126/68 mm Hg. b. Oxygen saturation drops from 99% to 95%. c. Heart rate increases from 66 to 92 beats/minute. d. Respiratory rate goes from 14 to 20 breaths/minute.

ANS: C A change in heart rate of more than 20 beats over the resting heart rate indicates that the patient should stop and rest. The increases in BP and respiratory rate, and the slight decrease in oxygen saturation, are normal responses to exercise.

A 56-yr-old patient who has no previous history of hypertension or other health problems suddenly develops a blood pressure (BP) of 198/110 mm Hg. After reconfirming the BP, it is appropriate for the nurse to tell the patient that a. a BP recheck should be scheduled in a few weeks. b. dietary sodium and fat content should be decreased. c. diagnosis, treatment, and ongoing monitoring will be needed. d. there is an immediate danger of a stroke, requiring hospitalization.

ANS: C A sudden increase in BP in a patient older than age 50 years with no previous hypertension history or risk factors indicates that the hypertension may be secondary to some other problem. The BP will need treatment and ongoing monitoring. If the patient has no other risk factors, a stroke in the immediate future is unlikely. There is no indication that dietary salt or fat intake have contributed to this sudden increase in BP, and reducing intake of salt and fat alone will not be adequate to reduce this BP to an acceptable level.

A 56-yr-old patient who has no previous history of hypertension or other health problems suddenly develops a blood pressure (BP) of 198/110 mm Hg. After reconfirming the BP, it is appropriate for the nurse to tell the patient that a. a BP recheck should be scheduled in a few weeks. b. dietary sodium and fat content should be decreased. c. diagnosis, treatment, and ongoing monitoring will be needed. d. there is an immediate danger of a stroke, requiring hospitalization.

ANS: C A sudden increase in BP in a patient older than age 50 years with no previous hypertension history or risk factors indicates that the hypertension may be secondary to some other problem. The BP will need treatment and ongoing monitoring. If the patient has no other risk factors, a stroke in the immediate future is unlikely. There is no indication that dietary salt or fat intake have contributed to this sudden increase in BP, and reducing intake of salt and fat alone will not be adequate to reduce this BP to an acceptable level. DIF: Cognitive Level: Apply (application)

32. A patient admitted to the coronary care unit (CCU) with an ST-segment-elevation myocardial infarction (STEMI) is restless and anxious. The blood pressure is 86/40 and heart rate is 123. Based on this information, which nursing diagnosis is a priority for the patient? a. Acute pain related to myocardial infarction b. Anxiety related to perceived threat of death c. Stress overload related to acute change in health d. Decreased cardiac output related to cardiogenic shock

ANS: C All the nursing diagnoses may be appropriate for this patient, but the hypotension and tachycardia indicate decreased cardiac output and shock from the damaged myocardium. This will result in decreased perfusion to all vital organs (e.g., brain, kidney, heart) and is a priority.

During discharge teaching with a 68-year-old patient who had a mitral valve replacement with a mechanical valve, the nurse instructs the patient on the a. use of daily aspirin for anticoagulation. b. correct method for taking the radial pulse. c. need for frequent laboratory blood testing. d. need to avoid any physical activity for 1 month.

ANS: C Anticoagulation with warfarin (Coumadin) is needed for a patient with mechanical valves to prevent clotting on the valve. This will require frequent international normalized ratio (INR) testing. Daily aspirin use will not be effective in reducing the risk for clots on the valve. Monitoring of the radial pulse is not necessary after valve replacement. Patients should resume activities of daily living as tolerated.

Which action by the nurse will determine if the therapies ordered for a patient with chronic constrictive pericarditis are effective? a. Assess for the presence of a paradoxical pulse. b. Monitor for changes in the patient's sedimentation rate. c. Assess for the presence of jugular venous distention (JVD). d. Check the electrocardiogram (ECG) for ST segment changes.

ANS: C Because the most common finding on physical examination for a patient with chronic constrictive pericarditis is jugular venous distention, a decrease in JVD indicates improvement. Paradoxical pulse, ST-segment ECG changes, and changes in sedimentation rates occur with acute pericarditis but are not expected in chronic constrictive pericarditis.

A patient who is on the telemetry unit develops atrial flutter, rate 150, with associated dyspnea and chest pain. Which action that is included in the hospital dysrhythmia protocol should the nurse do first? a. Obtain a 12-lead electrocardiogram (ECG). b. Notify the health care provider of the change in rhythm. c. Give supplemental O2 at 2 to 3 L/min via nasal cannula. d. Assess the patient's vital signs including O2 saturation.

ANS: C Because this patient has dyspnea and chest pain in association with the new rhythm, the nurse's initial actions should be to address the patient's airway, breathing, and circulation (ABC) by starting with O2 administration. The other actions are also important and should be implemented rapidly. DIF: Cognitive Level: Analyze (analysis)

The nurse performing an assessment of a patient who has chronic peripheral artery disease (PAD) of the legs and an ulcer on the right second toe would expect to find a. dilated superficial veins. b. swollen, dry, scaly ankles. c. prolonged capillary refill in all the toes. d. serosanguineous drainage from the ulcer

ANS: C Capillary refill is prolonged in PAD because of the slower and decreased blood flow to the periphery. The other listed clinical manifestations are consistent with chronic venous disease

The nurse will plan discharge teaching about the need for prophylactic antibiotics when having dental procedures for which patient? a. Patient admitted with a large acute myocardial infarction. b. Patient being discharged after an exacerbation of heart failure. c. Patient who had a mitral valve replacement with a mechanical valve. d. Patient being treated for rheumatic fever after a streptococcal infection.

ANS: C Current American Heart Association guidelines recommend the use of prophylactic antibiotics before dental procedures for patients with prosthetic valves to prevent infective endocarditis (IE). The other patients are not at risk for IE.

A 46-yr-old service-counter worker undergoes sclerotherapy for treatment of superficial varicose veins at an outpatient center. Which instructions should the nurse provide to the patient before discharge? a. Sitting at the work counter, rather than standing, is recommended. b. Exercise, such as walking or jogging, can cause recurrence of varicosities. c. Elastic compression stockings should be applied before getting out of bed. d. Taking an aspirin daily will help prevent clots from forming around venous valves.

ANS: C Elastic compression stockings are applied with the legs elevated to reduce pressure in the lower legs. Walking is recommended to prevent recurrent varicosities. Sitting and standing are both risk factors for varicose veins and venous insufficiency. An aspirin a day is not adequate to prevent venous thrombosis and would not be recommended for a patient who had just had sclerotherapy.

When caring for a patient with infective endocarditis of the tricuspid valve, the nurse should monitor the patient for the development of a. flank pain. b. splenomegaly. c. shortness of breath. d. mental status changes.

ANS: C Embolization from the tricuspid valve would cause symptoms of pulmonary embolus. Flank pain, changes in mental status, and splenomegaly would be associated with embolization from the left-sided valves.

After the nurse teaches the patient with stage 1 hypertension about diet modifications that should be implemented, which diet choice indicates that the teaching has been most effective? a. The patient avoids eating nuts or nut butters. b. The patient restricts intake of chicken and fish. c. The patient drinks low-fat milk with each meal. d. The patient has two cups of coffee in the morning.

ANS: C For the prevention of hypertension, the Dietary Approaches to Stop Hypertension (DASH) recommendations include increasing the intake of calcium-rich foods. Caffeine restriction and decreased protein intake are not included in the recommendations. Nuts are high in beneficial nutrients and 4 to 5 servings weekly are recommended in the DASH diet.

After the nurse teaches the patient with stage 1 hypertension about diet modifications that should be implemented, which diet choice indicates that the teaching has been most effective? a. The patient avoids eating nuts or nut butters. b. The patient restricts intake of chicken and fish. c. The patient drinks low-fat milk with each meal. d. The patient has two cups of coffee in the morning.

ANS: C For the prevention of hypertension, the Dietary Approaches to Stop Hypertension (DASH) recommendations include increasing the intake of calcium-rich foods. Caffeine restriction and decreased protein intake are not included in the recommendations. Nuts are high in beneficial nutrients and 4 to 5 servings weekly are recommended in the DASH diet. DIF: Cognitive Level: Apply (application)

An older patient has been diagnosed with possible white coat hypertension. Which action will the nurse plan to take next? a. Schedule the patient for regular blood pressure (BP) checks in the clinic. b. Instruct the patient about the need to decrease stress levels. c. Tell the patient how to self-monitor and record BPs at home. d. Inform the patient that ambulatory blood pressure monitoring will be needed.

ANS: C Having the patient self-monitor BPs at home will provide a reliable indication about whether the patient has hypertension. Regular BP checks in the clinic are likely to be high in a patient with white coat hypertension. Ambulatory blood pressure monitoring may be used if the data from self-monitoring are unclear. Although elevated stress levels may contribute to hypertension, instructing the patient about this is unlikely to reduce BP

12. Heparin is ordered for a patient with a non-ST-segment-elevation myocardial infarction (NSTEMI). What is the purpose of the heparin? a. Heparin enhances platelet aggregation. b. Heparin decreases coronary artery plaque size. c. Heparin prevents the development of new clots in the coronary arteries. d. Heparin dissolves clots that are blocking blood flow in the coronary arteries.

ANS: C Heparin helps prevent the conversion of fibrinogen to fibrin and decreases coronary artery thrombosis. It does not change coronary artery plaque, dissolve already formed clots, or enhance platelet aggregation.

Which information is most important for the nurse to include when teaching a patient with newly diagnosed hypertension? a. Most people are able to control BP through dietary changes. b. Annual BP checks are needed to monitor treatment effectiveness. c. Hypertension is usually asymptomatic until target organ damage occurs. d. Increasing physical activity alone controls blood pressure (BP) for most people.

ANS: C Hypertension is usually asymptomatic until target organ damage has occurred. Lifestyle changes (e.g., physical activity, dietary changes) are used to help manage BP, but drugs are needed for most patients. Home BP monitoring should be taught to the patient and findings checked by the health care provider frequently when starting treatment for hypertension and then every 3 months when stable.

Which information is most important for the nurse to include when teaching a patient with newly diagnosed hypertension? a. Most people are able to control BP through dietary changes. b. Annual BP checks are needed to monitor treatment effectiveness. c. Hypertension is usually asymptomatic until target organ damage occurs. d. Increasing physical activity alone controls blood pressure (BP) for most people.

ANS: C Hypertension is usually asymptomatic until target organ damage has occurred. Lifestyle changes (e.g., physical activity, dietary changes) are used to help manage BP, but drugs are needed for most patients. Home BP monitoring should be taught to the patient and findings checked by the health care provider frequently when starting treatment for hypertension and then every 3 months when stable. DIF: Cognitive Level: Apply (application)

A patient has a junctional escape rhythm on the monitor. The nurse will expect the patient to have a heart rate of _____ beats/min. a. 15 to 20 c. 40 to 60 b. 20 to 40 d. 60 to 100

ANS: C If the sinoatrial (SA) node fails to discharge, the atrioventricular (AV) node will automatically discharge at the normal rate of 40 to 60 beats/minute. The slower rates are typical of the bundle of His and Purkinje system and may be seen with failure of both the SA and AV node to discharge. The normal SA node rate is 60 to 100 beats/min.

A 19-yr-old student comes to the student health center at the end of the semester complaining that, "My heart is skipping beats." An electrocardiogram (ECG) shows occasional unifocal premature ventricular contractions (PVCs). What action should the nurse take next? a. Insert an IV catheter for emergency use. b. Start supplemental O2 at 2 to 3 L/min via nasal cannula. c. Ask the patient about current stress level and caffeine use. d. Have the patient taken to the nearest emergency department (ED)

ANS: C In a patient with a normal heart, occasional PVCs are a benign finding. The timing of the PVCs suggests stress or caffeine as possible etiologic factors. The patient is hemodynamically stable, so there is no indication that the patient needs supplemental O2, an IV, or to be seen in the ED. DIF: Cognitive Level: Apply (application)

A 19-yr-old student comes to the student health center at the end of the semester complaining that, "My heart is skipping beats." An electrocardiogram (ECG) shows occasional unifocal premature ventricular contractions (PVCs). What action should the nurse take next? a. Insert an IV catheter for emergency use. b. Start supplemental O2 at 2 to 3 L/min via nasal cannula. c. Ask the patient about current stress level and caffeine use. d. Have the patient taken to the nearest emergency department (ED).

ANS: C In a patient with a normal heart, occasional PVCs are a benign finding. The timing of the PVCs suggests stress or caffeine as possible etiologic factors. The patient is hemodynamically stable, so there is no indication that the patient needs supplemental O2, an IV, or to be seen in the ED. DIF: Cognitive Level: Apply (application)

The registered nurse (RN) is caring for a patient with a hypertensive crisis who is receiving sodium nitroprusside (Nipride). Which nursing action can the nurse delegate to an experienced licensed practical/vocational nurse (LPN/LVN)? a. Titrate nitroprusside to decrease mean arterial pressure (MAP) to 115 mm Hg. b. Evaluate effectiveness of nitroprusside therapy on blood pressure (BP). c. Set up the automatic blood pressure machine to take BP every 15 minutes. d. Assess the patient's environment for adverse stimuli that might increase BP.

ANS: C LPN/LVN education and scope of practice include the correct use of common equipment such as automatic blood pressure machines. The other actions require advanced nursing judgment and education, and should be done by RNs

Which action should the nurse take when administering the initial dose of oral labetalol (Normodyne) to a patient with hypertension? a. Encourage the use of hard candy to prevent dry mouth. b. Instruct the patient to ask for help if heart palpitations occur. c. Ask the patient to request assistance when getting out of bed. d. Teach the patient that headaches may occur with this medication.

ANS: C Labetalol decreases sympathetic nervous system activity by blocking both á- and b-adrenergic receptors, leading to vasodilation and a decrease in heart rate, which can cause severe orthostatic hypotension. Heart palpitations, dry mouth, dehydration, and headaches are possible side effects of other antihypertensives

2. Which nursing intervention will be most effective when assisting the patient with coronary artery disease (CAD) to make appropriate dietary changes? a. Give the patient a list of low-sodium, low-cholesterol foods that should be included in the diet. b. Emphasize the increased risk for heart problems unless the patient makes the dietary changes. c. Help the patient modify favorite high-fat recipes by using monosaturated oils when possible. d. Inform the patient that a diet containing no saturated fat and minimal salt will be necessary.

ANS: C Lifestyle changes are more likely to be successful when consideration is given to the patient's values and preferences. The highest percentage of calories from fat should come from monosaturated fats. Although low-sodium and low-cholesterol foods are appropriate, providing the patient with a list alone is not likely to be successful in making dietary changes. Completely removing saturated fat from the diet is not a realistic expectation. Up to 7% of calories in the therapeutic lifestyle changes (TLC) diet can come from saturated fat. Telling the patient about the increased risk without assisting further with strategies for dietary change is unlikely to be successful.

A 21-year-old woman is scheduled for percutaneous transluminal balloon valvuloplasty to treat mitral stenosis. Which information should the nurse include when explaining the advantages of valvuloplasty over valve replacement to the patient? a. Biologic valves will require immunosuppressive drugs after surgery. b. Mechanical mitral valves need to be replaced sooner than biologic valves. c. Lifelong anticoagulant therapy will be needed after mechanical valve replacement. d. Ongoing cardiac care by a health care provider is not necessary after valvuloplasty.

ANS: C Long-term anticoagulation therapy is needed after mechanical valve replacement, and this would restrict decisions about career and childbearing in this patient. Mechanical valves are durable and last longer than biologic valves. All valve repair procedures are palliative, not curative, and require lifelong health care. Biologic valves do not activate the immune system, and immunosuppressive therapy is not needed.

Which assessment finding for a patient who has been admitted with a right calf venous thromboembolism (VTE) requires immediate action by the nurse? a. Erythema of right lower leg c. New onset shortness of breath b. Complaint of right calf pain d. Temperature of 100.4°F (38°C)

ANS: C New onset dyspnea suggests a pulmonary embolus, which will require rapid actions such as O2 administration and notification of the health care provider. The other findings are typical of VTE

Propranolol (Inderal) is prescribed for a patient diagnosed with hypertension. The nurse should consult with the health care provider before giving this drug when the patient reveals a history of a. daily alcohol use. c. reactive airway disease. b. peptic ulcer disease. d. myocardial infarction (MI).

ANS: C Nonselective -blockers block 1- and 2-adrenergic receptors and can cause bronchospasm, especially in patients with a history of asthma. -Blockers will have no effect on the patient's peptic ulcer disease or alcohol use. -Blocker therapy is recommended after MI.

Propranolol (Inderal) is prescribed for a patient diagnosed with hypertension. The nurse should consult with the health care provider before giving this drug when the patient reveals a history of a. daily alcohol use. c. reactive airway disease. b. peptic ulcer disease. d. myocardial infarction (MI).

ANS: C Nonselective -blockers block 1- and 2-adrenergic receptors and can cause bronchospasm, especially in patients with a history of asthma. -Blockers will have no effect on the patient's peptic ulcer disease or alcohol use. -Blocker therapy is recommended after MI. DIF: Cognitive Level: Apply (application)

Which statement by a patient with restrictive cardiomyopathy indicates that the nurse's discharge teaching about self-management has been most effective? a. "I will avoid taking aspirin or other antiinflammatory drugs." b. "I will need to limit my intake of salt and fluids even in hot weather." c. "I will take antibiotics when my teeth are cleaned at the dental office." d. "I should begin an exercise program that includes things like biking or swimming."

ANS: C Patients with restrictive cardiomyopathy are at risk for infective endocarditis and should use prophylactic antibiotics for any procedure that may cause bacteremia. The other statements indicate a need for more teaching by the nurse. Dehydration and vigorous exercise impair ventricular filling in patients with restrictive cardiomyopathy. There is no need to avoid salt (unless ordered), aspirin, or NSAIDs.

Which information will the nurse include when teaching a patient who is scheduled for a radiofrequency catheter ablation for treatment of atrial flutter? a. The procedure prevents or minimizes the risk for sudden cardiac death. b. The procedure uses cold therapy to stop the formation of the flutter waves. c. The procedure uses electrical energy to destroy areas of the conduction system. d. The procedure stimulates the growth of new conduction pathways between the atria.

ANS: C Radiofrequency catheter ablation therapy uses electrical energy to "burn" or ablate areas of the conduction system as definitive treatment of atrial flutter (i.e., restore normal sinus rhythm) and tachydysrhythmias. All other statements regarding the procedure are incorrect. DIF: Cognitive Level: Apply (application)

A young adult patient tells the health care provider about experiencing cold, numb fingers when running during the winter, and Raynaud's phenomenon is suspected. The nurse will anticipate teaching the patient about tests for a. hyperglycemia. c. autoimmune disorders. b. hyperlipidemia. d. coronary artery disease.

ANS: C Secondary Raynaud's phenomenon may occur in conjunction with autoimmune diseases such as rheumatoid arthritis. Patients should be screened for autoimmune disorders. Raynaud's phenomenon is not associated with hyperlipidemia, hyperglycemia, or coronary artery disease.

The nurse working on the heart failure unit knows that teaching an older female patient with newly diagnosed heart failure is effective when the patient states that a. She will take furosemide (Lasix) every day at bedtime. b. The nitroglycerin patch is applied when any chest pain develops. c. She will call the clinic if her weight goes from 124 to 128 pounds in a week. d. An additional pillow can help her sleep if she is feeling short of breath at night.

ANS: C Teaching for a patient with heart failure includes information about the need to weigh daily & notify the HCP about an increase of 3 pounds in 2 days of 3 to 5 pounds in a week. Nitroglycerin patches are used primarily to reduce preload (not to prevent chest pain) in patients with heart failure and should be used daily, not on an as needed basis. Diuretics should be taken earlier in the day to avoid nocturia and sleep disturbances. The patient should call the clinic if increased orthopnea develops, rather than just compensating by further elevating the HOB.

Which assessment finding obtained by the nurse when assessing a patient with acute pericarditis should be reported immediately to the health care provider? a. Pulsus paradoxus 8 mm Hg b. Blood pressure (BP) of 168/94 c. Jugular venous distention (JVD) to jaw level d. Level 6 (0 to 10 scale) chest pain with a deep breath

ANS: C The JVD indicates that the patient may have developed cardiac tamponade and may need rapid intervention to maintain adequate cardiac output. Hypertension would not be associated with complications of pericarditis, and the BP is not high enough to indicate that there is any immediate need to call the health care provider. A pulsus paradoxus of 8 mm Hg is normal. Level 6/10 chest pain should be treated but is not unusual with pericarditis.

Which action by a new nurse who is giving fondaparinux (Arixtra) to a patient with a lower leg venous thromboembolism (VTE) indicates that more education about the drug is needed? a. The nurse avoids rubbing the injection site after giving the drug. b. The nurse injects the drug into the abdominal subcutaneous tissue. c. The nurse ejects the air bubble from the syringe before giving the drug. d. The nurse does not check partial thromboplastin time (PTT) before giving the drug

ANS: C The air bubble is not ejected before giving fondaparinux to avoid loss of drug. The other actions by the nurse are appropriate for subcutaneous administration of a low molecular weight heparin (LMWH). LMWHs typically do not require ongoing PTT monitoring and dose adjustment.

A patient has just been diagnosed with hypertension and has been started on captopril . Which information is most important to include when teaching the patient about this drug? a. Include high-potassium foods such as bananas in the diet. b. Increase fluid intake if dryness of the mouth is a problem. c. Change position slowly to help prevent dizziness and falls. d. Check blood pressure (BP) in both arms before taking the drug.

ANS: C The angiotensin-converting enzyme (ACE) inhibitors frequently cause orthostatic hypotension, and patients should be taught to change position slowly to allow the vascular system time to compensate for the position change. Increasing fluid intake may counteract the effect of the drug, and the patient is taught to use gum or hard candy to relieve dry mouth. The BP should be taken in the nondominant arm by newly diagnosed patients in the morning, before taking the drug, and in the evening. Because ACE inhibitors cause potassium retention, increased intake of high-potassium foods is inappropriate.

A patient has just been diagnosed with hypertension and has been started on captopril . Which information is most important to include when teaching the patient about this drug? a. Include high-potassium foods such as bananas in the diet. b. Increase fluid intake if dryness of the mouth is a problem. c. Change position slowly to help prevent dizziness and falls. d. Check blood pressure (BP) in both arms before taking the drug.

ANS: C The angiotensin-converting enzyme (ACE) inhibitors frequently cause orthostatic hypotension, and patients should be taught to change position slowly to allow the vascular system time to compensate for the position change. Increasing fluid intake may counteract the effect of the drug, and the patient is taught to use gum or hard candy to relieve dry mouth. The BP should be taken in the nondominant arm by newly diagnosed patients in the morning, before taking the drug, and in the evening. Because ACE inhibitors cause potassium retention, increased intake of high-potassium foods is inappropriate. DIF: Cognitive Level: Apply (application)

A patient who was admitted with a myocardial infarction experiences a 45-second episode of ventricular tachycardia, then converts to sinus rhythm with a heart rate of 98 beats/min. Which action should the nurse take next? a. Immediately notify the health care provider. b. Document the rhythm and continue to monitor the patient. c. Prepare to give IV amiodarone per agency dysrhythmia protocol. d. Perform synchronized cardioversion per agency dysrhythmia protocol.

ANS: C The burst of sustained ventricular tachycardia indicates that the patient has significant ventricular irritability, and antidysrhythmic medication administration is needed to prevent further episodes. The nurse should notify the health care provider after the medication is started. Cardioversion is not indicated given that the patient has returned to a sinus rhythm. Documentation and continued monitoring are not adequate responses to this situation. DIF: Cognitive Level: Analyze (analysis)

16. During the administration of the thrombolytic agent to a patient with an acute myocardial infarction (AMI), the nurse should stop the drug infusion if the patient experiences a. bleeding from the gums. b. increase in blood pressure. c. a decrease in level of consciousness. d. a nonsustained episode of ventricular tachycardia.

ANS: C The change in level of consciousness indicates that the patient may be experiencing intracranial bleeding, a possible complication of thrombolytic therapy. Some bleeding of the gums is an expected side effect of the therapy but not an indication to stop infusion of the thrombolytic medication. A decrease in blood pressure could indicate internal bleeding. A nonsustained episode of ventricular tachycardia is a common reperfusion dysrhythmia and may indicate that the therapy is effective.

35. The nurse obtains the following data when assessing a patient who experienced an ST-segment-elevation myocardial infarction (STEMI) 2 days previously. Which information is most important to report to the health care provider? a. The troponin level is elevated. b. The patient denies ever having a heart attack. c. Bilateral crackles are auscultated in the mid-lower lobes. d. The patient has occasional premature atrial contractions (PACs).

ANS: C The crackles indicate that the patient may be developing heart failure, a possible complication of myocardial infarction (MI). The health care provider may need to order medications such as diuretics or angiotensin-converting enzyme (ACE) inhibitors for the patient. Elevation in troponin level at this time is expected. PACs are not life-threatening dysrhythmias. Denial is a common response in the immediate period after the MI.

5. After the nurse has finished teaching a patient about the use of sublingual nitroglycerin (Nitrostat), which patient statement indicates that the teaching has been effective? a. "I can expect some nausea as a side effect of nitroglycerin." b. "I should only take the nitroglycerin if I start to have chest pain." c. "I will call an ambulance if I still have pain after taking 3 nitroglycerin 5 minutes apart." d. "Nitroglycerin helps prevent a clot from forming and blocking blood flow to my heart."

ANS: C The emergency medical services (EMS) system should be activated when chest pain or other symptoms are not completely relieved after 3 sublingual nitroglycerin tablets taken 5 minutes apart. Nitroglycerin can be taken to prevent chest pain or other symptoms from developing (e.g., before intercourse). Gastric upset (e.g., nausea) is not an expected side effect of nitroglycerin. Nitroglycerin does not impact the underlying pathophysiology of coronary artery atherosclerosis.

A patient reports dizziness and shortness of breath for several days. During heart monitoring in the emergency department (ED), the nurse obtains the following electrocardiographic (ECG) tracing. The nurse interprets this heart rhythm as a. junctional escape rhythm. b. accelerated idioventricular rhythm. c. third-degree atrioventricular (AV) block. d. sinus rhythm with premature atrial contractions (PACs).

ANS: C The inconsistency between the atrial and ventricular rates and the variable PR interval indicate that the rhythm is third-degree AV block. Sinus rhythm with PACs will have a normal rate and consistent PR intervals with occasional PACs. An accelerated idioventricular rhythm will not have visible P waves. DIF: Cognitive Level: Apply (application)

During change-of-shift report, the nurse obtains the following information about a hypertensive patient who received the first dose of nadolol (Corgard) during the previous shift. Which information indicates that the patient needs immediate intervention? a. The patient's pulse has dropped from 68 to 57 beats/min. b. The patient complains that the fingers and toes feel quite cold. c. The patient has developed wheezes throughout the lung fields. d. The patient's blood pressure (BP) reading is now 158/91 mm Hg.

ANS: C The most urgent concern for this patient is the wheezes, which indicate that bronchospasm (a common adverse effect of the noncardioselective -blockers) is occurring. The nurse should immediately obtain an O2 saturation measurement, apply supplemental O2, and notify the health care provider. The mild decrease in heart rate and complaint of cold fingers and toes are associated with -receptor blockade but do not require any change in therapy. The BP reading may indicate that a change in medication type or dose may be indicated. However, this is not as urgently needed as addressing the bronchospasm.

During change-of-shift report, the nurse obtains the following information about a hypertensive patient who received the first dose of nadolol (Corgard) during the previous shift. Which information indicates that the patient needs immediate intervention? a. The patient's pulse has dropped from 68 to 57 beats/min. b. The patient complains that the fingers and toes feel quite cold. c. The patient has developed wheezes throughout the lung fields. d. The patient's blood pressure (BP) reading is now 158/91 mm Hg.

ANS: C The most urgent concern for this patient is the wheezes, which indicate that bronchospasm (a common adverse effect of the noncardioselective -blockers) is occurring. The nurse should immediately obtain an O2 saturation measurement, apply supplemental O2, and notify the health care provider. The mild decrease in heart rate and complaint of cold fingers and toes are associated with -receptor blockade but do not require any change in therapy. The BP reading may indicate that a change in medication type or dose may be indicated. However, this is not as urgently needed as addressing the bronchospasm. DIF: Cognitive Level: Analyze (analysis)

During change-of-shift report, the nurse obtains the following information about a hypertensive patient who received the first dose of nadolol (Corgard) during the previous shift. Which information indicates that the patient needs immediate intervention? a. The patient's most recent blood pressure (BP) reading is 158/91 mm Hg. b. The patient's pulse has dropped from 68 to 57 beats/minute. c. The patient has developed wheezes throughout the lung fields. d. The patient complains that the fingers and toes feel quite cold.

ANS: C The most urgent concern for this patient is the wheezes, which indicate that bronchospasm (a common adverse effect of the noncardioselective b-blockers) is occurring. The nurse should immediately obtain an oxygen saturation measurement, apply supplemental oxygen, and notify the health care provider. The mild decrease in heart rate and complaint of cold fingers and toes are associated with b-receptor blockade but do not require any change in therapy. The BP reading may indicate that a change in medication type or dose may be indicated. However, this is not as urgently needed as addressing the bronchospasm

A patient recovering from heart surgery develops pericarditis and complains of level 6 (0 to 10 scale) chest pain with deep breathing. Which ordered PRN medication will be the most appropriate for the nurse to give? a. Fentanyl 1 mg IV b. IV morphine sulfate 4 mg c. Oral ibuprofen (Motrin) 600 mg d. Oral acetaminophen (Tylenol) 650 mg

ANS: C The pain associated with pericarditis is caused by inflammation, so nonsteroidal antiinflammatory drugs (NSAIDs) (e.g., ibuprofen) are most effective. Opioid analgesics are usually not used for the pain associated with pericarditis.

42. After reviewing information shown in the accompanying figure from the medical records of a 43-year-old, which risk factor modification for coronary artery disease should the nurse include in patient teaching? a. Importance of daily physical activity b. Effect of weight loss on blood pressure c. Dietary changes to improve lipid levels d. Ongoing cardiac risk associated with history of tobacco use

ANS: C The patient has an elevated low-density lipoprotein (LDL) cholesterol and low high-density lipoprotein (HDL) cholesterol, which will increase the risk of coronary artery disease. Although the blood pressure is in the prehypertensive range, the patient's waist circumference and body mass index (BMI) indicate an appropriate body weight. The risk for coronary artery disease a year after quitting smoking is the same as a nonsmoker. The patient's occupation indicates that daily activity is at the levels suggested by national guidelines.

A patient's heart monitor shows sinus rhythm, rate 64. The PR interval is 0.18 seconds at 1:00 AM, 0.22 seconds at 2:30 PM, and 0.28 seconds at 4:00 PM. Which action should the nurse take next? a. Place the transcutaneous pacemaker pads on the patient. b. Give atropine sulfate 1 mg IV per agency dysrhythmia protocol. c. Call the health care provider before giving scheduled metoprolol (Lopressor). d. Document the patient's rhythm and assess the patient's response to the rhythm.

ANS: C The patient has progressive first-degree atrioventricular (AV) block, and the -blocker should be held until discussing the drug with the health care provider. Documentation and assessment are appropriate but not fully adequate responses. The patient with first-degree AV block usually is asymptomatic and a pacemaker is not indicated. Atropine is sometimes used for symptomatic bradycardia, but there is no indication that this patient is symptomatic. DIF: Cognitive Level: Analyze (analysis)

A patient who is complaining of a "racing" heart and feeling "anxious" comes to the emergency department. The nurse places the patient on a heart monitor and obtains the following electrocardiographic (ECG) tracing. Which action should the nurse take next? a. Prepare to perform electrical cardioversion. b. Have the patient perform the Valsalva maneuver. c. Obtain the patient's vital signs including O2 saturation. d. Prepare to give a -blocker medication to slow the heart rate.

ANS: C The patient has sinus tachycardia, which may have multiple etiologies such as pain, dehydration, anxiety, and myocardial ischemia. Further assessment is needed before determining the treatment. Vagal stimulation or -blockade may be used after further assessment of the patient. Electrical cardioversion is used for some tachydysrhythmias but would not be used for sinus tachycardia. DIF: Cognitive Level: Analyze (analysis)

A patient who is complaining of a "racing" heart and feeling "anxious" comes to the emergency department. The nurse places the patient on a heart monitor and obtains the following electrocardiographic (ECG) tracing. Which action should the nurse take next? a. Prepare to perform electrical cardioversion. b. Have the patient perform the Valsalva maneuver. c. Obtain the patient's vital signs including O2 saturation. d. Prepare to give a -blocker medication to slow the heart rate.

ANS: C The patient has sinus tachycardia, which may have multiple etiologies such as pain, dehydration, anxiety, and myocardial ischemia. Further assessment is needed before determining the treatment. Vagal stimulation or -blockade may be used after further assessment of the patient. Electrical cardioversion is used for some tachydysrhythmias but would not be used for sinus tachycardia. DIF: Cognitive Level: Analyze (analysis)

30. Which electrocardiographic (ECG) change is most important for the nurse to report to the health care provider when caring for a patient with chest pain? a. Inverted P wave b. Sinus tachycardia c. ST-segment elevation d. First-degree atrioventricular block

ANS: C The patient is likely to be experiencing an ST-segment-elevation myocardial infarction (STEMI). Immediate therapy with percutaneous coronary intervention (PCI) or thrombolytic medication is indicated to minimize myocardial damage. The other ECG changes may also suggest a need for therapy, but not as rapidly.

Which intervention by a new nurse who is caring for a patient who has just had an implantable cardioverter-defibrillator (ICD) inserted indicates a need for more teaching about the care of patients with ICDs? a. The nurse administers amiodarone (Cordarone) to the patient. b. The nurse helps the patient fill out the application for obtaining a Medic Alert device. c. The nurse encourages the patient to do active range of motion exercises for all extremities. d. The nurse teaches the patient that sexual activity can be resumed when the incision is healed.

ANS: C The patient should avoid moving the arm on the ICD insertion site until healing has occurred to prevent displacement of the ICD leads. The other actions by the new nurse are appropriate for this patient. DIF: Cognitive Level: Apply (application)

18. In preparation for discharge, the nurse teaches a patient with chronic stable angina how to use the prescribed short-acting and long-acting nitrates. Which patient statement indicates that the teaching has been effective? a. "I will check my pulse rate before I take any nitroglycerin tablets." b. "I will put the nitroglycerin patch on as soon as I get any chest pain." c. "I will stop what I am doing and sit down before I put the nitroglycerin under my tongue." d. "I will be sure to remove the nitroglycerin patch before taking any sublingual nitroglycerin."

ANS: C The patient should sit down before taking the nitroglycerin to decrease cardiac workload and prevent orthostatic hypotension. Transdermal nitrates are used prophylactically rather than to treat acute pain and can be used concurrently with sublingual nitroglycerin. Although the nurse should check blood pressure before giving nitroglycerin, patients do not need to check the pulse rate before taking nitrates.

When caring for a patient with mitral valve stenosis, it is most important that the nurse assess for a. diastolic murmur. b. peripheral edema. c. shortness of breath on exertion. d. right upper quadrant tenderness.

ANS: C The pressure gradient changes in mitral stenosis lead to fluid backup into the lungs, resulting in hypoxemia and dyspnea. The other findings also may be associated with mitral valve disease but are not indicators of possible hypoxemia.

37. A patient who has chest pain is admitted to the emergency department (ED) and all of the following are ordered. Which one should the nurse arrange to be completed first? a. Chest x-ray b. Troponin level c. Electrocardiogram (ECG) d. Insertion of a peripheral IV

ANS: C The priority for the patient is to determine whether an acute myocardial infarction (AMI) is occurring so that reperfusion therapy can begin as quickly as possible. ECG changes occur very rapidly after coronary artery occlusion, and an ECG should be obtained as soon as possible. Troponin levels will increase after about 3 hours. Data from the chest x-ray may impact the patient's care but are not helpful in determining whether the patient is experiencing a myocardial infarction (MI). Peripheral access will be needed but not before the ECG.

41. A patient with diabetes mellitus and chronic stable angina has a new order for captopril (Capoten). The nurse should teach the patient that the primary purpose of captopril is to a. lower heart rate. b. control blood glucose levels. c. prevent changes in heart muscle. d. reduce the frequency of chest pain.

ANS: C The purpose for angiotensin-converting enzyme (ACE) inhibitors in patients with chronic stable angina who are at high risk for a cardiac event is to decrease ventricular remodeling. ACE inhibitors do not directly impact angina frequency, blood glucose, or heart rate.

The nurse needs to quickly estimate the heart rate for a patient with a regular heart rhythm. Which method will be best to use? a. Count the number of large squares in the R-R interval and divide by 300. b. Print a 1-minute electrocardiogram (ECG) strip and count the number of QRS complexes. c. Use the 3-second markers to count the number of QRS complexes in 6 seconds and multiply by 10. d. Calculate the number of small squares between one QRS complex and the next and divide into 1500.

ANS: C This is the quickest way to determine the ventricular rate for a patient with a regular rhythm. All the other methods are accurate but take longer. Cognitive Level: Analyze (analysis)

The nurse needs to quickly estimate the heart rate for a patient with a regular heart rhythm. Which method will be best to use? a. Count the number of large squares in the R-R interval and divide by 300. b. Print a 1-minute electrocardiogram (ECG) strip and count the number of QRS complexes. c. Use the 3-second markers to count the number of QRS complexes in 6 seconds and multiply by 10. d. Calculate the number of small squares between one QRS complex and the next and divide into 1500.

ANS: C This is the quickest way to determine the ventricular rate for a patient with a regular rhythm. All the other methods are accurate, but take longer. DIF: Cognitive Level: Analyze (analysis)

14. A patient with ST-segment elevation in three contiguous electrocardiographic (ECG) leads is admitted to the emergency department (ED) and diagnosed as having an ST-segment-elevation myocardial infarction (STEMI). Which question should the nurse ask to determine whether the patient is a candidate for thrombolytic therapy? a. "Do you have any allergies?" b. "Do you take aspirin on a daily basis?" c. "What time did your chest pain begin?" d. "Can you rate your chest pain using a 0 to 10 scale?"

ANS: C Thrombolytic therapy should be started within 6 hours of the onset of the myocardial infarction (MI), so the time at which the chest pain started is a major determinant of the appropriateness of this treatment. The other information will also be needed, but it will not be a factor in the decision about thrombolytic therapy.

A patient with heart failure has a new order of captopril 12.5 mg PO. After administering the first dose and teaching the patient about the drug, which statement by the patient indicates that teaching has been effective? a. I will be sure to take the medication with food. b. I will need to eat more potassium-rich foods in my diet c. I will call for help when I need to go to the bathroom. d. I will expect to feel more short of breath for the next few days

ANS: C Captopril can cause hypotension, especially after the initial dose, so it is important that the patient not get up out of bed without assistance until the nurse has had a chance to evaluate the effect of the first dose. The ACE inhibitors are potassium sparing, and the nurse should not teach the patient to purposely increase sources of dietary potassium. Increased shortness of breath is expected with the initiation of b-adrenergic blocker therapy for heart failure, not for ACE inhibitor therapy. ACE inhibitors are best absorbed when taken a hour before eating.

The nurse plans discharge teaching for a patient with chronic heart failure who has prescriptions for digoxin and hydrochlorothiazide. Appropriate instructions for the patient include a. Limit dietary sources of potassium b. Take the hydrochlorthiazide before bedtime c. Notify the HCP if nausea develops d. Skip the digoxin if the pulse is below 60 beats/minute

ANS: C Nausea is an indication of digoxin toxicity and should be reported so that the provided can assess the patient for toxicity and adjust the digoxin dose, if necessary. The patient will need to include potassium-containing foods in the diet to avoid hypokalemia. Patient should be taught to check their pulse daily before taking the digoxin and if the pulse is less than 60, to call their provider before taking the digoxin. Diureteics should be taken early in the morning to avoid sleep disruption.

Which topic will the nurse plan to include in discharge teaching for a patient with systolic heart failure and an ejection fraction of 33%? a. Need to begin an aerobic exercise program several times weekly b. Use of salt substitutes to replace table salt when cooking and at the table c. Benefits and side effects of ACE inhibitors d. Importance of making an annual appointment with the primary care provider

ANS: C The core measures for the treatment of heart failure established by The Joint Commission indicate that patients with an ejection fraction <40% receive an ACE inhibitor to decrease the progression of heart failure. Aerobic exercise may not be appropriate for a patient with this level of heart failure, salt substitutes are not usually recommended because of the risk of hyperkalemia, and the patient will need to see the primary care provider more frequently than annually.

While admitting an 82-year-old with acute decompensated heart failure to the hospital, the nurse learns that the patient lives alone and sometimes confuses the "water pill" with the "heart pill." When planning for the patient's discharge the nurse will facilitate a a. Consult with a psychologist. b. Transfer to a long-term care facility. c. Referral to a home health care agency. d. Arrangements for around-the-clock care.

ANS: C The data about the patient suggests that assistance in developing a system for taking medications correctly at home is needed. A home health nurse will assess the patient's home situation and help the patient develop a method for taking the two medications as directed. There is no evidence that the patient requires services such as a psychologist consult, long-term care, or around-the-clock home care.

The nurse is caring for a patient who is receiving IV furosemide (Lasix) and morphine for the treatment of acute decompensated heart failure with severe orthopnea. Which clinical finding is the best indicator that the treatment has been effective? a. Weight loss of 2 pounds in 24 hours b. Hourly urine output greater than 60 mL c. Reduction in patient complaints of chest pain d. Reduced dyspnea with the head of bed at 30 degrees

ANS: D Because the patients major clinical manifestation of ADHF is orthopnea (caused by the presence of fluid in the alveoli), the best indicator that the medications are effective is a decrease in dyspnea with the head of the bed at 30 degrees. The other assessment data also may indicate that diuresis or improvement in cardiac output has occurred, but are not as specific to evaluating this patient's response.

A 74-yr-old patient has just arrived in the emergency department. After assessment reveals a pulse deficit of 46 beats, the nurse will anticipate that the patient may require a. emergent cardioversion. b. a cardiac catheterization. c. hourly blood pressure (BP) checks. d. electrocardiographic (ECG) monitoring.

ANS: D Pulse deficit is a difference between simultaneously obtained apical and radial pulses. It indicates that there may be a cardiac dysrhythmia that would best be detected with ECG monitoring. Frequent BP monitoring, cardiac catheterization, and emergent cardioversion are used for diagnosis and/or treatment of cardiovascular disorders but would not be as helpful in determining the immediate reason for the pulse deficit.

The nurse teaches the patient being evaluated for rhythm disturbances with a Holter monitor to a. connect the recorder to a computer once daily. b. exercise more than usual while the monitor is in place. c. remove the electrodes when taking a shower or tub bath. d. keep a diary of daily activities while the monitor is worn.

ANS: D The patient is instructed to keep a diary describing daily activities while Holter monitoring is being accomplished to help correlate any rhythm disturbances with patient activities. Patients are taught that they should not take a shower or bath during Holter monitoring and that they should continue with their usual daily activities. The recorder stores the information about the patient's rhythm until the end of the testing, when it is removed and the data are analyzed.

The nurse is reviewing the 12-lead electrocardiograph (ECG) for a healthy 74-yr- old patient who is having an annual physical examination. What finding is of most concern to the nurse? a. A right bundle-branch block. c. The QRS duration is 0.13 seconds. b. The PR interval is 0.21 seconds. d. The heart rate (HR) is 41 beats/min.

ANS: D The resting HR does not change with aging, so the decrease in HR requires further investigation. Bundle-branch block and slight increases in PR interval or QRS duration are common in older individuals because of increases in conduction time through the AV node, bundle of His, and bundle branches

The nurse notes that a patient who was admitted with heart failure has jugular venous distention (JVD) when lying flat in bed. Which follow-up action should the nurse take next? a. Obtain vital signs, including oxygen saturation. b. Have the patient perform the Valsalva maneuver. c. Document this JVD finding in the patient's record. d. Observe for JVD with the patient elevated 45 degrees.

ANS: D When the patient is lying flat, the jugular veins are at the level of the right atrium, so JVD is a common (but not a clinically significant) finding. Obtaining vital signs and oxygen saturation is not warranted at this point. JVD is an expected finding when a patient performs the Valsalva maneuver because right atrial pressure increases. JVD that persists when the patient is sitting at a 30- to 45-degree angle or greater is significant. The nurse will document the JVD in the medical record if it persists when the head is elevated.

A 56-year-old patient who has no previous history of hypertension or other health problems suddenly develops a blood pressure (BP) of 198/110 mm Hg. After reconfirming the BP, it is appropriate for the nurse to tell the patient that a. a BP recheck should be scheduled in a few weeks. b. dietary sodium and fat content should be decreased. c. there is an immediate danger of a stroke and hospitalization will be required. d. diagnosis of a possible cause, treatment, and ongoing monitoring will be needed.

ANS: D A sudden increase in BP in a patient over age 50 with no previous hypertension history or risk factors indicates that the hypertension may be secondary to some other problem. The BP will need treatment and ongoing monitoring. If the patient has no other risk factors, a stroke in the immediate future is unlikely. There is no indication that dietary salt or fat intake have contributed to this sudden increase in BP, and reducing intake of salt and fat alone will not be adequate to reduce this BP to an acceptable level

The nurse is obtaining a health history from a 24-year-old patient with hypertrophic cardiomyopathy (HC). Which information obtained by the nurse is most important? a. The patient has a history of a recent upper respiratory infection. b. The patient has a family history of coronary artery disease (CAD). c. The patient reports using cocaine a "couple of times" as a teenager. d. The patient's 29-year-old brother died from a sudden cardiac arrest.

ANS: D About half of all cases of HC have a genetic basis, and it is the most common cause of sudden cardiac death in otherwise healthy young people. The information about the patient's brother will be helpful in planning care (such as an automatic implantable cardioverter-defibrillator [AICD

6. Which statement made by a patient with coronary artery disease after the nurse has completed teaching about therapeutic lifestyle changes (TLC) diet indicates that further teaching is needed? a. "I will switch from whole milk to 1% milk." b. "I like salmon and I will plan to eat it more often." c. "I can have a glass of wine with dinner if I want one." d. "I will miss being able to eat peanut butter sandwiches."

ANS: D Although only 30% of the daily calories should come from fats, most of the fat in the TLC diet should come from monosaturated fats such as are found in nuts, olive oil, and canola oil. The patient can include peanut butter sandwiches as part of the TLC diet. The other patient comments indicate a good understanding of the TLC diet.

1. When developing a teaching plan for a 61-year-old man with the following risk factors for coronary artery disease (CAD), the nurse should focus on the a. family history of coronary artery disease. b. increased risk associated with the patient's gender. c. increased risk of cardiovascular disease as people age. d. elevation of the patient's low-density lipoprotein (LDL) level.

ANS: D Because family history, gender, and age are nonmodifiable risk factors, the nurse should focus on the patient's LDL level. Decreases in LDL will help reduce the patient's risk for developing CAD.

11. Nadolol (Corgard) is prescribed for a patient with chronic stable angina and left ventricular dysfunction. To determine whether the drug is effective, the nurse will monitor for a. decreased blood pressure and heart rate. b. fewer complaints of having cold hands and feet. c. improvement in the strength of the distal pulses. d. the ability to do daily activities without chest pain.

ANS: D Because the medication is ordered to improve the patient's angina, effectiveness is indicated if the patient is able to accomplish daily activities without chest pain. Blood pressure and heart rate may decrease, but these data do not indicate that the goal of decreased angina has been met. The noncardioselective β-adrenergic blockers can cause peripheral vasoconstriction, so the nurse would not expect an improvement in distal pulse quality or skin temperature.

When analyzing the rhythm of a patient's electrocardiogram (ECG), the nurse will need to investigate further upon finding a(n) a. isoelectric ST segment. c. QT interval of 0.38 second. b. PR interval of 0.18 second. d. QRS interval of 0.14 second.

ANS: D Because the normal QRS interval is less than 0.12 seconds, the patient's QRS interval of 0.14 seconds indicates that the conduction through the ventricular conduction system is prolonged. The PR interval and QT interval are within normal range and ST segment should be isoelectric (flat). DIF: Cognitive Level: Apply (application)

The nurse is caring for a 70-year-old who uses hydrochlorothiazide (HydroDIURIL) and enalapril (Norvasc), but whose self-monitored blood pressure (BP) continues to be elevated. Which patient information may indicate a need for a change? a. Patient takes a daily multivitamin tablet. b. Patient checks BP daily just after getting up. c. Patient drinks wine three to four times a week. d. Patient uses ibuprofen (Motrin) daily to treat osteoarthritis.

ANS: D Because use of nonsteroidal antiinflammatory drugs (NSAIDs) can prevent adequate BP control, the patient may need to avoid the use of ibuprofen. A multivitamin tablet will help supply vitamin D, which may help lower BP. BP decreases while sleeping, so self-monitoring early in the morning will result in obtaining pressures that are at their lowest. The patient's alcohol intake is not excessive

The nurse is caring for a 70-yr-old patient who uses hydrochlorothiazide and enalapril (Norvasc) but whose self-monitored blood pressure (BP) continues to be elevated. Which patient information may indicate a need for a change? a. Patient takes a daily multivitamin tablet. b. Patient checks BP daily just after getting up. c. Patient drinks wine three to four times a week. d. Patient uses ibuprofen (Motrin) treat osteoarthritis.

ANS: D Because use of nonsteroidal antiinflammatory drugs (NSAIDs) can prevent adequate BP control, the patient may need to avoid the use of ibuprofen. A multivitamin tablet will help supply vitamin D, which may help lower BP. BP decreases while sleeping, so self-monitoring early in the morning will result in obtaining pressures that are at their lowest. The patient's alcohol intake is not excessive.

The nurse is caring for a 70-yr-old patient who uses hydrochlorothiazide and enalapril (Norvasc) but whose self-monitored blood pressure (BP) continues to be elevated. Which patient information may indicate a need for a change? a. Patient takes a daily multivitamin tablet. b. Patient checks BP daily just after getting up. c. Patient drinks wine three to four times a week. d. Patient uses ibuprofen (Motrin) treat osteoarthritis.

ANS: D Because use of nonsteroidal antiinflammatory drugs (NSAIDs) can prevent adequate BP control, the patient may need to avoid the use of ibuprofen. A multivitamin tablet will help supply vitamin D, which may help lower BP. BP decreases while sleeping, so self-monitoring early in the morning will result in obtaining pressures that are at their lowest. The patient's alcohol intake is not excessive. DIF: Cognitive Level: Apply (application)

Which topic should the nurse include in patient teaching for a patient with a venous stasis ulcer on the left lower leg? a. Need to increase carbohydrate intake b. Methods of keeping the wound area dry c. Purpose of prophylactic antibiotic therapy d. Application of elastic compression stockings

ANS: D Compression of the leg is essential to healing of venous stasis ulcers. High dietary intake of protein, rather than carbohydrates, is needed. Prophylactic antibiotics are not routinely used for venous ulcers. Moist dressings are used to hasten wound healing

After the nurse teaches the patient with stage 1 hypertension about diet modifications that should be implemented, which diet choice indicates that the teaching has been effective? a. The patient avoids eating nuts or nut butters. b. The patient restricts intake of chicken and fish. c. The patient has two cups of coffee in the morning. d. The patient has a glass of low-fat milk with each meal.

ANS: D For the prevention of hypertension, the Dietary Approaches to Stop Hypertension (DASH) recommendations include increasing the intake of calcium-rich foods. Caffeine restriction and decreased protein intake are not included in the recommendations. Nuts are high in beneficial nutrients and 4 to 5 servings weekly are recommended in the DASH diet.

Which action will be included in the plan of care when the nurse is caring for a patient who is receiving nicardipine (Cardene) to treat a hypertensive emergency? a. Keep the patient NPO to prevent aspiration caused by nausea and possible vomiting. b. Organize nursing activities so that the patient has undisturbed sleep for 6 to 8 hours at night. c. Assist the patient up in the chair for meals to avoid complications associated with immobility. d. Use an automated noninvasive blood pressure machine to obtain frequent blood pressure (BP) measurements.

ANS: D Frequent monitoring of BP is needed when the patient is receiving rapid-acting IV antihypertensive medications. This can be most easily accomplished with an automated BP machine or arterial line. The patient will require frequent assessments, so allowing 6 to 8 hours of undisturbed sleep is not appropriate. When patients are receiving IV vasodilators, bed rest is maintained to prevent decreased cerebral perfusion and fainting. There is no indication that this patient is nauseated or at risk for aspiration, so an NPO status is unnecessary

Which action will be included in the plan of care when the nurse is caring for a patient who is receiving nicardipine (Cardene) to treat a hypertensive emergency? a. Organize nursing activities so that the patient has undisturbed sleep for 8 hours at night. b. Keep the patient NPO to prevent aspiration caused by nausea and possible vomiting. c. Assist the patient up in the chair for meals to avoid complications associated with immobility. d. Use an automated noninvasive blood pressure machine to obtain frequent measurements.

ANS: D Frequent monitoring of BP is needed when the patient is receiving rapid-acting IV antihypertensive medications. This can be most easily accomplished with an automated BP machine or arterial line. The patient will require frequent assessments, so allowing 8 hours of undisturbed sleep is not reasonable. When patients are receiving IV vasodilators, bed rest is maintained to prevent decreased cerebral perfusion and fainting. There is no indication that this patient is nauseated or at risk for aspiration, so an NPO status is unnecessary.

Which action will be included in the plan of care when the nurse is caring for a patient who is receiving nicardipine (Cardene) to treat a hypertensive emergency? a. Organize nursing activities so that the patient has undisturbed sleep for 8 hours at night. b. Keep the patient NPO to prevent aspiration caused by nausea and possible vomiting. c. Assist the patient up in the chair for meals to avoid complications associated with immobility. d. Use an automated noninvasive blood pressure machine to obtain frequent measurements.

ANS: D Frequent monitoring of BP is needed when the patient is receiving rapid-acting IV antihypertensive medications. This can be most easily accomplished with an automated BP machine or arterial line. The patient will require frequent assessments, so allowing 8 hours of undisturbed sleep is not reasonable. When patients are receiving IV vasodilators, bed rest is maintained to prevent decreased cerebral perfusion and fainting. There is no indication that this patient is nauseated or at risk for aspiration, so an NPO status is unnecessary. DIF: Cognitive Level: Apply (application)

Which information should the nurse include when teaching a patient with newly diagnosed hypertension? a. Increasing physical activity will control blood pressure (BP) for most patients. b. Most patients are able to control BP through dietary changes. c. Annual BP checks are needed to monitor treatment effectiveness. d. Hypertension is usually asymptomatic until target organ damage occurs.

ANS: D Hypertension is usually asymptomatic until target organ damage has occurred. Lifestyle changes (e.g., physical activity, dietary changes) are used to help manage blood pressure, but drugs are needed for most patients. Home BP monitoring should be taught to the patient and findings checked by the health care provider frequently when starting treatment for hypertension and then every 3 months once stable

Which laboratory result for a patient with multifocal premature ventricular contractions (PVCs) is most important for the nurse to communicate to the health care provider? a. Blood glucose of 243 mg/dL c. Serum sodium of 134 mEq/L b. Serum chloride of 92 mEq/L d. Serum potassium of 2.9 mEq/L

ANS: D Hypokalemia increases the risk for ventricular dysrhythmias such as PVCs, ventricular tachycardia, and ventricular fibrillation. The health care provider will need to prescribe a potassium infusion to correct this abnormality. Although the other laboratory values are also abnormal, they are not likely to be the etiology of the patient's PVCs and do not require immediate correction. DIF: Cognitive Level: Analyze (analysis)

36. A patient had a non-ST-segment-elevation myocardial infarction (NSTEMI) 3 days ago. Which nursing intervention included in the plan of care is most appropriate for the registered nurse (RN) to delegate to an experienced licensed practical/vocational nurse (LPN/LVN)? a. Evaluation of the patient's response to walking in the hallway b. Completion of the referral form for a home health nurse follow-up c. Education of the patient about the pathophysiology of heart disease d. Reinforcement of teaching about the purpose of prescribed medications

ANS: D LPN/LVN education and scope of practice include reinforcing education that has previously been done by the RN. Evaluating the patient response to exercise after a NSTEMI requires more education and should be done by the RN. Teaching and discharge planning/ documentation are higher level skills that require RN education and scope of practice.

The registered nurse (RN) is caring for a patient with a hypertensive crisis who is receiving sodium nitroprusside . Which nursing action can the nurse delegate to an experienced licensed practical/vocational nurse (LPN/LVN)? a. Evaluate effectiveness of nitroprusside therapy on blood pressure (BP). b. Assess the patient's environment for adverse stimuli that might increase BP. c. Titrate nitroprusside to decrease mean arterial pressure (MAP) to 115 mm Hg. d. Set up the automatic noninvasive BP machine to take readings every 15 minutes.

ANS: D LPN/LVN education and scope of practice include the correct use of common equipment such as automatic noninvasive blood pressure machines. The other actions require advanced nursing judgment and education, and should be done by RNs. DIF: Cognitive Level: Apply (application)

Which action should the nurse take when giving the initial dose of oral labetalol to a patient with hypertension? a. Encourage the use of hard candy to prevent dry mouth. b. Teach the patient that headaches often occur with this drug. c. Instruct the patient to call for help if heart palpitations occur. d. Ask the patient to request assistance before getting out of bed.

ANS: D Labetalol decreases sympathetic nervous system activity by blocking both - and -adrenergic receptors, leading to vasodilation and a decrease in heart rate, which can cause severe orthostatic hypotension. Heart palpitations, dry mouth, dehydration, and headaches are possible side effects of other antihypertensives.

Which action should the nurse take when giving the initial dose of oral labetalol to a patient with hypertension? a. Encourage the use of hard candy to prevent dry mouth. b. Teach the patient that headaches often occur with this drug. c. Instruct the patient to call for help if heart palpitations occur. d. Ask the patient to request assistance before getting out of bed.

ANS: D Labetalol decreases sympathetic nervous system activity by blocking both - and -adrenergic receptors, leading to vasodilation and a decrease in heart rate, which can cause severe orthostatic hypotension. Heart palpitations, dry mouth, dehydration, and headaches are possible side effects of other antihypertensives. DIF: Cognitive Level: Apply (application)

The nurse establishes the nursing diagnosis of ineffective health maintenance related to lack of knowledge regarding long-term management of rheumatic fever when a 30-year-old recovering from rheumatic fever without carditis says which of the following? a. "I will need prophylactic antibiotic therapy for 5 years." b. "I will need to take aspirin or ibuprofen (Motrin) to relieve my joint pain." c. "I will call the doctor if I develop excessive fatigue or difficulty breathing." d. "I will be immune to further episodes of rheumatic fever after this infection."

ANS: D Patients with a history of rheumatic fever are more susceptible to a second episode. Patients with rheumatic fever without carditis require prophylaxis until age 20 and for a minimum of 5 years. The other patient statements are correct and would not support the nursing diagnosis of ineffective health maintenance.

43. After reviewing a patient's history, vital signs, physical assessment, and laboratory data, which information shown in the accompanying figure is most important for the nurse to communicate to the health care provider? a. Q waves on ECG b. Elevated troponin levels c. Fever and hyperglycemia d. Tachypnea and crackles in lungs

ANS: D Pulmonary congestion and tachypnea suggest that the patient may be developing heart failure, a complication of myocardial infarction (MI). Mild fever and hyperglycemia are common after MI because of the inflammatory process that occurs with tissue necrosis. Troponin levels will be elevated for several days after MI. Q waves often develop with ST-segment-elevation MI.

The nurse has identified a nursing diagnosis of acute pain related to inflammatory process for a patient with acute pericarditis. The priority intervention by the nurse for this problem is to a. teach the patient to take deep, slow breaths to control the pain. b. force fluids to 3000 mL/day to decrease fever and inflammation. c. remind the patient to request opioid pain medication every 4 hours. d. place the patient in Fowler's position, leaning forward on the overbed table.

ANS: D Sitting upright and leaning forward frequently will decrease the pain associated with pericarditis. Forcing fluids will not decrease the inflammation or pain. Taking deep breaths will tend to increase pericardial pain. Opioids are not very effective at controlling pain caused by acute inflammatory conditions and are usually ordered PRN. The patient would receive scheduled doses of a nonsteroidal antiinflammatory drug (NSAID).

The nurse is caring for a patient with mitral regurgitation. Referring to the figure below, where should the nurse listen to best hear any murmur that the patient has? a. 1 b. 2 c. 3 d. 4

ANS: D Sounds from the mitral valve are best heard at the apex of the heart, fifth intercostal space, midclavicular line.

To determine whether there is a delay in impulse conduction through the ventricles, the nurse will measure the duration of the patient's a. P wave. c. PR interval. b. Q wave. d. QRS complex.

ANS: D The QRS complex represents ventricular depolarization. The P wave represents the depolarization of the atria. The PR interval represents depolarization of the atria, atrioventricular node, a bundle of His, bundle branches, and the Purkinje fibers. The Q wave is the first negative deflection following the P wave and should be narrow and short. Cognitive Level: Understand (comprehension)Chapter

The nurse obtains a rhythm strip on a patient who has had a myocardial infarction and makes the following analysis: no visible P waves, PR interval not measurable, ventricular rate of 162, R-R interval regular, and QRS complex wide and distorted, and QRS duration of 0.18 second. The nurse interprets the patient's cardiac rhythm as a. atrial flutter. c. ventricular fibrillation. b. sinus tachycardia. d. ventricular tachycardia.

ANS: D The absence of P waves, wide QRS, rate greater than 150 beats/min, and the regularity of the rhythm indicate ventricular tachycardia. Atrial flutter is usually regular, has a narrow QRS configuration, and has flutter waves present representing atrial activity. Sinus tachycardia has P waves. Ventricular fibrillation is irregular and does not have a consistent QRS duration. DIF: Cognitive Level: Apply (application)

A patient has just been diagnosed with hypertension and has been started on captopril (Capoten). Which information is important to include when teaching the patient about this medication? a. Check blood pressure (BP) in both arms before taking the medication. b. Increase fluid intake if dryness of the mouth is a problem. c. Include high-potassium foods such as bananas in the diet. d. Change position slowly to help prevent dizziness and falls.

ANS: D The angiotensin-converting enzyme (ACE) inhibitors frequently cause orthostatic hypotension, and patients should be taught to change position slowly to allow the vascular system time to compensate for the position change. Increasing fluid intake may counteract the effect of the medication, and the patient is taught to use gum or hard candy to relieve dry mouth. The BP should be taken in the nondominant arm by newly diagnosed patients in the morning, before taking the medication, and in the evening. Because ACE inhibitors cause potassium retention, increased intake of high-potassium foods is inappropriate

24. A patient with hyperlipidemia has a new order for colesevelam (Welchol). Which nursing action is most appropriate when giving the medication? a. Have the patient take this medication with an aspirin. b. Administer the medication at the patient's usual bedtime. c. Have the patient take the colesevelam with a sip of water. d. Give the patient's other medications 2 hours after the colesevelam.

ANS: D The bile acid sequestrants interfere with the absorption of many other drugs, and giving other medications at the same time should be avoided. Taking an aspirin concurrently with the colesevelam may increase the incidence of gastrointestinal side effects such as heartburn. An increased fluid intake is encouraged for patients taking the bile acid sequestrants to reduce the risk for constipation. For maximum effect, colesevelam should be administered with meals.

The nurse has received change-of-shift report about the following patients on the progressive care unit. Which patient should the nurse see first? a. A patient with atrial fibrillation, rate 88 and irregular, who has a dose of warfarin (Coumadin) due b. A patient with second-degree atrioventricular (AV) block, type 1, rate 60, who is dizzy when ambulating c. A patient who is in a sinus rhythm, rate 98 and regular, recovering from an elective cardioversion 2 hours ago d. A patient whose implantable cardioverter-defibrillator (ICD) fired twice today and has a dose of amiodarone (Cordarone) due

ANS: D The frequent firing of the ICD indicates that the patient's ventricles are very irritable and the priority is to assess the patient and give the amiodarone. The other patients can be seen after the amiodarone is given. DIF: Cognitive Level: Analyze (analysis)

A patient at the clinic says, "I always walk after dinner, but lately my leg cramps and hurts after just a few minutes of starting. The pain goes away after I stop walking, though." The nurse should a. look for the presence of tortuous veins bilaterally on the legs. b. ask about any skin color changes that occur in response to cold. c. assess for unilateral swelling, redness, and tenderness of either leg. d. palpate for the presence of dorsalis pedis and posterior tibial pulses.

ANS: D The nurse should assess for other clinical manifestations of peripheral arterial disease in a patient who describes intermittent claudication. Changes in skin color that occur in response to cold are consistent with Raynaud's phenomenon. Tortuous veins on the legs suggest venous insufficiency. Unilateral leg swelling, redness, and tenderness indicate venous thromboembolism.

A patient develops sinus bradycardia at a rate of 32 beats/min, has a blood pressure (BP) of 80/42 mm Hg, and is complaining of feeling faint. Which action should the nurse take next? a. Recheck the heart rhythm and BP in 5 minutes. b. Have the patient perform the Valsalva maneuver. c. Give the scheduled dose of diltiazem (Cardizem). d. Apply the transcutaneous pacemaker (TCP) pads.

ANS: D The patient is experiencing symptomatic bradycardia and treatment with TCP is appropriate. Continued monitoring of the rhythm and BP is an inadequate response. Calcium channel blockers will further decrease the heart rate and the diltiazem should be held. The Valsalva maneuver will further decrease the rate. DIF: Cognitive Level: Apply (application)

The nurse knows that discharge teaching about the management of a new permanent pacemaker has been most effective when the patient states a. "It will be several weeks before I can return to my usual activities." b. "I will avoid cooking with a microwave oven or being near one in use." c. "I will notify the airlines when I make a reservation that I have a pacemaker." d. "I won't lift the arm on the pacemaker side until I see the health care provider."

ANS: D The patient is instructed to avoid lifting the arm on the pacemaker side above the shoulder to avoid displacing the pacemaker leads. The patient should notify airport security about the presence of a pacemaker before going through the metal detector, but there is no need to notify the airlines when making a reservation. Microwave oven use does not affect the pacemaker. The insertion procedure involves minor surgery that will have a short recovery period. DIF: Cognitive Level: Apply (application)

A patient admitted with acute dyspnea is newly diagnosed with dilated cardiomyopathy. Which information will the nurse plan to teach the patient about managing this disorder? a. A heart transplant should be scheduled as soon as possible. b. Elevating the legs above the heart will help relieve dyspnea. c. Careful compliance with diet and medications will prevent heart failure. d. Notify the doctor about any symptoms of heart failure such as shortness of breath.

ANS: D The patient should be instructed to notify the health care provider about any worsening of heart failure symptoms. Because dilated cardiomyopathy does not respond well to therapy, even patients with good compliance with therapy may have recurrent episodes of heart failure. Elevation of the legs above the heart will worsen symptoms (although this approach is appropriate for a patient with hypertrophic cardiomyopathy). The patient with terminal or end-stage cardiomyopathy may consider heart transplantation.

31. When caring for a patient with acute coronary syndrome who has returned to the coronary care unit after having angioplasty with stent placement, the nurse obtains the following assessment data. Which data indicate the need for immediate action by the nurse? a. Heart rate 102 beats/min b. Pedal pulses 1+ bilaterally c. Blood pressure 103/54 mm Hg d. Chest pain level 7 on a 0 to 10 point scale

ANS: D The patient's chest pain indicates that restenosis of the coronary artery may be occurring and requires immediate actions, such as administration of oxygen and nitroglycerin, by the nurse. The other information indicates a need for ongoing assessments by the nurse.

An older patient with chronic atrial fibrillation develops sudden severe pain, pulselessness, pallor, and coolness in the right leg. The nurse should notify the health care provider and immediately a. apply a compression stocking to the leg. b. elevate the leg above the level of the heart. c. assist the patient in gently exercising the leg. d. keep the patient in bed in the supine position.

ANS: D The patient's history and clinical manifestations are consistent with acute arterial occlusion, and resting the leg will decrease the O2 demand of the tissues and minimize ischemic damage until circulation can be restored. Elevating the leg or applying an elastic wrap will further compromise blood flow to the leg. Exercise will increase oxygen demand for the tissues of the

The health care provider has prescribed bed rest with the feet elevated for a patient admitted to the hospital with venous thromboembolism. Which action by the nurse to elevate the patient's feet is best? a. The patient is placed in the Trendelenburg position. b. Two pillows are positioned under the affected leg. c. The bed is elevated at the knee and pillows are placed under the feet. d. One pillow is placed under the thighs and two pillows are placed under the lower legs.

ANS: D The purpose of elevating the feet is to enhance venous flow from the feet to the right atrium, which is best accomplished by placing two pillows under the feet and one under the thighs. Placing the patient in the Trendelenburg position will lower the head below heart level, which is not indicated for this patient. Placing pillows under the calf or elevating the bed at the knee may cause blood stasis at the calf level.

38. After receiving change-of-shift report about the following four patients, which patient should the nurse assess first? a. 39-year-old with pericarditis who is complaining of sharp, stabbing chest pain b. 56-year-old with variant angina who is to receive a dose of nifedipine (Procardia) c. 65-year-old who had a myocardial infarction (MI) 4 days ago and is anxious about the planned discharge d. 59-year-old with unstable angina who has just returned to the unit after having a percutaneous coronary intervention (PCI)

ANS: D This patient is at risk for bleeding from the arterial access site for the PCI, so the nurse should assess the patient's blood pressure, pulse, and the access site immediately. The other patients should also be assessed as quickly as possible, but assessment of this patient has the highest priority.

8. A patient who has had chest pain for several hours is admitted with a diagnosis of rule out acute myocardial infarction (AMI). Which laboratory test should the nurse monitor to help determine whether the patient has had an AMI? a. Myoglobin b. Homocysteine c. C-reactive protein d. Cardiac-specific troponin

ANS: D Troponin levels increase about 4 to 6 hours after the onset of myocardial infarction (MI) and are highly specific indicators for MI. Myoglobin is released within 2 hours of MI, but it lacks specificity and its use is limited. The other laboratory data are useful in determining the patient's risk for developing coronary artery disease (CAD) but are not helpful in determining whether an acute MI is in progress.

While caring for a 23-year-old patient with mitral valve prolapse (MVP) without valvular regurgitation, the nurse determines that discharge teaching has been effective when the patient states that it will be necessary to a. take antibiotics before any dental appointments. b. limit physical activity to avoid stressing the heart. c. take an aspirin a day to prevent clots from forming on the valve. d. avoid use of over-the-counter (OTC) medications that contain stimulant drugs.

ANS: D Use of stimulant medications should be avoided by patients with MVP because these may exacerbate symptoms. Daily aspirin and restricted physical activity are not needed by patients with mild MVP. Antibiotic prophylaxis is needed for patients with MVP with regurgitation but will not be necessary for this patient.

A patient who has just been admitted with pulmonary edema is schedules to receive the following medications. Which medicatino should the nurse question before giving? a. Furosemide 60 mg b. Captopril 25 mg c. Digoxin 0.125 mg d. Carvedilol 3.125 mg

ANS: D Although carvedilol is appropriate for the treatment of chronic heart failure, it is not used for patients with ADHF because of the risk of worsening the heart failure. The other medications are appropriate for the patient with ADHF.

When teaching the patient with newly diagnosed heart failure about a 2000-mg sodium diet, the nurse explains that foods to be restricted include a. Canned and frozen fruits b. Fresh or frozen vegetables c. Eggs and other high-protein foods d. Milk, yogurt, and other milk products

ANS: D Milk and yogurt naturally contain a significant amount of sodium, and intake of these should be limited for patients on a diet that limits sodium to 2000 mg daily. Other milk products, such as processed cheeses, have very high levels of sodium and are not appropriate for a 2000 mg sodium diet. The other foods listed have minimal levels of sodium and can be eaten without restriction.

When teaching a patient about dietary management of stage 1 hypertension, which instruction is most appropriate? A Restrict all caffeine. B Restrict sodium intake. C Increase protein intake. D Use calcium supplements.

B Restrict sodium intake. The patient should decrease intake of sodium. This will help to control hypertension, which can be aggravated by excessive salt intake, which in turn leads to fluid retention. Caffeine and protein intake do not affect hypertension. Calcium supplements are not recommended to lower BP.

When assessing the patient for orthostatic hypotension, after taking the blood pressure (BP) and pulse (P) in the supine position, what should the nurse do next? A Repeat BP and P in this position. B Take BP and P with patient sitting. C Record the BP and P measurements. D Take BP and P with patient standing.

B Take BP and P with patient sitting. When assessing for orthostatic changes in BP after measuring BP in the supine position, the patient is placed in a sitting position and BP is measured within 1 to 2 minutes and then repositioned to the standing position with BP measured again, within 1 to 2 minutes. The results are then recorded with a decrease of 20 mm Hg or more in SBP, a decrease of 10 mm Hg or more in DBP, and/or an increase in pulse of greater than or equal to 20 beats/minute from supine to standing indicating orthostatic hypotension.

The nurse supervises an unlicensed assistant personnel (UAP) who is taking the blood pressure of 58-year-old female patient admitted with heart failure. The patient is obese. The nurse should intervene if what is observed? A The UAP waits 2 minutes after position changes to take orthostatic pressures. B The UAP deflates the blood pressure cuff at a rate of 8 to 10 mm Hg per second. C The UAP takes the blood pressure with the patient's arm at the level of the heart. D The UAP takes a forearm blood pressure because the largest cuff will not fit the patient's upper arm.

B The UAP deflates the blood pressure cuff at a rate of 8 to 10 mm Hg per second. The cuff should be deflated at a rate of 2 to 3 mm Hg per second. The arm should be supported at the level of the heart for accurate blood pressure measurements. If the maximum size blood pressure cuff does not fit the upper arm, the forearm may be used. Orthostatic blood pressures should be taken within 1 to 2 minutes of repositioning the patient.

A patient with varicose veins has been prescribed compression stockings. Which nursing instruction would be most appropriate? A) "Try to keep your stockings on 24 hours a day, as much as possible." B) "While you're still lying in bed in the morning, put on your stockings." C) "Dangle your feet at your bedside for 5 minutes before putting on your stockings." D) "Your stockings will be most effective if you can remove them for a few minutes several times a day."

B) "While you're still lying in bed in the morning, put on your stockings."

A nurse is caring for a patient with a diagnosis of deep venous thrombosis (DVT). The patient has an order to receive 30 mg enoxaparin (Lovenox). Which injection site should the nurse use to administer this medication safely? A) Buttock, upper outer quadrant B) Abdomen, anterior-lateral aspect C) Back of the arm, 2 inches away from a mole D) Anterolateral thigh, with no scar tissue nearby

B) Abdomen, anterior-lateral aspect

The nurse is reviewing the laboratory test results for a 68-yr-old patient whose warfarin (Coumadin) therapy was terminated during the preoperative period. On postoperative day 2, the international normalized ratio (INR) result is 2.7. Which action by the nurse is most appropriate? A) Hold the daily dose of warfarin. B) Administer the daily dose of warfarin. C) Teach the patient signs and symptoms of bleeding. D) Call the physician to request an increased dose of warfarin.

B) Administer the daily dose of warfarin.

A patient with critical limb ischemia had peripheral artery bypass surgery to improve circulation. What nursing care should be provided on postoperative day 1? A) Keep patient on bed rest. B) Assist patient to walk several times. C) Have patient sit in the chair several times. D) Place patient on their side with knees flexed.

B) Assist patient to walk several times.

Which assessment findings of the left lower extremity will the nurse identify as consistent with arterial occlusion (select all that apply.)? Select all that apply. A) Edematous B) Cold and mottled C) Complaints of paresthesia D) Pulse not palpable with Doppler E) Capillary refill less than three seconds F) Erythema and warmer than right lower extremity

B) Cold and mottled C) Complaints of paresthesia D) Pulse not palpable with Doppler

A 73-yr-old man with dementia has a venous ulcer related to chronic venous insufficiency. The nurse should provide teaching on which type of diet for this patient and his caregiver? A) Low-fat diet B) High-protein diet C) Calorie-restricted diet D) High-carbohydrate diet

B) High-protein diet

The nurse is caring for a patient with a recent history of deep vein thrombosis (DVT) who is scheduled for an emergency appendectomy. Vitamin K is ordered for immediate administration. The international normalized ratio (INR) value is 1.0. Which nursing action is most appropriate? A) Administer the medication as ordered. B) Hold the medication and record in the electronic medical record. C) Hold the medication until the lab result is repeated to verify results. D) Administer the medication and seek an increased dose from the health care provider.

B) Hold the medication and record in the electronic medical record.

The nurse is caring for a newly admitted patient with vascular insufficiency. The patient has a new order for enoxaparin (Lovenox) 30 mg subcutaneously. What should the nurse do to correctly administer this medication? A) Spread the skin before inserting the needle. B) Leave the air bubble in the prefilled syringe. C) Use the back of the arm as the preferred site. D) Sit the patient at a 30-degree angle before administration.

B) Leave the air bubble in the prefilled syringe.

Which assessment finding would alert the nurse that a postoperative patient is not receiving the beneficial effects of enoxaparin (Lovenox)? A) Crackles bilaterally in the lung bases B) Pain and swelling in a lower extremity C) Absence of arterial pulse in a lower extremity D) Abdominal pain with decreased bowel sounds

B) Pain and swelling in a lower extremity

A patient was just diagnosed with acute arterial ischemia in the left leg secondary to atrial fibrillation. Which early clinical manifestation must be reported to the physician immediately to save the patient's limb? A) Paralysis B) Paresthesia C) Cramping D) Referred pain

B) Paresthesia

A 67-yr-old man with peripheral artery disease is seen in the primary care clinic. Which symptom reported by the patient would indicate to the nurse that the patient is experiencing intermittent claudication? A) Patient complains of chest pain with strenuous activity. B) Patient says muscle leg pain occurs with continued exercise. C) Patient has numbness and tingling of all his toes and both feet. D) Patient states the feet become red if he puts them in a dependent position.

B) Patient says muscle leg pain occurs with continued exercise.

A 62-yr-old Hispanic male patient with diabetes mellitus has been diagnosed with peripheral artery disease (PAD). The patient is a smoker with a history of gout. To prevent complications, which factor is priority in patient teaching? A) Gender B) Smoking C) Ethnicity D) Comorbidities

B) Smoking

The patient had aortic aneurysm repair 6 hours ago. What priority nursing action will maintain graft patency? A) Assess output for renal dysfunction. B) Use IV fluids to maintain adequate BP. C) Use oral antihypertensives to maintain cardiac output. D) Maintain a low BP to prevent pressure on surgical site.

B) Use IV fluids to maintain adequate BP.

The nurse provides discharge instructions for a 40-year-old woman who is newly diagnosed with cardiomyopathy. Which statement, if made by the patient, indicates that further teaching is necessary? A."I will avoid lifting heavy objects." B. "I can drink alcohol in moderation." C."My family will need to take a CPR course." D. "I will reduce stress by learning guided imagery."

B. "I can drink alcohol in moderation." Patients with cardiomyopathy should avoid alcohol consumption, especially in patients with alcohol-related dilated cardiomyopathy. Avoiding heavy lifting and stress, as well as family members learning CPR, are recommended teaching points.

The nurse performs discharge teaching for a 68-year-old man who is newly diagnosed with infective endocarditis with a history of IV substance abuse. Which statement by the patient indicates to the nurse that teaching was successful? A."I will need antibiotics before having any invasive procedure or surgery." B. "I will inform my dentist about my hospitalization for infective endocarditis." C."I should not be alarmed if I have difficulty breathing or pink-tinged sputum." D. "An elevated temperature is expected and can be managed by taking acetaminophen."

B. "I will inform my dentist about my hospitalization for infective endocarditis." Patients with infective endocarditis should inform their dental providers of their health history. Antibiotic prophylaxis is recommended for patients with a history of infective endocarditis who have certain dental procedures performed. Antibiotics are not indicated before genitourinary or gastrointestinal procedures unless an infection is present. Patients should immediately report the presence of fever or clinical manifestations indicating heart failure to their health care provider.

The patient had a history of rheumatic fever and has been diagnosed with mitral valve stenosis. The patient is planning to have a biologic valve replacement. What protective mechanisms should the nurse teach the patient about using after the valve replacement? A. Long-term anticoagulation therapy B. Antibiotic prophylaxis for dental care C. Exercise plan to increase cardiac tolerance D. Take β-adrenergic blockers to control palpitations.

B. Antibiotic prophylaxis for dental care The patient will need to use antibiotic prophylaxis for dental care to prevent endocarditis. Long-term anticoagulation therapy is not used with biologic valve replacement unless the patient has atrial fibrillation. An exercise plan to increase cardiac tolerance is needed for a patient with heart failure. Taking β-adrenergic blockers to control palpitations is prescribed for mitral valve prolapse, not valve replacement.

The nurse conducts a complete physical assessment on a patient admitted with infective endocarditis. Which finding is significant? A. Respiratory rate of 18 and heart rate of 90 B. Regurgitant murmur at the mitral valve area C. Heart rate of 94 and capillary refill time of 2 seconds D. Point of maximal impulse palpable in fourth intercostal space

B. Regurgitant murmur at the mitral valve area A regurgitant murmur of the aortic or mitral valves would indicate valvular disease, which is a complication of endocarditis. All the other findings are within normal limits.

A patient with hypertension who has just started taking atenolol (Tenormin) returns to the health clinic after 2 weeks for a follow-up visit. The blood pressure (BP) is unchanged from the previous visit. Which action should the nurse take first? a. Tell the patient why a change in drug dosage is needed. b. Ask the patient if the medication is being taken as prescribed. c. Inform the patient that multiple drugs are often needed to treat hypertension. d. Question the patient regarding any lifestyle changes made to help control BP. ANS: B

Because nonadherence with antihypertensive therapy is common, the nurse's initial action should be to determine whether the patient is taking the atenolol as prescribed. The other actions also may be implemented, but these would be done after assessing patient adherence with the prescribed therapy. DIF: Cognitive Level: Analyze (analysis)

The nurse teaches a 28-year-old man newly diagnosed with hypertension about lifestyle modifications to reduce his blood pressure. Which statement by the patient requires an intervention by the nurse? A "I will avoid adding salt to my food during or after cooking." B "If I lose weight, I might not need to continue taking medications." C "I can lower my blood pressure by switching to smokeless tobacco." D "Diet changes can be as effective as taking blood pressure medications."

C "I can lower my blood pressure by switching to smokeless tobacco." Nicotine contained in tobacco products (smoking and chew) cause vasoconstriction and increase blood pressure. Persons with hypertension should restrict sodium to 1500 mg/day by avoiding foods high in sodium and not adding salt in preparation of food or at meals. Weight loss can decrease blood pressure between 5 to 20 mm Hg. Following dietary recommendations (such as the DASH diet) lowers blood pressure, and these decreases compare with those achieved with blood pressure-lowering medication.

The nurse is caring for a patient admitted with chronic obstructive pulmonary disease (COPD), angina, and hypertension. Before administering the prescribed daily dose of atenolol 100 mg PO, the nurse assesses the patient carefully. Which adverse effect is this patient at risk for, given the patient's health history? A Hypocapnia B Tachycardia C Bronchospasm D Nausea and vomiting

C Bronchospasm Atenolol is a cardioselective β1-adrenergic blocker that reduces blood pressure and could affect the β2-receptors in the lungs with larger doses or with drug accumulation. Although the risk of bronchospasm is less with cardioselective β-blockers than nonselective β-blockers, atenolol should be used cautiously in patients with COPD.

The patient has chronic hypertension. Today she has gone to the ED, and her blood pressure has risen to 200/140. What is the priority assessment for the nurse to make? A Is the patient pregnant? B Does the patient need to urinate? C Does the patient have a headache or confusion? D Is the patient taking antiseizure medications as prescribed?

C Does the patient have a headache or confusion? The nurse's priority assessments include neurologic deficits, retinal damage, heart failure, pulmonary edema, and renal failure. The headache or confusion could be seen with hypertensive encephalopathy from increased cerebral capillary permeability leading to cerebral edema. Pregnancy can lead to secondary hypertension. Needing to urinate and taking antiseizure medication do not support a hypertensive emergency.

When teaching how lisinopril (Zestril) will help lower the patient's blood pressure, which mechanism of action should the nurse use to explain it? A Blocks β-adrenergic effects. B Relaxes arterial and venous smooth muscle. C Inhibits conversion of angiotensin I to angiotensin II. D Reduces sympathetic outflow from central nervous system.

C Inhibits conversion of angiotensin I to angiotensin II. Lisinopril is an angiotensin-converting enzyme (ACE) inhibitor that inhibits the conversion of angiotensin I to angiotensin II, which reduces angiotensin II-mediated vasoconstriction and sodium and water retention. Beta blockers result in vasodilation and decreased heart rate. Direct vasodilators relax arterial and venous smooth muscle. Central acting α-adrenergic antagonists reduce sympathetic outflow from the CNS to produce vasodilation and decreased SVR and BP.

The nurse is teaching a women's group about prevention of hypertension. What information should be included in the teaching for all the women (select all that apply)? A Lose weight. B Limit nuts and seeds. C Limit sodium and fat intake. D Increase fruits and vegetables. E Exercise 30 minutes most days.

C Limit sodium and fat intake. D Increase fruits and vegetables. E Exercise 30 minutes most days. Primary prevention of hypertension is to make lifestyle modifications that prevent or delay the increase in BP. Along with exercise for 30 minutes on most days, the DASH eating plan is a healthy way to lower BP by limiting sodium and fat intake, increasing fruits and vegetables, and increasing nutrients that are associated with lowering BP. Nuts and seeds and dried beans are used for protein intake. Weight loss may or may not be necessary for the individual.

A 67-year-old woman with a history of coronary artery disease and prior myocardial infarction is admitted to the emergency department with a blood pressure of 234/148 mm Hg and started on IV nitroprusside (Nitropress). What should the nurse determine as an appropriate goal for the first hour of treatment? A Mean arterial pressure lower than 70 mm Hg B Mean arterial pressure no more than 120 mm Hg C Mean arterial pressure no lower than 133 mm Hg D Mean arterial pressure between 70 and 110 mm Hg

C Mean arterial pressure no lower than 133 mm Hg The initial treatment goal is to decrease mean arterial pressure by no more than 25% within minutes to 1 hour. If the patient is stable, the goal for BP is 160/100 to 110 mm Hg over the next 2 to 6 hours. Lowering the blood pressure too much may decrease cerebral, coronary, or renal perfusion and could precipitate a stroke, myocardial infarction, or renal failure. Additional gradual reductions toward a normal blood pressure should be implemented over the next 24 to 48 hours if the patient is clinically stable.

In caring for a patient admitted with poorly controlled hypertension, which laboratory test result should the nurse understand as indicating the presence of target organ damage? A BUN of 15 mg/dL B Serum uric acid of 3.8 mg/dL C Serum creatinine of 2.6 mg/dL D Serum potassium of 3.5 mEq/L

C Serum creatinine of 2.6 mg/dL The normal serum creatinine level is 0.6-1.3 mg/dL. This elevated level indicates target organ damage to the kidneys. The other lab results are within normal limits.

A postoperative patient asks the nurse why the physician ordered daily administration of enoxaparin (Lovenox). Which reply by the nurse is most appropriate? A) "This medication will help prevent breathing problems after surgery, such as pneumonia." B) "This medication will help lower your blood pressure to a safer level, which is very important after surgery." C) "This medication will help prevent blood clots from forming in your legs until your level of activity, such as walking, returns to normal." D) "This medication is a narcotic pain medication that will help take away any muscle aches caused by positioning on the operating room table."

C) "This medication will help prevent blood clots from forming in your legs until your level of activity, such as walking, returns to normal."

The nurse is caring for a patient who has been receiving warfarin (Coumadin) and digoxin (Lanoxin) as treatment for atrial fibrillation. Because the warfarin has been discontinued before surgery, the nurse should diligently assess the patient for which complication early in the postoperative period until the medication is resumed? A) Decreased cardiac output B) Increased blood pressure C) Cerebral or pulmonary emboli D) Excessive bleeding from incision or IV sites

C) Cerebral or pulmonary emboli

The nurse is caring for a preoperative patient who has an order for vitamin K by subcutaneous injection. The nurse should verify that which laboratory study is abnormal before administering the dose? A) Hematocrit (Hct) B) Hemoglobin (Hgb) C) Prothrombin time (PT) D) Partial thromboplastin time (PTT)

C) Prothrombin time (PT)

A 55-year-old female patient develops acute pericarditis after a myocardial infarction. It is most important for the nurse to assess for which clinical manifestation of a possible complication? A. Presence of a pericardial friction rub B. Distant and muffled apical heart sounds C. Increased chest pain with deep breathing D. Decreased blood pressure with tachycardia

D. Decreased blood pressure with tachycardia Cardiac tamponade is a serious complication of acute pericarditis. Signs and symptoms indicating cardiac tamponade include narrowed pulse pressure, tachypnea, tachycardia, a decreased cardiac output, and decreased blood pressure. The other symptoms are consistent with acute pericarditis.

Right sided Heart failure results in peripheral edema? TRUE OR FALSE

TRUE **Right side Heart failure results in inadequate right ventricle output and systemic congestion (peripheral edema)

A patient is admitted to the hospital in hypertensive emergency (BP 244/142 mmHg). Sodium nitroprusside is started to treat the elevated BP. Which management strategy(ies) would be appropriate for this patient (select all that apply)? a. Measuring hourly urine output b. Decreasing the MAP by 50% within the first hour c. Continuous BP monitoring with an intraarterial line d. Maintaining bed rest and providing tranquilizers to lower the BP e. Assessing the patient for signs and symptoms of heart failure and changes in mental status

a, c, e

The nurse hears a murmur between the S1 and S2 heart sounds at the patient's left fifth intercostal space and midclavicular line. How will the nurse record this information? a. Systolic murmur heard at mitral area b. Systolic murmur heard at Erb's point c. Diastolic murmur heard at aortic area d. Diastolic murmur heard at the point of maximal impulse

a. Systolic murmur heard at mitral area The S1 signifies the onset of ventricular systole. S2 signifies the onset of diastole. A murmur occurring between these two sounds is a systolic murmur. The mitral area is the intersection of the left fifth intercostal space and the midclavicular line. The other responses describe murmurs heard at different landmarks on the chest and/or during the diastolic phase of the cardiac cycle.

The standard policy on the cardiac unit states, "Notify the health care provider for mean arterial pressure (MAP) less than 70 mm Hg." The nurse will need to call the health care provider about the a. postoperative patient with a BP of 116/42 mm Hg. b. newly admitted patient with a BP of 150/87 mm Hg. c. patient with left ventricular failure who has a BP of 110/70 mm Hg. d. patient with a myocardial infarction who has a BP of 140/86 mm Hg.

a. postoperative patient with a BP of 116/42 mm Hg. The mean arterial pressure (MAP) is calculated using the formula MAP = (systolic BP + 2 diastolic BP)/3. The MAP for the postoperative patient in answer 3 is 67. The MAP in the other three patients is higher than 70 mm Hg.

After teaching about ways to decrease risk factors for CAD, the nurse recognizes that additional instruction is needed when the patient says a. "I would like to add weight lifting to my exercise program." b. "I can only keep my blood pressure normal with medication" c. "I can change my diet to decrease my intake of saturated fats." d. "I will change my lifestyle to reduce activities that increase my stress."

a. "I would like to add weight lifting to my exercise program." Rationale: Risk factors for coronary artery disease include elevated serum levels of lipids, elevated BP, tobacco use, physical inactivity, obesity, diabetes, metabolic syndrome, certain psychologic states, and elevated homocysteine levels. Weight lifting is not a cardioprotective exercise. An example of health-promoting regular physical activity is brisk walking (3 to 4 miles/hr) for at least 30 minutes five or more times each week.

A patient is admitted to the ICU with a diagnosis of unstable angina. Which drug(s) would the nurse expect the patient to receive (select all that apply)? a. ACE inhibitor b. Antiplatelet therapy c. Thrombolytic therapy d. Prophylactic antibiotics e. Intravenous nitroglycerin

a. ACE inhibitor b. Antiplatelet therapy e. Intravenous nitroglycerin Rationale: In addition to oxygen, several drugs may be used to treat unstable angina (UA): IV nitroglycerin, aspirin (chewable), and morphine. For patients with UA with negative cardiac biomarkers and ongoing angina, a combination of aspirin, heparin, and a glycoprotein IIb/IIIa inhibitor (e.g., eptifibatide [Integrilin]) is recommended. Angiotensin-converting enzyme (ACE) inhibitors decrease myocardial oxygen demand by producing vasodilation, reducing blood volume, and slowing or reversing cardiac remodeling.

A patient with newly discovered high BP has an average reading of 158/98 mmHg after 3 months of exercise and diet modifications. Which management strategy will be a priority for this patient? a. Medication will be required because the BP is still not at goal b. BP monitoring should continue for another 3 months to confirm a diagnosis of hypertension c. Lifestyle changes are less important, since they were not effective, and medications will be started d. More vigorous changes in the patient's lifestyle are needed for a longer time before starting medications

a. Medication will be required because the BP is still not at goal

13. The nurse hears a murmur between the S1 and S2 heart sounds at the patients left fifth intercostal space and midclavicular line. How will the nurse record this information? a. Systolic murmur heard at mitral area b. Systolic murmur heard at Erbs point c. Diastolic murmur heard at aortic area d. Diastolic murmur heard at the point of maximal impulse

a. Systolic murmur heard at mitral area

4. To auscultate for S3 or S4 gallops in the mitral area, the nurse listens with the a. bell of the stethoscope with the patient in the left lateral position. b. diaphragm of the stethoscope with the patient in a supine position. c. bell of the stethoscope with the patient sitting and leaning forward. d. diaphragm of the stethoscope with the patient lying flat on the left side.

a. bell of the stethoscope with the patient in the left lateral position.

18. The standard policy on the cardiac unit states, Notify the health care provider for mean arterial pressure (MAP) less than 70 mm Hg. The nurse will need to call the health care provider about the a. postoperative patient with a BP of 116/42. b. newly admitted patient with a BP of 150/87. c. patient with left ventricular failure who has a BP of 110/70. d. patient with a myocardial infarction who has a BP of 140/86.

a. postoperative patient with a BP of 116/42.

The nurse and unlicensed assistive personnel (UAP) on the telemetry unit are caring for four patients. Which nursing action can be delegated to the UAP? a. Teaching a patient about exercise electrocardiography b. Attaching ECG monitoring electrodes after a patient bathes c. Checking the catheter insertion site for a patient who is recovering from a coronary angiogram d. Monitoring a patient who has just returned to the unit after a transesophageal echocardiogram

b. Attaching ECG monitoring electrodes after a patient bathes UAP can be educated in standardized lead placement for ECG monitoring. Assessment of patients who have had procedures where airway maintenance (transesophageal echocardiography) or bleeding (coronary angiogram) is a concern must be done by the registered nurse (RN). Patient teaching requires RN level education and scope of practice.

When assessing a newly admitted patient, the nurse notes a murmur along the left sternal border. To acquire more information about the murmur, which action will the nurse take? a. Palpate the peripheral pulses. b. Determine the timing of the sound. c. Find the point of maximal impulse. d. Compare apical and radial pulse rates.

b. Determine the timing of the sound. Murmurs are caused by turbulent blood flow, such as occurs when blood flows through a damaged valve. Relevant information includes the position in which the murmur is heard best (e.g., sitting and leaning forward), the timing of the murmur in relation to the cardiac cycle (e.g., systole, diastole), and where on the thorax the murmur is heard best. The other information is important in the cardiac assessment but will not provide information that is relevant to the murmur.

When admitting a patient for a cardiac catheterization and coronary angiogram, which information about the patient is most important for the nurse to communicate to the health care provider? a. The patient's pedal pulses are +1. b. The patient is allergic to shellfish. c. The patient had a heart attack 1 year ago. d. The patient has not eaten anything today.

b. The patient is allergic to shellfish. The contrast dye used for the procedure is iodine based, so patients who have shellfish allergies will require treatment with medications such as corticosteroids and antihistamines before the angiogram. The other information is also communicated to the health care provider but will not require a change in the usual precardiac catheterization orders or medications.

To auscultate for S3 or S4 gallops in the mitral area, the nurse listens with the a. diaphragm of the stethoscope with the patient lying flat. b. bell of the stethoscope with the patient in the left lateral position. c. diaphragm of the stethoscope with the patient in a supine position. d. bell of the stethoscope with the patient sitting and leaning forward.

b. bell of the stethoscope with the patient in the left lateral position. Gallop rhythms generate low-pitched sounds and are most easily heard with the bell of the stethoscope. Sounds associated with the mitral valve are accentuated by turning the patient to the left side, which brings the heart closer to the chest wall. The diaphragm of the stethoscope is best to use for the higher pitched sounds such as S1 and S2.

When auscultating over the patient's abdominal aorta, the nurse hears a loud humming sound. The nurse documents this finding as a a. thrill. c. murmur. b. bruit. d. normal finding.

b. bruit. A bruit is the sound created by turbulent blood flow in an artery. Thrills are palpable vibrations felt when there is turbulent blood flow through the heart or in a blood vessel. A murmur is the sound caused by turbulent blood flow through the heart. Auscultating a bruit in an artery is not normal and indicates pathology.

A registered nurse (RN) is observing a student nurse who is doing a physical assessment on a patient. The RN will need to intervene immediately if the student nurse a. presses on the skin over the tibia for 10 seconds to check for edema. b. palpates both carotid arteries simultaneously to compare pulse quality. c. documents a murmur heard along the right sternal border as a pulmonic murmur. d. places the patient in the left lateral position to check for the point of maximal impulse.

b. palpates both carotid arteries simultaneously to compare pulse quality. The carotid pulses should never be palpated at the same time to avoid vagal stimulation, dysrhythmias, and decreased cerebral blood flow. The other assessment techniques also need to be corrected. However, they are not dangerous to the patient.

The nurse has received the laboratory results for a patient who developed chest pain 4 hours ago and may be having a myocardial infarction. The laboratory test result most helpful in indicating myocardial damage will be a. myoglobin. c. homocysteine (Hcy) b. troponins T and I. d. creatine kinase-MB (CK-MB).

b. troponins T and I. Cardiac troponins start to elevate 4 to 6 hours after myocardial injury and are highly specific to myocardium. They are the preferred diagnostic marker for myocardial infarction. Myoglobin rises in response to myocardial injury within 30 to 60 minutes. It is rapidly cleared from the body, thus limiting its use in the diagnosis of myocardial infarction. Low-density lipoprotein cholesterol is useful in assessing cardiovascular risk but is not helpful in determining whether a patient is having an acute myocardial infarction. Creatine kinase (CK-MB) is specific to myocardial injury and infarction and increases 4 to 6 hours after the infarction occurs. It is often trended with troponin levels. Homocysteine (Hcy) is an amino acid that is produced during protein catabolism. Elevated Hcy levels can be either hereditary or acquired from dietary deficiencies of vitamin B6, cobalamin (vitamin B12), or folate. Elevated levels of Hcy have been linked to a higher risk of CVD, peripheral vascular disease, and stroke.

12. When assessing a newly admitted patient, the nurse notes a murmur along the left sternal border. To document more information about the murmur, which action will the nurse take next? a. Find the point of maximal impulse. b. Determine the timing of the murmur. c. Compare the apical and radial pulse rates. d. Palpate the quality of the peripheral pulses.

b. Determine the timing of the murmur.

The nurse is caring for a patient who is 2 days post MI. The patient reports that she is experiencing chest pain. She states, "It hurts when I take a deep breath." Which action would be a priority? a. Notify the physician STAT and obtain a 12-lead ECG b. Obtain vital signs and auscultate for a pericardial friction rub c. Apply high-flow O2 by face mask and auscultate breath sounds d. Medicate the patient with PRN analgesic and reevaluate in 30 minutes

b. Obtain vital signs and auscultate for a pericardial friction rub Rationale: Acute pericarditis is inflammation of the visceral and/or parietal pericardium. It often occurs 2 to 3 days after an acute myocardial infarction. Chest pain may vary from mild to severe and is aggravated by inspiration, coughing, and movement of the upper body. Sitting in a forward position often relieves the pain. The pain is usually different from pain associated with a myocardial infarction. Assessment of the patient with pericarditis may reveal a friction rub over the pericardium.

21. The nurse and unlicensed assistive personnel (UAP) on the telemetry unit are caring for four patients. Which nursing action can be delegated to the UAP? a. Teaching a patient scheduled for exercise electrocardiography about the procedure b. Placing electrodes in the correct position for a patient who is to receive ECG monitoring c. Checking the catheter insertion site for a patient who is recovering from a coronary angiogram d. Monitoring a patient who has just returned to the unit after a transesophageal echocardiogram

b. Placing electrodes in the correct position for a patient who is to receive ECG monitoring

19. When admitting a patient for a cardiac catheterization and coronary angiogram, which information about the patient is most important for the nurse to communicate to the health care provider? a. The patients pedal pulses are +1. b. The patient is allergic to shellfish. c. The patient had a heart attack a year ago. d. The patient has not eaten anything today.

b. The patient is allergic to shellfish.

7. A patient is scheduled for a cardiac catheterization with coronary angiography. Before the test, the nurse informs the patient that a. it will be important to lie completely still during the procedure. b. a flushed feeling may be noted when the contrast dye is injected. c. monitored anesthesia care will be provided during the procedure. d. arterial pressure monitoring will be required for 24 hours after the test.

b. a flushed feeling may be noted when the contrast dye is injected.

In teaching a patient about coronary artery disease, the nurse explains that the changes that occur in this disorder include (select all that apply) a. diffuse involvement of plaque formation in coronary veins b. abnormal levels of cholesterol, especially low-density lipoproteins c. accumulation of lipid and fibrous tissue within the coronary arteries d. development of angina due to a decreased blood supply to the heart muscle e. chronic vasoconstriction of coronary arteries leading to permanent vasospasm

b. abnormal levels of cholesterol, especially low-density lipoproteins c. accumulation of lipid and fibrous tissue within the coronary arteries d. development of angina due to a decreased blood supply to the heart muscle Rationale: Atherosclerosis is the major cause of coronary artery disease (CAD) and is characterized by a focal deposit of cholesterol and lipids, primarily within the intimal wall of the artery. The endothelial lining of the coronary arteries becomes inflamed from the presence of unstable plaques and the oxidation of low-density lipoprotein (LDL) cholesterol. Fibrous plaque causes progressive changes in the endothelium of the arterial wall. The result is a narrowing of the vessel lumen and a reduction in blood flow to the myocardial tissue.

While obtaining subjective assessment date from a patient with hypertension, the nurse recognizes that a modifiable risk factor for the development of hypertension is: a. a low-calcium diet b. excessive alcohol consumption c. a family history of hypertension d. consumption of a high-protein diet

b. excessive alcohol consumption

14. A registered nurse (RN) is observing a student nurse who is doing a physical assessment on a patient. The RN will need to intervene immediately if the student nurse a. presses on the skin over the tibia for 10 seconds to check for edema. b. palpates both carotid arteries simultaneously to compare pulse quality. c. documents a murmur heard along the right sternal border as a pulmonic murmur. d. places the patient in the left lateral position to check for the point of maximal impulse.

b. palpates both carotid arteries simultaneously to compare pulse quality.

A hospitalized patient with a history of chronic stable angina tells the nurse that she is having chest pain. The nurse bases his actions on the knowledge that ischemia a. will always progress to myocardial infarction b. will be relieved by rest, nitroglycerin, or both c. indicates that irreversible myocardial damage is occurring d. is frequently associated with vomiting and extreme fatigue

b. will be relieved by rest, nitroglycerin, or both Rationale: Chronic stable angina is chest pain that occurs intermittently over a long period with the same pattern of onset, duration, and intensity of symptoms. The chest pain is relieved by rest or by rest and medication (e.g., nitroglycerin). The ischemia is transient and does not cause myocardial damage.

When the nurse is monitoring a patient who is undergoing exercise (stress) testing on a treadmill, which assessment finding requires the most rapid action by the nurse? a. Patient complaint of feeling tired b. Sinus tachycardia at a rate of 110 beats/min c. Inversion of T waves on the electrocardiogram d. Blood pressure (BP) increase from 134/68 to 150/80 mm Hg

c. Inversion of T waves on the electrocardiogram ECG changes associated with coronary ischemia (such as T-wave inversions and ST segment depression) indicate that the myocardium is not getting adequate O2 delivery and that the exercise test should be terminated immediately. Increases in BP and heart rate are normal responses to aerobic exercise. Feeling tired is also normal as the intensity of exercise increases during the stress testing.

A transesophageal echocardiogram (TEE) is ordered for a patient with possible endocarditis. Which action included in the standard TEE orders will the nurse need to accomplish first? a. Start an IV line. c. Place the patient on NPO status. b. Start O2 per nasal cannula. d. Give lorazepam (Ativan) 1 mg IV.

c. Place the patient on NPO status. The patient will need to be NPO for 6 hours preceding the TEE, so the nurse should place the patient on NPO status as soon as the order is received. The other actions also will need to be accomplished but not until just before or during the procedure.

Which action will the nurse implement for a patient who arrives for a calcium-scoring CT scan? a. Insert an IV catheter. b. Administer oral sedative medications. c. Teach the patient about the procedure. d. Confirm that the patient has been fasting.

c. Teach the patient about the procedure. The nurse will need to teach the patient that the procedure is rapid and involves little risk. None of the other actions are necessary.

Which information obtained by the nurse who is admitting the patient for magnetic resonance imaging (MRI) will be important to report to the health care provider before the MRI? a. The patient has an allergy to shellfish. b. The patient has a history of atherosclerosis. c. The patient has a permanent cardiac pacemaker. d. The patient took the prescribed heart medications today

c. The patient has a permanent cardiac pacemaker. MRI is contraindicated for patients with implanted metallic devices such as pacemakers. The other information does not affect whether or not the patient can have an MRI.

A patient is scheduled for a cardiac catheterization with coronary angiography. Before the test, the nurse informs the patient that a. it will be important not to move at all during the procedure. b. monitored anesthesia care will be provided during the procedure. c. a flushed feeling may be noticed when the contrast dye is injected. d. arterial pressure monitoring will be required for 24 hours after the test.

c. a flushed feeling may be noticed when the contrast dye is injected. A sensation of warmth or flushing is common when the contrast material is injected, which can be anxiety producing unless it has been discussed with the patient. The patient may receive a sedative drug before the procedure, but monitored anesthesia care is not used. Arterial pressure monitoring is not routinely used after the procedure to monitor blood pressure. The patient is not immobile during cardiac catheterization and may be asked to cough or take deep breaths.

During a physical examination of an older patient, the nurse palpates the point of maximal impulse (PMI) in the sixth intercostal space lateral to the left midclavicular line. The best follow-up action for the nurse to take will be to a. ask about risk factors for atherosclerosis. b. determine family history of heart disease. c. assess for symptoms of left ventricular hypertrophy. d. auscultate carotid arteries for the presence of a bruit.

c. assess for symptoms of left ventricular hypertrophy. The PMI should be felt at the intersection of the fifth intercostal space and left midclavicular line. A PMI located outside these landmarks indicates possible cardiac enlargement, such as with left ventricular hypertrophy (LVH). The other assessments are part of a general cardiac assessment but do not represent follow-up for LVH. Cardiac enlargement is not necessarily associated with atherosclerosis or carotid artery disease.

23. When the nurse is screening patients for possible peripheral arterial disease, indicate where the posterior tibial artery will be palpated. a. 1 b. 2 c. 3 d. 4

c. 3

A patient is recovering from an uncomplicated MI. Which rehabilitation guideline is a priority to include in the teaching plan? a. Refrain from sexual activity for a minimum of 3 weeks b. Plan a diet program that aims for a 1- to 2-pound weight loss per week c. Begin an exercise program that aims for at least five 30-minute sessions per week d. Consider the use of erectile agents and prophylactic NTG before engaging in sexual activity

c. Begin an exercise program that aims for at least five 30-minute sessions per week Rationale: Physical activity should be regular, rhythmic, and repetitive, with the use of large muscles to build up endurance (e.g., walking, cycling, swimming, rowing). Physical activity sessions should be at least 30 minutes long. Instruct the patient to begin slowly at personal tolerance (perhaps only 5 to 10 minutes) and build up to 30 minutes.

15. Which action will the nurse implement for a patient who arrives for a calcium-scoring CT scan? a. Insert an IV catheter. b. Administer oral sedative medications. c. Teach the patient about the procedure. d. Confirm that the patient has been fasting.

c. Teach the patient about the procedure.

16. Which information obtained by the nurse who is admitting the patient for magnetic resonance imaging (MRI) will be most important to report to the health care provider before the MRI? a. The patient has an allergy to shellfish. b. The patient has a history of atherosclerosis. c. The patient has a permanent ventricular pacemaker. d. The patient took all the prescribed cardiac medications today.

c. The patient has a permanent ventricular pacemaker.

The most common finding in individuals at risk for sudden cardiac death is a. aortic valve disease b. mitral valve disease c. left ventricular dysfunction d. atherosclerotic heart disease

c. left ventricular dysfunction Rationale: Left ventricular dysfunction (ejection fraction less than 30%) and ventricular dysrhythmias after myocardial infarction are the strongest predictors of sudden cardiac death (SCD).

To determine the effects of therapy for a patient who is being treated for heart failure, which laboratory test result will the nurse plan to review? a. Troponin c. Low-density lipoprotein (LDL) b. Homocysteine (Hcy) d. B-type natriuretic peptide (BNP)

d. B-type natriuretic peptide (BNP) Increased levels of BNP are a marker for heart failure. The other laboratory results would be used to assess for myocardial infarction (troponin) or risk for coronary artery disease (Hcy and LDL).

While doing the hospital admission assessment for a thin older adult, the nurse observes pulsation of the abdominal aorta in the epigastric area. Which action should the nurse take next? a. Teach the patient about aneurysms. b. Notify the hospital rapid response team. c. Instruct the patient to remain on bed rest. d. Document the finding in the patient chart.

d. Document the finding in the patient chart. Visible pulsation of the abdominal aorta is commonly observed in the epigastric area for thin individuals. The nurse should simply document the finding in the admission assessment. Unless there are other abnormal findings (such as a bruit, pain, or hyper/hypotension) associated with the pulsation, the other actions are not necessary.

The nurse notes that a patient who was admitted with heart failure has jugular venous distention (JVD) when lying flat in bed. Which follow-up action should the nurse take next? a. Obtain vital signs, including oxygen saturation. b. Have the patient perform the Valsalva maneuver. c. Document this JVD finding in the patient's record. d. Observe for JVD with the patient elevated 45 degrees.

d. Observe for JVD with the patient elevated 45 degrees. When the patient is lying flat, the jugular veins are at the level of the right atrium, so JVD is a common (but not a clinically significant) finding. Obtaining vital signs and oxygen saturation is not warranted at this point. JVD is an expected finding when a patient performs the Valsalva maneuver because right atrial pressure increases. JVD that persists when the patient is sitting at a 30- to 45-degree angle or greater is significant. The nurse will document the JVD in the medical record if it persists when the head is elevated.

The nurse is reviewing the 12-lead electrocardiograph (ECG) for a healthy 74-yr-old patient who is having an annual physical examination. What finding is of most concern to the nurse? a. A right bundle-branch block. c. The QRS duration is 0.13 seconds. b. The PR interval is 0.21 seconds. d. The heart rate (HR) is 41 beats/min.

d. The heart rate (HR) is 41 beats/min. The resting HR does not change with aging, so the decrease in HR requires further investigation. Bundle-branch block and slight increases in PR interval or QRS duration are common in older individuals because of increases in conduction time through the AV node, bundle of His, and bundle branches.

A 74-yr-old patient has just arrived in the emergency department. After assessment reveals a pulse deficit of 46 beats, the nurse will anticipate that the patient may require a. emergent cardioversion. b. a cardiac catheterization. c. hourly blood pressure (BP) checks. d. electrocardiographic (ECG) monitoring.

d. electrocardiographic (ECG) monitoring. Pulse deficit is a difference between simultaneously obtained apical and radial pulses. It indicates that there may be a cardiac dysrhythmia that would best be detected with ECG monitoring. Frequent BP monitoring, cardiac catheterization, and emergent cardioversion are used for diagnosis and/or treatment of cardiovascular disorders but would not be as helpful in determining the immediate reason for the pulse deficit.

The nurse teaches the patient being evaluated for rhythm disturbances with a Holter monitor to a. connect the recorder to a computer once daily. b. exercise more than usual while the monitor is in place. c. remove the electrodes when taking a shower or tub bath. d. keep a diary of daily activities while the monitor is worn.

d. keep a diary of daily activities while the monitor is worn. The patient is instructed to keep a diary describing daily activities while Holter monitoring is being accomplished to help correlate any rhythm disturbances with patient activities. Patients are taught that they should not take a shower or bath during Holter monitoring and that they should continue with their usual daily activities. The recorder stores the information about the patient's rhythm until the end of the testing, when it is removed and the data are analyzed.

6. While doing the admission assessment for a thin 76-year-old patient, the nurse observes pulsation of the abdominal aorta in the epigastric area. Which action should the nurse take? a. Teach the patient about aneurysms. b. Notify the hospital rapid response team. c. Instruct the patient to remain on bed rest. d. Document the finding in the patient chart.

d. Document the finding in the patient chart.

17. When the nurse is monitoring a patient who is undergoing exercise (stress) testing on a treadmill, which assessment finding requires the most rapid action by the nurse? a. Patient complaint of feeling tired b. Pulse change from 87 to 101 beats/minute c. Blood pressure (BP) increase from 134/68 to 150/80 mm Hg d. Newly inverted T waves on the electrocardiogram

d. Newly inverted T waves on the electrocardiogram

8. While assessing a patient who was admitted with heart failure, the nurse notes that the patient has jugular venous distention (JVD) when lying flat in bed. Which action should the nurse take next? a. Document this finding in the patients record. b. Obtain vital signs, including oxygen saturation. c. Have the patient perform the Valsalva maneuver. d. Observe for JVD with the patient upright at 45 degrees.

d. Observe for JVD with the patient upright at 45 degrees.

A major consideration in the management of the other adult with hypertension is to: a. prevent primary hypertension from converting to secondary hypertension b. recognize that the older adult is less likely to adhere to the drug therapy regimen than a younger adult c. ensure that the patient receives larger initial doses of antihypertensive drugs because of impaired absorption d. use careful technique in assessing the BP of the patient because of the possible presence of ab auscultatory gap

d. use careful technique in assessing the BP of the patient because of the possible presence of ab auscultatory gap

A nurse is caring for a client in a clinic who asks the nurse why her provider prescribed 1 aspirin per day. which of the following is an appropriate response by the nurse? 1. "Aspirin reduces the formation of blood clots that could cause a heart attack." 2. "Aspirin relieves the pain due to MI." 3. "Aspirin dissolves clots that are forming in your coronary arteries." 4. "Aspirin relieves headaches that are caused by other medications."

1. Aspirin reduces the formation of blood clots that could cause a heart attack-aspirin decreases Plt aggregation that can = MI

ER nurse is assessing PT with bradydysrhythmia. Which finding should he expect? 1. Confusion 2. Friction Rub 3. HTN 4. Dry skin

1. Confusion- as a result of decreased tissue perfusion

PT just had an insertion of a temporary venous pacemaker via the femoral artery and its set to VVI rate of 70/min. Which finding should the RN report? Select all 1. Cool and clammy foot with cap refill of 5 sec 2. Observed pacing spike followed by a QRS complex 3. Twitching of intercostal muscle 4. HR of 84 5. BP 104/62

1. Cool and clammy foot with cap refill of 5 sec-indicates femoral hematoma r/t insertion of the lead wires 3. Twitching of intercostal muscle - indicates lead wire perforation and stimulation of the diaphragm

A nurse is caring for a client who has severe PAD. The nurse should expect that the client will sleep most comfortably in which of the following positions? 1. With the affected limb hanging from the bed 2. With the affected limb elevated on pillows 3. With the head of the bed raised 4. In a side-lying, recumbent position

1. With the affected limb hanging from the bed-dependent position will relieve pain

You are providing teaching to a PT who is 2 days post op following a heart transplant. Which following statements should the nurse include in the teaching? 1. You may no longer be able to feel chest pain 2. Your level of activity intolerance will not change 3. after 6 months you will no longer need to restrict you Na 4. You will be able to stop taking immunosuppressants after 12 months

1. You may no longer be able to feel chest pain- transplant PTs no longer feel chest pain due to denervation of the heart

A nurse is admitting a client who has a suspected MI and a history of angina. which of the following findings will help the nurse distinguish angina from an MI? 1. angina can be relieved with rest and nitroglycerin 2. the pain of an MI resolves in less than 15 min. 3. the type of activity that causes an MI can be identified. 4. angina can occur for longer than 30 min.

1. angina can be relieved with rest and nitroglycerin

A nurse is caring for a client and reviewing a new prescription for an afterload-reducing medication. The nurse should recognize that this medication is administered for which of the following types of shock? 1. cardiogenic 2. obstructive 3. hypovolemic 4. distributive

1. cardiogenic- reducing after load will allow the heart to pump more effectively. **2. obstructive-high after load is due to obstruction so afterload reducers will not work 3. hypovolemic-fluid replacement and reduction of further fluid loss are the focus of hypovolemic shock 4. distributive-afterload reducing meds should not be given due to already decreased after load

A nurse is assessing a client who has splinter hemorrhages in her nail beds and reports a fever. For which of the following conditions is the client at risk? 1. infective endocarditis 2. pericarditis 3. myocarditis 4. rheumatic endocarditis

1. infective endocarditis- splinter hemorrhages are r/t infective endocarditis

Which of these are expected findings of pulmonary edema? Select all 1.Tachypnea 2.Persistent cough 3.Increase urinary output 4.Thick yellow sputum 5.Orthopnea

1.Tachypnea- expected 2.Persistent cough-will be frothy pink and indicates pulm. edema 5.Orthopnea- Expected **3.Increase UOP- would be decreased 4.Thick yellow sputum- would be pink and frothy sputum

You're completing an admisson on PT who has a HX: mitral valve insufficiency. Which is expected? 1. Hoarseness 2. Petechiae 3. Crackles in lungs 4. Splenomegaly

3. Crackles in lungs- expected due to pulmonary congestion

You are caring for PT when he begins to go into SVT HR is 200-210 a minute what intervention do you anticipate? 1. Delivery of a precordial thump 2. Vagal stimulation 3. Adminster atropine IV 4. Defibrillation

2. Vagal stimulation- this can help return HR to normal sinus rhythm temporarily

A nurse is caring for a client who has chronic venous insufficiency. The provider prescribed thigh high compressions stockings. the nurse should instruct the client to 1. Massage both legs firmly with lotion prior to applying the stockings 2. apply the stockings in the morning upon awakening and before getting out of bed 3. roll the stockings down to the knees if they will not stay up on the thighs 4. remove the stockings while out of bed for 1 hr, 4 times a day to allow to rest

2. apply the stockings in the morning upon awakening and before getting out of bed-reduces venous stasis and assists w/ venous return legs are less edema at AM **1. Massage both legs firmly with lotion prior to applying the stockings-massage can dislodge a clot 3. roll the stockings down to the knees if they will not stay up on the thighs- can restrict circulation=edema 4. remove the stockings while out of bed for 1 hr, 4 times a day to allow to rest- removed before bedtime to provide continuous venous support

A nurse is providing teaching for a client who is to have a bone marrow biopsy of the iliac crest. Which of the following statements made by the client indicates a need for further teaching? 1. "Cancer can be detected in the fluid being tested." 2. "I will feel heavy pressure sensation in my hip bone." 3. "The type of antibiotic I need to take can be determined by the test." 4. "I will be awake during the procedure."

3. "The type of antibiotic I need to take can be determined by the test."-culture/sens. will determine what antibiotic will be needed not biopsy

RN is caring for a PT who had an onset of chest pain 24hr ago. The RN should recognize that an increase in which of the following is diagnostic of an MI? 1. Myoglobin 2. C-reactive protein 3. Creatine Kinase MB 4. Homocysteine

3. Creatine Kinase MB- specific to see myocardium and is elevated when the muscle in injured peaks at 24 hr after chest pain

You are reviewing the lab results of several PTs who have PAD. You plan to provide dietary teaching for the PT with which of the following? 1. Cholesterol 180mg, HDL 70mg, LDL 90mg 2.Cholesterol 185mg, HDL 50mg, LDL 120mg 3.Cholesterol 190mg, HDL 25mg, LDL 160mg 4.Cholesterol 195mg, HDL 55mg, LDL 125mg

3. Cholesterol 190mg, HDL 25mg, LDL 160mg ** cholesterol should be less than 200 HDL above 40 LDL less than 100

You are admitting a Pt with a leg ulcer and a HX: of diabetes mellitus. What focused assessment would help differentiate between an arterial ulcer and a venous stasis ulcer? 1. Explore PTs HX: of PAD 2. Note the presence or absence of pain at the site 3. Inquire about the presence or absence of claudication 4. Ask the PT if has had a recent infection.

3. Inquire about the presence or absence of claudication-arterial will have claudication venous will not

A nurse in the emergency department is assisting with the admission of a client who has a possible dissecting abdominal aortic aneurysm. Which of the following is the priority nursing intervention? 1. administer pain medication as prescribed 2. ensure a warm environment 3. administer IV fluids as prescribed 4. initiate a 12-lead ECG

3. administer IV fluids as prescribed-ABCs big risk is inadequate circulatory volume. treat w/ fluids

A nurse in the emergency department is caring for a client who has a allergic reaction to a bee sting. The client is experiencing wheezing and swelling of the tongue. Which of the following medications should the nurse expect to administer first? 1. methylprednisolone (Solu-Medrol) IV bolus 2. diphenhydramine (benadryl) subcutaneously 3. epinephrine (adrenaline) IV 4. albuterol (proventil) inhaler

3. epinephrine (adrenaline) IV-ABCs rapid acting to promote oxygenation

A nurse is admitting a client with a suspected occlusion of a graft of the abdominal aorta. Which of the following is an expected clinical finding? 1. increased urine output 2. bounding pedal pulse 3. increased abdominal grith 4. redness of the lower extremities

3. increased abdominal grith- expected **1. increased urine output-would be decreased 2. bounding pedal pulse-would be decreased or absent 4. redness of the lower extremities- Pallor/ cyanosis would be expected

A nurse is presenting a community education program on recommended lifestyle changes to prevent angina and MI. which of the following changes should the nurse recommend be made first? 1. diet modification 2. relaxation exercises 3. smoking cessation 4. taking omega-3 capsules

3. smoking cessation- ABCs O2 is priority Nicotine causes vasoconstriction

A nurse is instructiong a client who has angina about a new prescription for metoprolol tartrate. which of the following statements by the client indicates understanding of the teaching? 1. "I should place the tablet under my tongue." 2. "I should have my clotting time checked weekly." 3. "I will report any ringing in my ears." 4. "I will call my doctor if my pulse is less than 60."

4. "I will call my doctor if my pulse is less than 60."

You are caring for a PT who had an anterior MI. PTs hx shows she is 1 wk post op open cholecystectomy the RN knows that which intervention is contraindicated? 1. Administering IV morphine 2. giving O2 at 2L NC 3. Helping client to BSC 4. Assisting in thrombolytic therapy

4. Assisting in thrombolytic therapy- having surgery within 3 wks is a contraindication for thrombolytic therapy

What medications should the nurse expect to include in the teaching plan to decrease the risk of cardiovascular events and death for PAD patients (select all that apply.)? Select all that apply. A) Ramipril (Altace) B) Cilostazol (Pletal) C) Simvastatin (Zocor) D) Clopidogrel (Plavix) E) Warfarin (Coumadin) F) Aspirin (acetylsalicylic acid)

A) Ramipril (Altace) C) Simvastatin (Zocor) F) Aspirin (acetylsalicylic acid)

A nurse in a clinic is caring for a client who has suspected anemia. The nurse should anticipate a prescription from the provider for which of the following tests? 1. INR 2. platelet count 3. WBC count 4. Hgb

4. Hgb- used to confirm anemia **1. INR- for warfarin therapy 2. platelet count-identfies altered immune response 3. WBC count- identifies infection

You are caring for a PT with angina and is scheduled for a stress test at 1100 which Pt statement requires rescheduling? 1. Im still hungry after my cereal this morning 2. I didn't take my heart pills b/c Dr. said not to 3. I have had chest pain a couple times after seeing the Dr. last week 4. I smoked a cigarette this morning to calm my nerves about this test

4. I smoked a cigarette this morning to calm my nerves about this test--this can change the outcome and place you at additional risk

When performing a cardiac assessment what is the point of maximal impulse? 1. 2nd intercostal space right to the sternum 2. 2nd intercostal space left to the sternum 3. 5th intercostal space to the left of thermal border 4. Left 5th intercostal space in the midclavicular line

4. Left 5th intercostal space in the midclavicular line-this is best to hear the apex of the heart which is considered maximal impulse

Assessment of a patient's peripheral IV site reveals that phlebitis has developed over the past several hours. Which intervention should the nurse implement first? A) Remove the patient's IV catheter. B) Apply an ice pack to the affected area. C) Decrease the IV rate to 20 to 30 mL/hr. D) Administer prophylactic anticoagulants.

A) Remove the patient's IV catheter.

You are reviewing the chart of a PT who is receiving heparin therapy for a DVT. Which intervention should the RN anticipate taking if the PTs aPTT is 96 seconds? 1. Increase heparin infusion rate by 2mL/hr 2. Continue monitor the heparin infusion as prescribed 3. Request a prothrombin time PT 4. Stop the heparin infusion

4. Stop the heparin infusion- pt is above the therapeutic range of 1-5-2 times the control. should discontinue to prevent harm

A nurse is preforming a physical assessment of a client who has chronic peripheral arterial disease (PAD). which of the following is an expected finding? 1. Edema around the clients ankles and feet 2. ulceration around the clients medial malleoli 3. scaling eczema of the clients lower legs with stasis dermatitis 4. pallor on elevation of the clients limbs and rubor when when his limbs are dependent

4. pallor on elevation of the clients limbs and rubor when when his limbs are dependent- seen in chronic PAD

A 32-yr-old woman is prescribed diltiazem (Cardizem) for Raynaud's phenomenon. To evaluate the effectiveness of the medication, which assessment will the nurse perform? A) Improved skin turgor B) Decreased cardiac rate C) Improved finger perfusion D) Decreased mean arterial pressure

C) Improved finger perfusion

To determine the effects of therapy for a patient who is being treated for heart failure, which laboratory test result will the nurse plan to review? a. Troponin c. Low-density lipoprotein (LDL) b. Homocysteine (Hcy) d. B-type natriuretic peptide (BNP)

ANS: D Increased levels of BNP are a marker for heart failure. The other laboratory results would be used to assess for myocardial infarction (troponin) or risk for coronary artery disease (Hcy and LDL).

The registered nurse (RN) is caring for a patient with a hypertensive crisis who is receiving sodium nitroprusside . Which nursing action can the nurse delegate to an experienced licensed practical/vocational nurse (LPN/LVN)? a. Evaluate effectiveness of nitroprusside therapy on blood pressure (BP). b. Assess the patient's environment for adverse stimuli that might increase BP. c. Titrate nitroprusside to decrease mean arterial pressure (MAP) to 115 mm Hg. d. Set up the automatic noninvasive BP machine to take readings every 15 minutes.

ANS: D LPN/LVN education and scope of practice include the correct use of common equipment such as automatic noninvasive blood pressure machines. The other actions require advanced nursing judgment and education, and should be done by RNs.

Which action could the nurse delegate to unlicensed assistive personnel (UAP) trained as electrocardiogram (ECG) technicians working on the cardiac unit? a. Select the best lead for monitoring a patient with an admission diagnosis of Dressler syndrome. b. Obtain a list of herbal medications used at home while admitting a new patient with pericarditis. c. Teach about the need to monitor the weight daily for a patient who has hypertrophic cardiomyopathy. d. Check the heart monitor for changes in rhythm while a patient who had a valve replacement ambulates.

ANS: D Under the supervision of registered nurses (RNs), UAP check the patient's cardiac monitor and obtain information about changes in heart rate and rhythm with exercise. Teaching and obtaining information about home medications (prescribed or complementary) and selecting the best leads for monitoring patients require more critical thinking and should be done by the RN.

The nurse is caring for a patient admitted with a history of hypertension. The patient's medication history includes hydrochlorothiazide (Hydrodiuril) daily for the past 10 years. Which parameter would indicate the optimal intended effect of this drug therapy? A Weight loss of 2 lb B Blood pressure 128/86 C Absence of ankle edema D Output of 600 mL per 8 hours

B Blood pressure 128/86 Hydrochlorothiazide may be used alone as monotherapy to manage hypertension or in combination with other medications if not effective alone. After the first few weeks of therapy, the diuretic effect diminishes, but the antihypertensive effect remains. Since the patient has been taking this medication for 10 years, the most direct measurement of its intended effect would be the blood pressure.

The nurse admits a 73-year-old male patient with dementia for treatment of uncontrolled hypertension. The nurse will closely monitor for hypokalemia if the patient receives which medication? A Clonidine (Catapres) B Bumetanide (Bumex) C Amiloride (Midamor) D Spironolactone (Aldactone)

B Bumetanide (Bumex) Bumetanide is a loop diuretic. Hypokalemia is a common adverse effect of this medication. Amiloride is a potassium-sparing diuretic. Spironolactone is an aldosterone-receptor blocker. Hyperkalemia is an adverse effect of both amiloride and spironolactone. Clonidine is a central-acting α-adrenergic antagonist and does not cause electrolyte abnormalities.

A 25-year-old patient with a group A streptococcal pharyngitis does not want to take the antibiotics prescribed. What should the nurse tell the patient to encourage the patient to take the medications and avoid complications of the infection? A. "The complications of this infection will affect the skin, hair, and balance." B. "You will not feel well if you do not take the medicine and get over this infection." C. "Without treatment, you could get rheumatic fever, which can lead to rheumatic heart disease." D. "You may not want to take the antibiotics for this infection, but you will be sorry if you do not."

C. "Without treatment, you could get rheumatic fever, which can lead to rheumatic heart disease." Rheumatic fever (RF) is not common because of effective use of antibiotics to treat streptococcal infections. Without treatment, RF can occur and lead to rheumatic heart disease, especially in young adults. The complications do not include hair or balance. Saying that the patient will not feel well or that the patient will be sorry if the antibiotics are not taken is threatening to the patient and inappropriate for the nurse to say.

A 72-year-old man with a history of aortic stenosis is admitted to the emergency department. He reports severe left-sided chest pressure radiating to the jaw. Which medication, if ordered by the health care provider, should the nurse question? A. Aspirin B. Oxygen C. Nitroglycerin D. Morphine sulfate

C. Nitroglycerin Aspirin, oxygen, nitroglycerin, and morphine sulfate are all commonly used to treat acute chest pain suspected to be caused by myocardial ischemia. However, nitroglycerin should be used cautiously or avoided in patients with aortic stenosis as a significant reduction in blood pressure may occur. Chest pain can worsen because of a drop in blood pressure.

What nursing action should the nurse prioritize during the care of a patient who has recently recovered from rheumatic fever? A. Teach the patient how to manage his or her physical activity. B. Teach the patient about the need for ongoing anticoagulation. C. Teach the patient about the need for continuous antibiotic prophylaxis. D. Teach the patient about the need to maintain standard infection control procedures.

C. Teach the patient about the need for continuous antibiotic prophylaxis. Patients with a history of rheumatic fever frequently require ongoing antibiotic prophylaxis, an intervention that necessitates education. This consideration is more important than activity management in preventing recurrence. Anticoagulation is not indicated in this patient population. Standard precautions are indicated for all patients.

A 44-year-old man is diagnosed with hypertension and receives a prescription for benazepril (Lotensin). After the nurse teaches him about the medication, which statement by the patient indicates his correct understanding? A "If I take this medication, I will not need to follow a special diet." B "It is normal to have some swelling in my face while taking this medication." C "I will need to eat foods such as bananas and potatoes that are high in potassium." D "If I develop a dry cough while taking this medication, I should notify my doctor."

D "If I develop a dry cough while taking this medication, I should notify my doctor." Benazepril is an angiotensin-converting enzyme inhibitor. The medication inhibits breakdown of bradykinin, which may cause a dry, hacking cough. Other adverse effects include hyperkalemia. Swelling in the face could indicate angioedema and should be reported immediately to the prescriber. Patients taking drug therapy for hypertension should also attempt lifestyle modifications to lower blood pressure such as a reduced-sodium diet.

In reviewing medication instructions with a patient being discharged on antihypertensive medications, which statement would be most appropriate for the nurse to make when discussing guanethidine (Ismelin)? A "A fast heart rate is a side effect to watch for while taking guanethidine." B "Stop the drug and notify your doctor if you experience any nausea or vomiting." C "Because this drug may affect the lungs in large doses, it may also help your breathing." D "Make position changes slowly, especially when rising from lying down to a standing position."

D "Make position changes slowly, especially when rising from lying down to a standing position." Guanethidine is a peripheral-acting α-adrenergic antagonist and can cause marked orthostatic hypotension. For this reason, the patient should be instructed to rise slowly, especially when moving from a recumbent to a standing position. Support stockings may also be helpful. Tachycardia or lung effects are not evident with guanethidine.

The nurse is preparing to administer a scheduled dose of enoxaparin (Lovenox) 30 mg subcutaneously. What should the nurse do to administer this medication correctly? A) Remove the air bubble in the prefilled syringe. B) Aspirate before injection to prevent IV administration. C) Rub the injection site after administration to enhance absorption. D) Pinch the skin between the thumb and forefinger before inserting the needle.

D) Pinch the skin between the thumb and forefinger before inserting the needle.

What is a priority nursing intervention in the care of a patient with a diagnosis of chronic venous insufficiency (CVI)? A) Application of topical antibiotics to venous ulcers B) Maintaining the patient's legs in a dependent position C) Administration of oral and/or subcutaneous anticoagulants D) Teaching the patient the correct use of compression stockings

D) Teaching the patient the correct use of compression stockings

The patient with pericarditis is complaining of chest pain. After assessment, which intervention should the nurse expect to implement to provide pain relief? A. Corticosteroids B. Morphine sulfate C. Proton pump inhibitor D. Nonsteroidal antiinflammatory drugs

D. Nonsteroidal antiinflammatory drugs Nonsteroidal antiinflammatory drugs (NSAIDs) will control pain and inflammation. Corticosteroids are reserved for patients already taking corticosteroids for autoimmune conditions or those who do not respond to NSAIDs. Morphine is not necessary. Proton pump inhibitors are used to decrease stomach acid to avoid the risk of GI bleeding from the NSAIDs.

An 80-year-old patient with uncontrolled type 1 diabetes mellitus is diagnosed with aortic stenosis. When conservative therapy is no longer effective, the nurse knows that the patient will need to do or have what done? A. Aortic valve replacement B. Take nitroglycerin for chest pain. C. Open commissurotomy (valvulotomy) procedure D. Percutaneous transluminal balloon valvuloplasty (PTBV) procedure

D. Percutaneous transluminal balloon valvuloplasty (PTBV) procedure

What is cardioversion?

Delivery of direct countercheck to the heart synced with QRS complex. **used for atria dysrhythmias, SVT, Ventricular tachycardia with pulse. TREATMENT OF CHOICE FOR PTs WHO ARE SYMPTOMATIC

What part of the ECG would you look for atrial depolarization?

P wave - atrial depolarization QRS- vetricular depolarization T- Ventricular reploarization

Electrical management for bradycardia would be?

Pacemaker for any rhythm less than 60/min

What are complications of Heart failure?

Pulmonary edema Cardiogenic shock Cardiac tamponade

Complications of Cardioversion?

Pulmonary embolism- (AEB dyspnea, chest pain, air hunger, decreased sat) Stoke(CVA)- (AEB decrease LOC, slurred speech, muscle weakness/paralysis) MI- (AEB chest pain and elevated or depressed ST segment)

Which BP-regulating mechanism(s) can result in the development of hypertension if defective (select all that apply)? a. Release of norepinephrine b. Secretion of prostaglandins c. Stimulation of the sympathetic nervous system d. Stimulation of the parasympathetic nervous system e. Activation of the renin-angiotensin-aldosterone system

a, c, e

The nurse is reviewing the laboratory results for newly admitted patients on the cardiovascular unit. Which laboratory result is most important to communicate as soon as possible to the health care provider? a. High troponin I level b. Increased triglyceride level c. Very low homocysteine level d. Elevated high-sensitivity C-reactive protein level

a. High troponin I level The elevation in troponin I indicates that the patient has had an acute myocardial infarction. Further assessment and interventions are indicated. The other laboratory results are indicative of increased risk for coronary artery disease but are not associated with acute cardiac problems that need immediate intervention.


Conjuntos de estudio relacionados

CALIFORNIA REAL ESTATE PROPERTY MANAGEMENT: CHAPTER 6: MARKETING. TERMS AND QUIZ

View Set

Managerial Accounting - Chapter 2

View Set

TExES FULL Practice Study Questions

View Set

Brunner & Suddarth: Test Bank (Chapter 67)

View Set

Chapter 1 The First American Way of War

View Set